Você está na página 1de 119

https://www.facebook.com/groups/LLB.

GujUni/ Page - 1 of 119

CORE COURSE
210 INTERPRETATION OF STATUTES & PRINCIPLES OF LEGISLATION

Imp ---> There will not be any updates/ versions before Apr-2017 Uni exams.
Best wishes to one and all. Cheers.
Stay connected using facebook.
Source : Public domain print/ internet contents. URLs of some such resources are listed
herein. Credits/ copyrights duly acknowledged.
04-Mar-2017. Exam centric version-1.3 compiled by ketan.bhatt@iitbombay.org in
academic pursuit. Follow URLs for details. Dedicated to students of the subject. No
claim is made/ implied about truthfulness of this document.
Gujarat University Syllabus is in BOLD text. References to questions listed herein below,
are to such questions which were asked in Gujarat University examinations.
This PDF doc has navigation built-in. ie viewer shall be able to quickly move around the
PDF with mouse clicks. However, sometimes due to browser limitations, click-able links
may not work. In that case just download the PDF and open it in any standard PDF
viewer, eg Adobe Reader.
Lastly, | : | This doc also promotes the cause of law students. Do
suggest better answers and do point-out mistakes. Or alternately, feel free to update it
(for non-commercial purpose) as you deem fit. But, do keep sharing with the world.
Thank you.
Refer (not in any particular order) :
Bare acts are a good source, in any subject of law.
http://www.lawyersclubindia.com/articles/Interpretation-of-Statute-5430.asp
https://www.everycrsreport.com/reports/97-589.html
https://www.bcasonline.org/Referencer2015-16/Taxation/Income
%20Tax/interpretation_of_taxing_statutes.html
http://lawtimesjournal.in/interpretation-of-statutes-important-questions/
http://www.caaa.in/Image/Interpretation%20of%20Statutes.pdf by CA. Rajkumar S.
Adukia
https://www.scribd.com/doc/7118910/Interpretation-of-Statutes#
http://lawtimesjournal.in/interpretation-of-statutes-important-questions/
http://14.139.60.114:8080/jspui/bitstream/123456789/714/17/Principles%20of
%20Interpretation%20of%20Statutes.pdf

https://www.facebook.com/LLB.GujUni/ http://duralex.bhatt.net.in/
https://www.facebook.com/groups/LLB.GujUni/ Page - 2 of 119

CONTENTS
210 Interpretation Of Statutes & Principles Of Legislation

Module-1) General And Rules Of Statutory Interpretation :

Module-2) Aids To Interpretation :

Module-3) Presumptions In Statutory Interpretation :

Module-4) Interpretation With Reference To The Subject Matter & Purpose


And Principles Of Constitutional Interpretation :

Module-5) Practical Section ---> These are not part of the text of syllabus.

Module-6) Case-laws : -- These are not part of the text of syllabus. Neverthless,
questions on landmark cases are asked regularly .

OBJECTIVES OF THE COURSE :


Legislation is the major source of law of the modem era. Legislatures enact laws after
much deliberation. No doubt in this process they have to take into account the present
and future needs of the people. What. are the matters to be reckoned with by
legislature while enacting laws? With the emergence of legislation, interpretation of
statutes became a method by which judiciary explores the intention behind the statues.
Judicial interpretation involves construction of words, phrases and expressions. In their
attempt to make the old and existing statutes contextually relevant, courts used to
develop certain rules, doctrines and principles of interpretation. Judiciary plays a highly
creative role in this respect. What are the techniques adopted by courts in construing
statutes? How for are they successful in their strategy?

https://www.facebook.com/LLB.GujUni/ http://duralex.bhatt.net.in/
https://www.facebook.com/groups/LLB.GujUni/ Page - 3 of 119

Module-1) GENERAL AND RULES OF STATUTORY INTERPRETATION :

1.1) Meaning of the term Statute.


1.2) Commencement, operation and repeal of statutes
1.3) Purpose and importance of Interpretation of Statutes
1.4) Meaning : Interpretation and construction
1.5) Rules of Statutory Interpretation
1.5.1) Primary Rules
1.5.2) Literal Rule
1.5.3) Golden Rule
1.5.4) Mischief Rule (Rule in Heydon's Case)
1.5.5) Rule of Harmonious Construction (also see para-4.2.1)
1.5.6) Secondary Rules:
Noscitur a sociis
Ejusdem generis
Reddendo singula singulis

GO TO CONTENTS.

MODULE-1 QUESTIONS :

General introduction to the subject.


Explain the meaning of "Interpretation and explain the basic principles of
Interpretation. (Apr-2012, Mar-2015)
In the construction of statute, the intent of the legislature is of supreme
importance. Discuss. (Oct-2012, Apr2013, Mar-2014)
Write short note : The basic principle of interpretation. (Oct-2012)
Discuss : Purpose and importance of Interpretation of Statutes.
Explain the meaning of term delegated legislation and state its limitations and
difference between conditional and delegated legislation. (Apr-2016)
Write short note : Codifying and consolidating statutes. (Oct-2012, Apr2013)
What are the perpetual and temporary statutes ? Explain the consequences of the
repeal of temporary statute. (Apr-2012, Mar-2015, Apr-2016)
Distinguish between perpetual and temporary statutes and explain the effect of
expiry of temporary statutes. (Oct-2012, Apr2013, Mar-2014)

https://www.facebook.com/LLB.GujUni/ http://duralex.bhatt.net.in/
https://www.facebook.com/groups/LLB.GujUni/ Page - 4 of 119

Write short note : Repeal of the statute. (Oct-2012, Apr2013)


Interpretation must be effective and workable." Discuss with case laws. (Oct-2012,
Mar-2014, Mar-2015)
Discuss in detail : Rules of Statutory Interpretation.
Discuss : Literal Rule.
Explain in detail with illustration the golden rule as applied to interpretation of
statutes and discuss when the rule can be ignored. (Oct-2012, Apr2013, Mar-2014,
Apr-2016)
What is the Golden rule of Interpretation? Explain this rule with important and
historical decisions on this issue. (Apr-2012, Mar-2015)
Write short note : Heydon's Case (Mischief Rule). (Apr-2012, Oct-2012, Apr2013,
Mar-2014, Mar-2015, Apr-2016)
Explain : Subjective and objective interpretation. (Apr-2016)
Explain Purposive approach to interpretation of statute.
Explain in detail the rule of Harmonious Construction with case laws. (Apr-2012,
Oct-2012, Apr2013, Mar-2015, Apr-2016)
What is Harmonions or Consistent interpretation ? Explain the rule relating to it.
(Mar-2014)
Are above 2 questions different ? Compare syllabus para-1.5.5 and 4.2.1)
Write short note : "Statute must be read as a whole. (Oct-2012)
Discuss : Noscitur a sociis.
Write short note : Principle of Ejusdem Generis. (Apr-2012, Oct-2012, Mar-2014, Mar-
2015)
Discuss : Reddendo singula singulis.
What are the subsidiary rules for Interpretation ? Explain in detail the rules with
respect to mandatory and directory provisions of any statute. (Apr-2012, Mar-
2015)
Write short note : Mandatory and directory provision of the statute with case laws.
(Oct-2012, Apr2013, Mar-2014, Mar-2015)
Write short note : Same word same meaning. (Oct-2012)

GO TO CONTENTS.

MODULE-1 ANSWERS :

https://www.facebook.com/LLB.GujUni/ http://duralex.bhatt.net.in/
https://www.facebook.com/groups/LLB.GujUni/ Page - 5 of 119

General introduction to the subject interpretation of statutes.


Explain the meaning of "Interpretation and explain the basic principles of
Interpretation. (Apr-2012, Mar-2015)
In the construction of statute, the intent of the legislature is of supreme
importance. Discuss. (Oct-2012, Apr2013, Mar-2014)
Write short note : The basic principle of interpretation. (Oct-2012)
Discuss : Purpose and importance of Interpretation of Statutes.
ANS :
Refer :
http://www.caaa.in/Image/Interpretation%20of%20Statutes.pdf by CA. Rajkumar
S. Adukia
https://www.scribd.com/doc/7118910/Interpretation-of-Statutes#
http://www.lawyersclubindia.com/articles/Interpretation-of-Statute-5430.asp
https://www.lawteacher.net/lecture-notes/english-legal-system/statutory-
interpretation.php
Intro :
Lord Denning in Seaford Court Estates Ltd. Vs Asher, English Knowledge is not an
instrument of mathematical precision... It would certainly save the judges from the
trouble if the acts of parliament were drafted with divine precision and perfect
clarity. However, in the absence of it, a judge cannot simply fold hand and blame
the draftsman...
The object of interpretation of statutes is to determine the intention of the
legislature conveyed expressly or impliedly in the language used in a statute.
It is of general believed that the law is deemed to be what the Court interprets it to
be. The very concept of interpretation connotes the introduction of elements which
are necessarily extrinsic to the words in the statute.
Salmond : the essence of law lies in the spirit, not its letter, for the letter is
significant only as being the external manifestation of the intention that underlies
it.
It is not always possible to accurately convert the real intend of the legislation
into written words. Legislators can not conceive/ assume wide variety of
conditions while making of any particular statute.
There will often be equally good or equally unconvincing arguments for two
competing interpretation.
The legislature becomes functus officio after enacting the statues. ie courts
cannot go back to the legislature and ask for the exact meaning of the statute.

https://www.facebook.com/LLB.GujUni/ http://duralex.bhatt.net.in/
https://www.facebook.com/groups/LLB.GujUni/ Page - 6 of 119

Thus it is totally on the Judges to interpret such provisions so that both are
effective.
Meaning of the term Statute :
Statute generally is defined as the written will of the legislature solemnly
expressed according to the forms necessary to constitute it the law of the State.
Black's Law Dictionary : A Statute is a formal written enactment of a legislative
authority that governs a country, state, city, or county. Typically, statutes command
or prohibit something, or declare policy.
The word is often used to distinguish law made by legislative bodies from the
judicial decisions of the common law and the regulations issued by Government
agencies.
A statute is a will of legislature conveyed in the form of text.
Article 13 (3) (a) of the constitution : The Constitution of India does not use the
term Statute but it uses the term law. Law includes any ordinance, order, bye-
law, rule, regulation, notification, custom or usage having the force of law.
Therefore, a Statute is the will of the legislature and Indian Statute is an Act of the
Central or State Legislature.
Statutes include Acts passed by the Imperial or Provincial Legislature in Pre-
Independence days as well as Regulations.
Statutes generally refer to the laws and regulations of every sort, every provision
of law which permits or prohibit anything.
Classification of statutes :
A Statute may generally be classified with reference to its duration, nature of
operation, object and extent of application. Types of classifications of Statutes may
be elaborated as follows :
A. Classification with reference to basis of Duration :
(i) Perpetual statutes - It is perpetual when no time is fixed for its duration and
such a statute remains in force until its repeal which may be express or implied.
It is a Perpetual Statute when no time is fixed for its duration and such statute
remains in force until its repeal, which may be express or implied. It is perpetual
in the sense that it is not obligated by efflux of time or by non-user.
(ii) Temporary statutes - A Temporary statute is one where its duration is only
for a specified time and it expires on the expiry of the specified time unless it is
repealed earlier. The duration of temporary Statute may be extended by fresh
Statute or by exercise of power conferred under the original statute. The expired
statute may be revived by re-enacting it in similar terms or by enacting a statute
expressly saying that the expired Act is herewith revived.

https://www.facebook.com/LLB.GujUni/ http://duralex.bhatt.net.in/
https://www.facebook.com/groups/LLB.GujUni/ Page - 7 of 119

B. Classification with reference to Nature of Operation :


(i) Prospective statutes A statute which operates upon acts and transactions
which have not occurred when the statutes takes effect, that is which regulates
the future is a Prospective statute.
(ii) Retrospective statutes A statute which operates upon acts and transactions
which occurred prior to enactment are called Retrospective statutes.
(iii) Directory statutes A directory statute is generally affirmative in its terms,
recommends a certain act or omissions, but imposes no penalty on non-
observance of its provisions. When the statute is passed for the purposes of
enabling something which are not essential and may be disregarded without
invalidating the things to be done are called directory statutes.
(iv) Mandatory statutes A Mandatory statute is one which compels
performance of certain acts and directs that a certain thing must be done in a
certain manner or form.
A type of Mandatory Statute is the Imperative Statute which are often
negative or prohibitory in its terms and makes certain acts or omissions
absolutely necessary and subjects a contravention of its provision to a
penalty.
When the statute is passed for the purposes of enabling something to be done
and prescribes the formalities which are to attend its performance, those
prescribed formalities which are essential to the validity of the things which
are done are called imperative or absolute.
While, imperative Statutes must be strictly observed. Directory Statute may be
substantially complied with.
C. Classification with reference to Objective :
(i) Enabling statutes These statutes are which enlarges the common law where
it is too strict or narrow. It is a statute which makes it lawful to do something
which would not otherwise be lawful.
(ii) Disabling statutes These statutes restrict or cut down rights existing at
common law.
(iii) Permissive statute This type of statute allows certain acts to be done
without commanding that they be performed.
(iv) Prohibitory statute This type of statute which forbids the doing of certain
things.
(v) Codifying Statute It presents and orderly and authoritative statement of
the leading rules of law on a given subject, whether those rules are to be found
in statute law or common law.
(vi) Consolidating statute The purpose of consolidating statute is to present

https://www.facebook.com/LLB.GujUni/ http://duralex.bhatt.net.in/
https://www.facebook.com/groups/LLB.GujUni/ Page - 8 of 119

the whole body of statutory law on a subject in complete form repeating the
former statute.
(vii) Curative or validating Statute - It is passed to cure defects in the prior law
and too validate legal proceedings, instruments or acts of public and private
administrative powers which in the absence of such statute would be void for
want of conformity with existing legal requirements but which would have been
valid if the statute has so provided at the time of enacting.
(viii) Repealing Statute A statute which either expressly or by necessary
implication revokes or terminates another statute is a repealing statute.
(ix) Amending Statute It is a Statute which makes and addition to or operates
to change the original law so as to effect an improvement or more effectively
carry out the purpose for which the original law was passed.
What is interpretation of statutes ?
Interpretation means the art of finding out the true sense of an enactment by
giving the words of the enactment their natural and ordinary meaning.
The process of statute making and the process of interpretation of statutes are two
distinct activities. Statutes are made by legislatures/ executives, while
interpretation is done by the courts.
Interpretation of statutes is the process of ascertaining the true meaning of the
words used in a statute.
The Court is not expected to interpret arbitrarily and therefore there have been
certain principles which have evolved out of the continuous exercise by the Courts.
These are called principles of interpretation or rules of interpretation.
The object of interpretation of statutes is to determine the intention of the
legislature conveyed expressly or impliedly in the language used. As stated by
SALMOND, "by interpretation or construction is meant, the process by which the
courts seek to ascertain the meaning of the legislature through the medium of
authoritative forms in which it is expressed."
Interpretation thus is a familiar process of considerable significance. In relation to
statute law, interpretation is of importance because of the inherent nature of
legislation as a source of law.
India connection :
Interpretation is as old as language. Elaborate rules of interpretation were evolved
even at a very early stage of the Hindu civilization and culture. The importance of
avoiding literal interpretation was also stressed in various ancient text books
Merely following the texts of the law, decisions are not to be rendered, for, if such
decisions are wanting in equity, a gross failure of Dharma is caused.
Mimamsa Rules of Interpretation ("reflection" or "critical investigation") :

https://www.facebook.com/LLB.GujUni/ http://duralex.bhatt.net.in/
https://www.facebook.com/groups/LLB.GujUni/ Page - 9 of 119

Elaborate rules of interpretation were evolved even at a very early stage of


Hindu civilization and culture. However, Indian historical rules of interpretation of
statute have not got the due recognition in the present study of rules of
interpretation. Many few peoples are aware about the existence of such rules
even in India. But in actual, there are many modern rules whose foundation has
been laid down in ancient rules of interpretation. These ancient Indian rules are
popularly known as Mimamsa Rules of Interpretation.
Mimamsa rules were given by Jaimini, the author of Mimamsat Sutras, which
are primarily for Vedanta and have contributed a lot in formulation and
development of Hindu Law. Mimamsa rules which were originally meant for
srutis, were employed for the interpretation of Smritis also. (Law Commission of
India, 60th Report, Chapter 2, para 2.2).
The basic aim of these rules is to give interpretation of the Vedas, the earliest
scriptures of Hinduism, and to provide a philosophical justification for the
observance of Vedic rituals. A basic discussion on six important Mimamsa rules is
as follow :
1) Upakarma-Upasamhara : This rule is basically to ensure unity of thought in
the beginning as well as in the end and it further indicates that statute should
be read as a whole. Look into preamble and epilogue at the first;
2) Abhyasa : This rule is an indication of repetitive process, meaning thereby
that what has been repeatedly said is because of legislatures continuous
effort to support his aim;
3) Apurvata : It is to see by this rule whether there is some thing novel to be
achieved by the legislation;
4) Phala : This rule suggests that there is need to go through word to word of
the statute because each word has specific thing to add on or to indicate
upon;
5) Atharvada : This rule indicates help of external aids is useful to interpret
any statute. This rule is widely followed in the modern context;
6) Upapatti : This is the last but most important rule which in literal sense is
known as logical deduction. This rule has its importance in case of ambiguity
in the enactment.
Mimamsa rules of interpretation are Indias one of great achievements, but
regrettably few people in our country are aware about the great intellectual
achievements of ancestors and the intellectual treasury they have bequeathed
upon India. As rightly stated by Justice Katju that Maxwell and Craies usually get
quoted on issue of Interpretation but Indian indigenous system of interpretation
is not been quoted even by Indian Lawyers in Indian Courts.
Recently one of the Supreme Court judgments, Ispat Industries Ltd. v.

https://www.facebook.com/LLB.GujUni/ http://duralex.bhatt.net.in/
https://www.facebook.com/groups/LLB.GujUni/ Page - 10 of 119

Commissioner of Customs, has refereed these rules while deciding an appeal


under the Customs Tariff Act, 1975.
Purpose and importance of Interpretation of Statutes :
Many times the use of language in the legislation even does not carry the clear cut
meaning in dictionaries. It contains many alternative meanings applicable in
different contexts and for different purposes so that no clear field for the
application of a word becomes identified. In such a situation, importance of
interpretation comes into picture.
If words of a statute do not carry clear cut meaning, then one judge takes the
narrow view and the other the broad one, and as a result the law will connote
different things for different persons and soon there will be race for window
shopping for justice.
For proper and healthy application of law, it is important to have uniform expansion
of language or words used by the authorities/ lawmakers.
Interpretation of Statutes is required for two basic reasons viz. to ascertain :
Legislative Language - Legislative language may be complicated for a layman,
and hence may require interpretation; and
Legislative Intent - The intention of legislature or Legislative intent assimilates
two aspects:
the concept of meaning, i.e., what the word means; and
the concept of purpose and object or the reason or spirit pervading
through the statute.
Necessity of interpretation would arise only where the language of a statutory
provision is ambiguous, not clear or where two views are possible or where the
provision gives a different meaning defeating the object of the statute.
If the language is clear and unambiguous, no need of interpretation would arise.
In this regard, a Constitution Bench of five Judges of the Supreme Court in R.S.
Nayak v A.R. Antulay, AIR 1984 SC 684 has held :
... If the words of the Statute are clear and unambiguous, it is the plainest duty
of the Court to give effect to the natural meaning of the words used in the
provision. The question of construction arises only in the event of an ambiguity
or the plain meaning of the words used in the Statute would be self defeating.
(para 18)
Again Supreme Court in Grasim Industries Ltd. v Collector of Customs, Bombay,
(2002)4 SCC 297 has followed the same principle and observed :
Where the words are clear and there is no obscurity, and there is no ambiguity
and the intention of the legislature is clearly conveyed, there is no scope for
court to take upon itself the task of amending or altering the statutory

https://www.facebook.com/LLB.GujUni/ http://duralex.bhatt.net.in/
https://www.facebook.com/groups/LLB.GujUni/ Page - 11 of 119

provisions. (para 10)


We always need to keep in mind that articulating a law is not equal to the
execution of law. For the purpose of execution, proper understanding of law or
statute is utmost important and better understanding is only possible through
proper interpretation of the statute.
General Principles / Rules :
The art of correct interpretation only depend on the ability to read what is stated in
plain language, read between the lines, read through the provision, examining the
intent of the Legislature and call upon case laws and other aids to interpretation.
Use of aids : In the process of interpretation, several aids are used. They may be
statutory or non-statutory.
Statutory aids may be illustrated by the General Clauses Act, 1897 and by
specific definitions contained in individuals Acts whereas,
non-statutory aids is illustrated by common law rules of interpretation (including
certain presumptions relating to interpretation) and also by case-laws relating to
the interpretation of statutes.
Never ending process :
It is not within the human powers to foresee the manifold permutations and
combinations that may arise in the actual implementation of the act and also to
provide for each one of them in terms free from all ambiguities. Hence
interpretation of statutes becomes an ongoing exercise as newer facts and
conditions continue to arise.
"Statutory Interpretation in a Nutshell" : Famous 1938 article by Professor John
Willis :
'a court invokes whichever of the rules produces a result that satisfies its sense
of justice in the case before it. Although the literal rule is the one most
frequently referred to in express terms, the courts treat all three as valid and
refer to them as occasion demands, but, naturally enough, do not assign any
reason for choosing one rather than another.'
Thus, on some occasions the literal rule would be preferred to the mischief rule:
on others the reverse would be the case. It was impossible to predict with
certainty which approach would be adopted in a particular case.
Sir Rupert Cross, Statutory Interpretation (3rd ed, 1995), suggested that the
English approach involves not so much a choice between alternative rules as a
progressive analysis in which the judge first considers the ordinary meaning of
the words in the general context of the statute, a broad view being taken of
what constitutes the "context", and then moves to consider other possibilities
where the ordinary meaning leads to an absurd result. This unified "contextual"

https://www.facebook.com/LLB.GujUni/ http://duralex.bhatt.net.in/
https://www.facebook.com/groups/LLB.GujUni/ Page - 12 of 119

approach is supported by dicta in decisions of the House of Lords where general


principles of statutory interpretation have been discussed.
Reference is now frequently made by judges to the concept of "purposive"
statutory construction, ie one that will "promote the general legislative purpose
underlying the provisions" (per Lord Denning MR in Notham v London Borough of
Barnet [1978] 1 WLR 220). There will be a comparison of readings of the
provision in question based on the literal or grammatical meaning of words with
readings based on a purposive approach. In Pepper (Inspector of Taxes) v Hart
[1993] AC 593, Lord Browne-Wilkinson referred to "the purposive approach to
construction now adopted by the courts in order to give effect to the true
intentions of the legislature". Lord Griffiths stated:
"The days have long passed when the courts adopted a strict constructionist
view of interpretation which required them to adopt the literal meaning of the
language. The courts now adopt a purposive approach which seeks to give effect
to the true purpose of legislation and are prepared to look at much extraneous
material that bears upon the background against which the legislation was
enacted."
Principles of Interpretation or Rules of Interpretation :
<Here is brief summary of rules. Detailed discussion elsewhere in this doc.>
Primary Rules :
Literal Rule (aka Plain Meaning Rule) - It means that statutes are to be
interpreted using the ordinary meaning of the language of the statute unless a
statute explicitly defines some of its terms otherwise. In other words, the law
must be read, word for word, and it should not divert from its true meaning.
Golden rule - Adhere to the ordinary meaning of the words used unless that is at
variance with the intention of the legislature (to be collected from the statute
itself), or leads to any manifest absurdity or repugnance, in which case it allows
the language to be varied or modified so as to avoid such inconvenience.
Mischief rule - The Mischief Rule is used by judges in statutory interpretation in
order to discover legislature's intention, as determined by examining secondary
sources, such as committee reports, treatises, law review articles and
corresponding statutes.
Its main aim is to determine the "mischief and defect" that the statute in
question has set out to remedy, and what ruling would effectively implement
this remedy. Smith vs. Hughes [1960] 2 All E.R. 859
Rule of Harmonious Construction - when there are two provisions in a statute,
which are in conflict with each other, they should be interpreted such that effect
can be given to both and the construction which renders either of them
inoperative and useless should not be adopted except in the last resort. Bengal

https://www.facebook.com/LLB.GujUni/ http://duralex.bhatt.net.in/
https://www.facebook.com/groups/LLB.GujUni/ Page - 13 of 119

immunity Co. vs. State of Bihar (1955) 6 STC 446 (SC).


Secondary Rules (aka Rules of Language) :
Noscitur a sociis - When a word is ambiguous, its meaning may be determined
by reference to the rest of the statute.
Ejusdem Generis - When a list of two or more specific descriptors are followed
by more general descriptors, the otherwise wide meaning of the general
descriptors must be restricted to the same class, if any, of the specific words
that precede them e.g. vehicles in "cars,motor bikes,motor powered vehicles"
would be interpreted in a limited sense and therefore cannot be interpreted as
including air planes.
Reddendo Singula Singulis - When a list of words has a modifying phrase at the
end, the phrase refers only to the last word, e.g., firemen, policemen, and
doctors in a hospital. Here,"in a hospital" only applies to doctors and not to
firemen or policemen.
Interpretation of statutes and judicial activism :
It is a well established fact that the legislature is highest law making body and the
court is merely an interpreter of the law. But actually the fact is by interpreting the
law the court can make comprehensive changes in the actual implementation and
overall maneuver of the law. This can be easily be gathered by analyzing the
statutory interpretation made by Indian judiciary and its effect on India and its
citizens as a whole.
The living example of such effect is interpretation of Part III (Fundamental Rights)
of Indian Constitution and especially Article 21, wider and liberal interpretation of
this article by the Honble Supreme Court of India has granted many fundamental
beneficial rights to the citizens of the country and even ensured actual execution of
these rights by liberally interpreting the concept of locus standi with further
evolution of Public Interest Litigation through which any public spirited person can
file a petition on behalf of those who has no access to Court.
Such evolutions with an important art of interpretation have ensured principles of
rule of law and equal justice or justice at door step in the developing country like
India.
But here it is important to mention that the manner and expansion of interpretation
by judiciary has been criticized by many and termed as over judicial activism with
interference in the field of legislature.

GO TO MODULE-1 QUESTIONS.
GO TO CONTENTS.

https://www.facebook.com/LLB.GujUni/ http://duralex.bhatt.net.in/
https://www.facebook.com/groups/LLB.GujUni/ Page - 14 of 119

Explain the meaning of term delegated legislation and state its limitations and
difference between conditional and delegated legislation. (Apr-2016)
ANS :
Refer :
Page-39, 53 of https://www.scribd.com/doc/251211773/Administrative-Law
http://www.shareyouressays.com/111363/what-is-the-importance-of-conditional-
legislation
http://mayank-lawnotes.blogspot.in/2007/01/administrative-law.html
What is delegated legislation ?
Introduction :
According to the traditional theory, the function of the executive is to administer
the law enacted by the legislature, and in the ideal State, the legislative power
must be exercised exclusively by the legislators who are directly responsible to
the electorate.
But, in truth, apart from pure administrative functions, the executive performs
many legislative and judicial functions also.
It has, therefore, been rightly said that the delegated legislation is so
multitudinous that a statute book would not only be incomplete but misleading
unless it be read along with delegated legislation which amplifies and
supplements the law of the land.
Definitions :
It is very difficult to give any precise definition of the expression delegated
legislation.
It is equally difficult to state with certainty the scope of such delegated
legislation.
Mukherjea, J. rightly says:
Delegated legislation is an expression which covers a multitude of confusion.
It is an excuse for the legislators, a shield for the administrators and a
provocation to the constitutional jurists...
According to Salmond, legislation is either supreme or subordinate. Whereas the
former proceeds from sovereign or supreme power, the latter flow from any
authority other than the sovereign power, and is, therefore, dependent for its
existence and continuance on superior or supreme authority.
Delegated legislation thus is a legislation made by a body or person other than
the Sovereign in Parliament by virtue of powers conferred by such sovereign
under the statute.
A simple meaning of the expression delegated legislation may be given as:

https://www.facebook.com/LLB.GujUni/ http://duralex.bhatt.net.in/
https://www.facebook.com/groups/LLB.GujUni/ Page - 15 of 119

When the function of legislation is entrusted to organs other than the


legislature by the legislature itself, the legislation made by such organs is
called delegated legislation.
Growth of Delegated Legislature :
No doubt, it is the twentieth century which has witnessed rapid growth of
delegated legislation in almost all legal systems of the world. But that does not
mean that it is a new phenomenon or that there was no delegation of legislative
power by Legislature to Executive in the past.
Ever since statute came to be enacted by Parliament, there was delegation of
legislative function.
A statute of 1337 contained a clause which made it felony to export wool,
unless it was ordained by the King and his Council.
In fifteenth and sixteenth centuries, there was frequent use of Henry VIII
Clause.
The Statute of Sewers of 1531 empowered Commissioners to make, re-make,
repeal and amend laws, to pass decrees and to levy cess. Thus, the
Commissioners used to exercise legislative, administrative and judicial powers
at a time.
Mutiny Act, 1717 conferred on the Crown power to legislate for the Army
without the aid of Parliament.
In nineteenth century, delegated legislation became more common and
considerably increased due to social and economic reforms.
In the twentieth century, output of delegated legislation by executive is
several times more than the output of enactments by a competent legislature.
In course of time, through a series of decisions, the Supreme Court has
confirmed the principle that the legislature can delegate its legislative power
subject to its laying down legal principles and provide standards for the guidance
of the delegate to promulgate delegated legislation, otherwise the law will be
bad on account of excessive delegation.
Whatever may be the test to determine the constitutionality of delegated
legislation, the fact remains that due to the compulsions of modern
administration courts have allowed extensive delegation of legislative powers,
especially in the area of tax and welfare legislation.
Reasons for Growth of Delegated Legislation :
Many factors are responsible for the rapid growth of delegated legislation in
every modern democratic State.
The traditional theory of laissez faire has been given up by every State and the
old police State has now become a welfare State.

https://www.facebook.com/LLB.GujUni/ http://duralex.bhatt.net.in/
https://www.facebook.com/groups/LLB.GujUni/ Page - 16 of 119

Because of this radical change in the philosophy as to the role to be played by


the State, its functions have increased. Consequently, delegated legislation has
become essential and inevitable.
Some of the reasons for growth of delegated legislation are as follows :
1. Pressure upon Parliamentary Time
2. Technicality
3. Flexibility
4. Experiment
5. Emergency
6. Complexity of Modern Administration
1. Pressure upon Parliamentary Time : As a result of the expanding horizons of
State activity, the bulk of legislation is so great that it is not possible for the
legislature to devote sufficient time to discuss all the matters in detail.
Therefore, legislature formulates the general policy and empowers the executive
to fill in the details by issuing necessary rules, regulations, bye-laws, etc.
In the words of Sir Cecil Carr, delegated legislation is a growing child called
upon to relieve the parent of the strain of overwork and capable of attending
to minor matters, while the parent manages the main business.
2. Technicality : Sometimes, the subject-matter on which legislation is required
is so technical in nature that the legislator, being himself a common man, cannot
be expected to appreciate and legislate on the same, and the assistance of
experts may be required. Members of Parliament may be the best politicians but
they are not experts to deal with highly technical matters which are required to
handled by experts. Here the legislative power may be conferred on expert to
deal with the technical problems,
e.g. gas, atomic energy, drugs, electricity, etc.
3. Flexibility : At the time of passing any legislative enactment, it is impossible
to foresee all the contingencies, and some provision is required to be made for
these unforeseen situations demanding exigent action. A legislative amendment
is a slow and cumbersome process, but by the device of delegated legislation,
the executive can meet the situation expeditiously. For that purpose, in many
statutes, a removal of difficulty clause is found empowering the administration
overcome difficulties by exercising delegated power.
e.g. bank-rate, police regulation export and import, foreign exchange, etc.
4. Experiment : The practice of delegated legislation enables the executive to
experiment. This method permits rapid utilization of experience and
implementation of necessary changes in application of the provisions in the light
of such experience, e.g. in road traffic matters, an experiment may be conducted

https://www.facebook.com/LLB.GujUni/ http://duralex.bhatt.net.in/
https://www.facebook.com/groups/LLB.GujUni/ Page - 17 of 119

and in the light of its application necessary changes could be made. Delegated
legislation thus allows employment and application of past experience.
5. Emergency : In times of emergency, quick action is required to be taken. The
legislative process is not equipped to provide for urgent solution to meet the
situation. Delegated legislation is the only convenient remedy. Therefore, in
times of war and other national emergencies, such as aggression, break down of
law and order, strike, 'bandh', etc. the executive is vested with special and
extremely wide powers to deal with the situation. There was substantial growth
of delegated legislation during the two World Wars. Similarly, in situation of
epidemics, floods, inflation, economic depression, etc. immediate remedial
actions are necessary which may not be possible by lengthy legislative process
and delegated legislation is the only convenient remedy.
6. Complexity of Modern Administration : The complexity of modem
administration and the expansion of the functions of the State to the economic
and social sphere have rendered it necessary to resort to new forms of
legislation and to give wide powers to various authorities on suitable occasions.
By resorting to traditional legislative process, the entire object may be frustrated
by vested interests and the goal of control and regulation over private trade and
business may not be achieved at all.
Conclusion :
The practice of empowering the executive to make subordinate legislation within
the prescribed sphere has evolved out of practical necessity and pragmatic needs
of the modem welfare State.
There has, therefore, been rapid growth of delegated legislation in all countries
and it has become indispensable in the modem administrative era.
What is conditional legislation ?
The idea behind conditional legislation is that the legislature makes the law which
is full and complete in all respects, but it is not brought into operation immediately.
The enforcement of the law is made dependent upon the fulfillment of a condition,
and what is delegated to the outside agency is the authority to determine, by
exercising its own judgment, whether or not the condition has been fulfilled.
Thus in conditional legislation, the law is there but its taking effect is made to
depend upon determination of some fact or condition by an outside agency.
For example, it would be a case of conditional legislation where the legislature
empowers an authority-
(a) To extend the duration of an Act within the maximum period fixed by the
legislature;
(b) To extend the existing laws to any territory;

https://www.facebook.com/LLB.GujUni/ http://duralex.bhatt.net.in/
https://www.facebook.com/groups/LLB.GujUni/ Page - 18 of 119

(c) To determine the extent to which limits fixed by law should be applied;
(d) To determine the time of applying law;
(e) To bring an Ordinance into force if the State Government is satisfied about
existence of an emergency and declares it to be in force in the State. The
Ordinance provides for the establishment of the special courts the time and
within the limits considered necessary by the State Government.
In Sardar Inder Singh v. State of Rajasthan, AIR 1957 SC 510
it was laid down that when an appropriate Legislature enacts a law and
authorizes an outside authority to bring it into force in such area or at such time
as it may decide, that is conditional and not delegated legislation.
In Lachmi Narain V. India, the Supreme Court has itself stated that
no useful purpose is served by calling a power conferred by a statute as
conditional legislation instead of delegated legislation. There is no difference
between them in principle, for conditional legislation like delegated legislation
has a content, howsoever small and restricted, of the law-making power itself,
and in neither case can the person be entrusted with the power act beyond the
limits which circumscribe the power.
In the High Court of Australia case of Baxter v. Ah- Way, C.L.R. 626 at page 637 :
The question raised in this Australian case related to the validity of certain
provisions of the Customs Act of 1901.
The Act prohibited the importation of certain goods which were specifically
mentioned and then gave power to the Governor- General-in-Council to
include, by proclamation, other goods also within the prohibited list.
The validity of the provision was challenged on the ground of its being an
improper delegation of legislative powers. This contention was repelled and it
was held that,
this was not a case of delegation of legislative power but of conditional legis-
lation of the type which was held valid by the Privy Council in the case of Reg.
v. Burah, 3 A.C.,889: 51 A. 178.
The aim of all legislatures is to project their minds as far as possible into the
future, and to provide in terms as general as possible for all contingencies
likely to arise in the application of the law. But it is not possible to provide
specifically for all cases and, therefore legislation from the very earliest times
and particularly in modern times, has taken the form of conditional legislation,
leaving it to some specified authority to determine :
(i) The circumstances in which the law shall be applied, or
(ii) Whether its operation shall be extended to, or Legislation, or

https://www.facebook.com/LLB.GujUni/ http://duralex.bhatt.net.in/
https://www.facebook.com/groups/LLB.GujUni/ Page - 19 of 119

(iii) The particular class of persons or goods to which it shall be applied.


Conditional legislation can be broadly classified into three categories :-
1. In the first category when the Legislature has completed the task of enacting
a Statute, the entire superstructure of the legislation is ready but its future
applicability to a given area is left to the subjective satisfaction of the delegate
who being satisfied about the conditions indicating the ripe time for applying the
machinery of the said Act to a given area exercises that power as a delegate of
the parent legislative body.
When the Act itself is complete and is enacted to be uniformly applied in
future to all those who are to be covered by the sweep of the Act, the
Legislature can be said to have complied its task. All that it leaves to the
delegate is to apply the same uniformly to a given area indicated by the
parent Legislature itself but at an appropriate time.
This would be an act of pure and simple conditional legislation depending
upon the subjective satisfaction of the delegate as to when the said Act
enacted and completed by the parent Legislature is to be made effective.
2. Second category of conditional legislations : Here the delegate has to decide
whether and under what circumstances a completed Act of the parent legislation
which has already come into force is to be partially withdrawn from operation in
a given area or in given cases so as not to be applicable to a given class of
persons who are otherwise admittedly governed by the Act.
3. Third category of conditional legislation : Here depending upon satisfaction of
the delegate on objective facts placed by one class of persons seeking benefit of
such an exercise with a view to deprive the rival class of persons who otherwise
might have already got statutory benefits under the Act and who are likely to
lose the existing benefit because of exercise of such a power by the delegate.
In such type of cases the satisfaction of the delegate has necessarily to be
based on objective consideration of the relevant data for and against the
exercise of such power.
In such a third category of cases of conditional legislation the Legislature
fixes up objective conditions for the exercise of power by the delegate to be
applied to past or existing facts and for deciding whether the rights or
liabilities created by the Act are to be denied or extended to particular areas,
persons or groups.
Difference between delegated and conditional legislations :
In the case of conditional legislation, the legislation is complete in itself but its
operation is made to depend on fulfillment of certain conditions and what is
delegated to an outside authority, is the power to determine according to its own
judgment whether or not those conditions are fulfilled.

https://www.facebook.com/LLB.GujUni/ http://duralex.bhatt.net.in/
https://www.facebook.com/groups/LLB.GujUni/ Page - 20 of 119

In case of delegated legislation proper, some portion of the legislative power of


the Legislature is delegated to the outside authority. The Legislature, though
competent to perform both the essential and ancillary legislative functions,
performs only the former and parts with the latter, i.e., the ancillary function of
laying down details in favour of another for executing the policy of the statute
enacted.
Conditional legislation contains no element of delegation of legislative power and is,
therefore, not open to attack on the ground of excessive delegation,
Delegated legislation does confer some legislative power on some outside
authority and is therefore open to attack on the ground of excessive delegation.
In the conditional legislation the delegate's power is that of determining when a
legislative declared rule of conduct shall become effective :
Delegated legislation involves delegation of rule-making power which
constitutionally may be exercised by the administrative agent.
In the case of conditional legislation the legislature conditionally leavs to the
discretion of an external authority the time and manner of carrying its legislation
into effect as to also the determination of the area to which it is to extend.
By delegated legislation the delegate completes the legislation by supplying
details within the limits prescribed by the statute
the legislature having laid down the broad principles of its policy in the
legislation can then leave the details to be supplied by the administrative
authority.

GO TO MODULE-1 QUESTIONS.
GO TO CONTENTS.

Write short note : Codifying and consolidating statutes. (Oct-2012, Apr2013)


ANS :
Refer :
https://www.scribd.com/document/323887540/Codifying-and-Consolidating-
Statutes-docx
1. Codifying Statutes :
What is codifying statute ?
A Codifying Statute presents an orderly and an authoritative statement of the
leading rules of law on a given subject, whether those rules are to be found in a
statue law or common law.
The Codifying Statute exhaustively states the entire law on a particular subject.

https://www.facebook.com/LLB.GujUni/ http://duralex.bhatt.net.in/
https://www.facebook.com/groups/LLB.GujUni/ Page - 21 of 119

Such a statute subsumes in its code both the pre-existing statutory provision
and also the common law rules relating to the matter.
Indication that the Act is a Code will generally be found in the preamble, if any
or in the long title.
It has been said that a Codifying Act is presumed not to alter the law unless a
contrary intention appears.
Example, the Code of Civil Procedure, 1908.
Lord Herschell Rule :
A Codifying Act is approached in quite a different spirit from a Consolidating Act.
The principles applicable to the construction of a codifying statute are well stated
in an oft-quoted passage of Lord Herschel :
I think the proper course is, in the first instance, to examine the language of
Statute and to ask what is its natural meaning, uninfluenced by any
considerations derived from the previous state of the law, and not to start
with inquiring how the law previously stood, and then, assuming that it was
probably intended to leave it unaltered, to see if the words of the enactment
will bear an interpretation in conformity with this view.
Features of Codifying Statutes :
a) A Codifying Statute may be a Code only with respect to a particular branch of
a subject. It may not cover other branches of the same subject.
the Payment of Bonus Act, 1965 does not cover all categories of bonus and is
restricted to the subject of profit bonus. The result is that the Act speaks as a
complete code on the subject of profit bonus and does not annihilate by
implication other different and distinct kinds of bonus such as customary
bonus.
b) In contrast to an ordinary enactment, a Code is self contained and complete.
the essence of a Codifying Statute is to be exhaustive on matters in respect of
which it declares the law and it is not the province of the judge to disregard or
go outside the letter of enactment according to its true construction.
c) When the Code covers a situation, it is not permissible to apply general
principles. The court has to proceed on the mandate of the Code only.
when the Code was silent or ambiguous, resort to the principles of private law
may be necessary so that the courts may resolve difficulties by application of
common law or equitable principles. If the statute law covers the situation, it
will be an impermissible exercise of the judicial function to go beyond the
statutory provision by applying such principles merely because they may
appear to achieve a fairer solution to the problem being considered.
On this principle, it was held in King Emperor vs. Dahu Raut4, that a matter

https://www.facebook.com/LLB.GujUni/ http://duralex.bhatt.net.in/
https://www.facebook.com/groups/LLB.GujUni/ Page - 22 of 119

concerning admission and disposal of criminal appeals has to be dealt with in


terms of the Code of Criminal Procedure and not outside those provisions.
Similarly, in L. Janakirama Iyer vs. P.P.M. Nilkanto Iyer5, it was observed that
a question of res judicata in relation to a suit has to be decided solely on
terms of Section 11 of Civil Procedure Code and not on general principles of
res judicata.
Construction of Codifying Statute :
While construing a Codifying Act, first the language used in the Act should be
examined without any reference or influence of previous law.
2. Consolidating statutes :
What is a consolidating statute ?
Consolidating Statute is a Statute which presents whole body of statutory law on
the subject in complete form repealing the former Statute.
In other words, it is a Statute which consolidates various laws on a particular
subject at one place. It collects all statutory enactments on a specific subject
and gives them a shape of one Statute.
eg The Companies Act is an example of a Consolidating Statute.
According to Watson, the very object of consolidation is to collect the statutory
law on a particular subject and bring it down to date, in order that it may form a
useful Code applicable to the circumstances existing at that time when
Consolidating Act was passed.
Features of Consolidating Statutes :
a) A Consolidating Statute is not intended to alter the law and therefore it is
relevant to refer to the previous state of law or to judicial decisions interpreting
the repealed acts for the purposes of construction of corresponding provisions in
Consolidating Act.
b) A Consolidating Act maybe an Amending Act. This additional purpose is
usually indicated in the preamble of long title by use of words An Act to
consolidate and amend.
Construction of Consolidating Statutes :
The provisions in a Consolidating Act may have their origin in different
legislations. If there is any inconsistency between two such provisions,
respective dates of their first enactment may be referred to.
When a question arises as to construction of a section in a Consolidating Statute,
it may actually be a question of construction of an earlier act in which that
section first appeared.
However, the rule for construction of Consolidating Act is to examine the

https://www.facebook.com/LLB.GujUni/ http://duralex.bhatt.net.in/
https://www.facebook.com/groups/LLB.GujUni/ Page - 23 of 119

language used in the Act itself without any reference to repealed statutes. If
Consolidating Act fails to provide guidance as to its proper interpretation, THEN
the repealed enactments maybe looked to.
The presumption that same words employed in the same act at different places
bear the same meaning has no application to Consolidating Acts when it is
shown that different provisions where same words occur had their origin in
different legislations.
For arriving at correct interpretation of a section in a Consolidating Statute,
courts have consulted the earlier repealed acts in which that section had its
origin.
In Director of Public Prosecutions v. Schildkamp1, the question related to the
construction of Section 332(3) of Companies Act, 1948. This section, prior to
Consolidating Act coming into force, was enacted in Companies Act, 1928 as
Section 75 (3). Therefore Section 332 (3) was construed in the light of the
provisions of the original 1928 Act.
Difference between Codifying Statutes and Consolidating statutes :
Codifying Statute :
1. It presents an orderly statement of leading rules of law on a given subject.
2. Former statute survives.
3. It is self contained and complete.
4. Unless the Code is ambiguous or silent on an issue, the court cannot go
outside the letter of law.
5. Presumption that the same words when used at different places in same act
would bear same meaning holds good and applicable.
Consolidating Statutes :
1. It presents whole body of statutory law on the subject repealing former
statute.
2. Former statutes are repealed.
3. It is not self contained and complete.
4. Judges may refer to earlier state of law and the judicial decisions interpreting
repealed acts.
5. Presumption has no application.

GO TO MODULE-1 QUESTIONS.
GO TO CONTENTS.

https://www.facebook.com/LLB.GujUni/ http://duralex.bhatt.net.in/
https://www.facebook.com/groups/LLB.GujUni/ Page - 24 of 119

What are the perpetual and temporary statutes ? Explain the consequences of the
repeal of temporary statute. (Apr-2012, Mar-2015, Apr-2016)
Distinguish between perpetual and temporary statutes and explain the effect of
expiry of temporary statutes. (Oct-2012, Apr2013, Mar-2014)
Discuss : Commencement, operation and repeal of statutes.
Write short note : Repeal of the statute. (Oct-2012, Apr2013)
ANS :
Refer :
http://www.alameenlaw.in/modelpapers-dec2016.html
Commencement, operation and repeal of statutes :

Effect of repeal :
According to the General Clauses act, 1897, when this act or regulation made after
the commencement of this act repeals any enactments hitherto made or hereafter
to be made, then unless a different intention appears, the repeal shall not :-
a. revive anything not in force or existing at the time at which the repeal takes
effect
b. affect the previous operation of any enactment so repealed or anything duly
done or suffered there under
c. affect any rights, privilege, obligation or liability acquired or incurred under
any enactment so repealed
d. effect any penalty, forfeiture or punishment incurred in respect of any offence
committed against any enactment so repealed
e. affect any investigation, legal proceedings or remedy in respect of any such
right, privilege, obligation liability, penalty, forfeiture, or punishment as foresaid;
and such investigation, legal proceeding, or remedy may be instituted, continued
or punishment may be imposed as if the repealing Act or regulation had not
been passed.
Effect of expiry of temporary statutes :
When the duration of a statute is for a specified time it is temporary statute.
After a temporary statute expires it cannot be made effective by merely amending
the same.
Revival of the expired statute can be done only by re-enacting it on similar terms
or expressly saying that the expired act is herewith revived.
The effect of expiry of temporary statute can be discussed under the following sub
headings :

https://www.facebook.com/LLB.GujUni/ http://duralex.bhatt.net.in/
https://www.facebook.com/groups/LLB.GujUni/ Page - 25 of 119

1) Legal proceedings under expired statute : after the expiry of an act can the
legal proceedings under that act be initiated or continued?
If that act has a saving clause its effect would be similar to Sec 6 of the
General clause act which says repeal of the statute shall not affect the legal
proceeding.
But if such saving clause is not present than the proceedings started under
the temporary statute would terminateas soon as the statute expires. Thus in
the absence of a saving clause no person can be prosecuted/ convicted under
the expired act and the on-going proceedings will be terminated at once with
the expiry of the temporary act.
Case law : Rayala corporation vs director of enforcement.
2) Notifications, orders, rules etc made under temporary statute:-
with the expiry of temporary act, any notification, ordered or rules issued
under it will also come to an end and cannot be revived even if the provisions
of expired act is re-enacted.
3) Expiry does not make statute dead for all purpose :-
even without the saving clause the expiry does not make the statute dead for
all purpose.
The nature of rights and obligations under the act has to be considered in
determining whether such rights and obligations are continuing after the
expiry of the act or no.
case law : In State of Orissa vs Bhupendra Kumar, it was held that a person
who has been prosecuted and sentenced during the continuance of a
temporary act for violating its provisions cannot be released before he serves
out his sentence, even if the temporary act expires before the expiry of full
period of the sentence.
4) Repeal by temporary statute :- if a temporary statute repeals an existing
statute, the question which arises is if the repealing temporary statute expires
will the repealed statute revive?
Answer is : This will depend on the construction of the repealing statute.
Mode of Repeal : (i) Express Repeal and (ii) Repeal by implication
(i) Express repeal :
Express repeal of a statute is usually made by stating that the earlier statute or
a particular provision therein is thereby repealed.
Usually enactments repealed are mentioned in a schedule attached to the
repealing statute.
Such express repeal needs no construction of the later statute. The use of any

https://www.facebook.com/LLB.GujUni/ http://duralex.bhatt.net.in/
https://www.facebook.com/groups/LLB.GujUni/ Page - 26 of 119

particular form of words is not necessary to bring about an express repeal. All
that is necessary is that the words used show an intention to abrogate the Act or
provision in question.
The usual form is to
use the words 'is or are hereby repealed' and to mention the Acts sought to
be repealed in the repealing section or to catalogue them in a Schedule.
use of words 'shall cease to have effect', is also not uncommon. When the
object is to repeal only a portion of an Act words 'shall be omitted'
arenormally used.
The legislative practice in India shows that 'omission' of a provision is treated as
amendment which signifies deletion of that provision and is not different from
repeal.
It has been held that there is no real distinction between repeal and an
amendment.
It has also been held that where a provision of an Act is omitted by an Act and
the said Act simultaneously re-enacts a new provision which substantially
covers the field occupied by the repealed provision with certain modification,
in that event such renactment is regarded having force continuously and the
modification or changes are treated as amendment coming into force with
effect from the date of enforcement of re-enacted provision.
(ii) Repeal by implication :
Instead of express repeal, repeal may also occure by implicaion in new provision.
Eg substitution of a provision results in repeal of the earlier provision and its
replacement by the new provision. Substitution thus combines repeal and fresh
enactment.
The usual forms of repeal by implication are :
a) all provisions inconsistent with the Act are repealed, or
b) all Acts and parts of Acts in conflict with the provisions of this Act are
hereby repealed, or 'all laws and parts of laws in conflict herewith are
expressly repealed.
Provisions of this character leave the question open as to what laws are
inconsistent and are intended to be so repealed.
All rules of law which apply to implied repeals generally will be applicable to
repeals brought about in the aforesaid manner.
Scope of express or implied repeal : The question often arises as to the extent or
scope of the repeal.
Where a repealing clause expressly refers to a portion of the prior Act, the
remainder of such Act will not usually be repealed, as presumptions raised that no

https://www.facebook.com/LLB.GujUni/ http://duralex.bhatt.net.in/
https://www.facebook.com/groups/LLB.GujUni/ Page - 27 of 119

further repeal is necessary, unless there is irreconcilable inconsistency between


them.
In like manner, if the repealing clause is by its terms confined to a particular Act,
quoted by title it will not be extended to an Act upon a different subject.
And a general Act repealing all Acts inconsistent therewith, will usually apply to
general Acts and not to special or local laws.
The reason behind this rule finds its foundation in two premises : the special Act
is not repealed because it is not named, or because there is no absolute
inconsistency between the general Act and the special Act. Consequently, if the
repealing Act named the special Act or if the two were irreconcilably
inconsistent, the special Act would also be terminated.
Where there is direct reference in the repealing clause to a particular Act, it is a
case of express repeal.
But where there is no direct reference, the matter will have to be determined by
taking into account the exact meaning and scope of the words contained in the
repealing clause and the principles of law which govern the interpretation of the
same.
Principles governing the cases of implied repeal can in such a case be called in
aid to determine whether there is effective repeal of the earlier statute.
Prior statutes are held to be repealed by implication by subsequent statutes if the
prior enactment is special and the subsequent one is general. This rule must not be
pressed too far. If a special enactment and a subsequent general enactment are
absolutely repugnant and inconsistent with one another, the Courts have no
alternative but to declare the prior special enactment as repealed by the
subsequent general Act.
In all such cases the legislative intention, rather than grammar or letter of the
enactment, is the determining factor
Presumption against Repeal :
There is a presumption against a repeal by implication. The reason for the
presumption is that the legislature while enacting a law has a complete knowledge
of the existing laws on the subject-matter and therefore when it does not provide a
repealing provision, it gives out an intention not to repeal the existing legislation.
The burden to show that there has been a repeal by implication lies on the party
asserting it. Courts can lean against implied repeal. If by any fair interpretation
both the statutes can stand together, there will be no implied repeal. If possible,
implied repeal shall be avoided.
Where two enactments are entirely affirmative and identical no question of
inconsistency can arise. But if a later statute describes an offence created by an

https://www.facebook.com/LLB.GujUni/ http://duralex.bhatt.net.in/
https://www.facebook.com/groups/LLB.GujUni/ Page - 28 of 119

earlier one and provides for a different punishment, or varies procedure, THEN the
earlier statute is repealed by implication.
Similarly where a latter statute expresses in affirmative language its applicability to
whole of India, which would imply the repeal of an earlier statute or colonial law on
the same subject.
Hence a statute is repealed by implication in the following cases, namely
a) If its provisions are plainly repugnant to those of the subsequent statute..
b) If the two standing together would lead to wholly absurd consequences
c) if the entire subject-matter of the first is taken away by the second.
Consequences of Repeal :
Under the common law rule the consequences of repeal of a statute are very
drastic.
Except as to transactions past and closed, a statute after its repeal is as completely
obliterated as if it had never been enacted.
The effect is to destroy all inchoate rights and all causes of action that may have
arisen under the repealed statute. Therefore, leaving aside the cases where
proceedings were commenced, prosecuted and already brought to a finality before
the repeal, no proceeding under the repealed statute can be commenced or
continued after the repeal.
Another result of repeal under the common law rule is to revive the law in force at
the commencement of the repealed statute. Thus if one statute is repealed by a
second which in turn is repealed by a third, the effect is to revive the first statute
unless a contrary intention is indicated in the third statute.
The confusion resulting from all these consequences gave rise to the practice of
inserting saving clauses to prevent the obliteration of a statute in spite of its
repeal to keep intact rights acquired or accrued and liabilities incurred during its
operation and permit continuance or institution of any legal proceedings or
recourse to any remedy which may have been available before the repeal for
enforcement of such rights and liabilities. Thus, offences committed during the
continuance of a statute can now (due to saving clause) be prosecuted and
punished even after its repeal, a course which would not have been possible under
the common law rule of complete obliteration of a repealed statute.
The saving of rights and liabilities is in respect of those rights and liabilities which
were acquired or incurred under a repealed statute and not under the general law
which is modified by a statute.
In respect of rights and liabilities acquired or incurred under the general law
which is modified by a statute the inquiry should be as to how far the statute is
retrospective.

https://www.facebook.com/LLB.GujUni/ http://duralex.bhatt.net.in/
https://www.facebook.com/groups/LLB.GujUni/ Page - 29 of 119

The distinction between what is, and what is not a right preserved by the provisions
of section 6,General Clauses Act is often one of great fineness. What is unaffected
by the repeal of a statute is a right acquired or accrued under it and not a mere
"hope or expectation of', or liberty to apply for, acquiring a right.
A distinction is drawn between a legal proceeding for enforcing a right acquired
or accrued and a legal proceeding for acquisition of a right. The former is saved
whereas the latter is not.

GO TO MODULE-1 QUESTIONS.
GO TO CONTENTS.

Interpretation must be effective and workable." Discuss with case laws. (Oct-2012,
Mar-2014, Mar-2015)
ANS :
Refer :
http://www.caaa.in/Image/Interpretation%20of%20Statutes.pdf by CA. Rajkumar
S. Adukia
http://www.lawyersclubindia.com/articles/Interpretation-of-Statute-5430.asp
What is interpretation of statutes ?
Interpretation of something means ascertaining the meaning or significance of that
thing or ascertaining an explanation of something that is not immediately obvious.
Construction and Interpretation of a statute is an age-old process and as old as
language.
Interpretation of statute is the process of ascertaining the true meaning of the
words used in a statute.
When the language of the statute is clear, there is no need for the rules of
interpretation. But, in certain cases, more than one meaning may be derived from
the same word or sentence. It is therefore necessary to interpret the statute to find
out the real intention of the statute.
Interpretation of statutes has been an essential part of English law since Heydon's
Case in 1854 and although it can seem complex, the main rules used in interpretation
are easy to learn.
The concept of interpretation of a Statute cannot be static one. Interpretation of
statutes becomes an ongoing exercise as newer facts and conditions continue to arise.
The purpose of Interpretation of Statutes is to help the Judge to ascertain the
intention of the Legislature not to control that intention or to confine it within the
limits, which the Judge may deem reasonable or expedient.

https://www.facebook.com/LLB.GujUni/ http://duralex.bhatt.net.in/
https://www.facebook.com/groups/LLB.GujUni/ Page - 30 of 119

Some Important points to remember in the context of interpreting Statutes :


Statute must be read as a whole in Context (harmonious construction)
Statute should be Construed so as to make it Effective and Workable if
statutory provision is ambiguous and capable of various constructions, then that
construction must be adopted which will give meaning and effect to the other
provisions of the enactment rather than that which will give none.
The process of construction combines both the literal and purposive approaches.
The purposive construction rule highlights that you should shift from literal
construction when it leads to absurdity.
ut res magis valeat quam pereaf (It is better for a thing to have effect than
to be made void) :
The Courts while pronouncing upon the constitutionality must prefer an
interpretation which keeps the statute within the competence of the Legislature.
ie courts must lean against an interpretation which reduces a statute to a nullity.
Here, it is important to analyze that in Indian Context, there is hardly any example
where a statute have been declared void for sheer vagueness, although
theoretically it may be possible to reach such a conclusion in case of absolute
intractability of the language used or when the language is absolutely meaningless.
This principle has further been defined by the Court itself as if the choice is
between two interpretations, we should avoid an interpretation which would reduce
the legislation to futility and should rather accept the bolder interpretation based
on the view that Parliament would legislate only for the purpose of bringing about
an effective result.
Thus, statutes should be interpreted as effective as workable as is possible while
lining with following rules.

GO TO MODULE-1 QUESTIONS.
GO TO CONTENTS.

Discuss in detail : Rules of Statutory Interpretation, (i) Primary Rules, (ii) Literal
Rule.
Discuss : Literal Rule.
Explain in detail with illustration the golden rule as applied to interpretation of
statutes and discuss when the rule can be ignored. (Oct-2012, Apr2013, Mar-2014,
Apr-2016)
What is the Golden rule of Interpretation? Explain this rule with important and
historical decisions on this issue. (Apr-2012, Mar-2015)

https://www.facebook.com/LLB.GujUni/ http://duralex.bhatt.net.in/
https://www.facebook.com/groups/LLB.GujUni/ Page - 31 of 119

Write short note : Heydon's Case (Mischief Rule). (Apr-2012, Oct-2012, Apr2013,
Mar-2014, Mar-2015, Apr-2016)
Explain : Subjective and objective interpretation. (Apr-2016)
Explain : Purposive Interpretation/ Approach.
Explain in detail the rule of Harmonious Construction with case laws. (Apr-2012,
Oct-2012, Apr2013, Mar-2015, Apr-2016)
What is Harmonions or Consistent interpretation ? Explain the rule relating to it.
(Mar-2014)
Are above 2 questions different ? Compare syllabus para-1.5.5 and 4.2.1)
Discuss : Noscitur a sociis,
Discuss : Reddendo singula singulis.
Write short note : Principle of Ejusdem Generis. (Apr-2012, Oct-2012, Mar-2014,
Mar-2015)
ANS :
Refer :
https://www.scribd.com/doc/7118910/Interpretation-of-Statutes
https://en.wikipedia.org/wiki/Purposive_approach
http://www.legalservicesindia.com/article/article/harmonious-and-beneficial-
construction-1941-1.html
http://hanumant.com/IOS-Unit4-RulesOfInterpretation.html
https://www.scribd.com/doc/7118910/Interpretation-of-Statutes
Introduction :
The term interpretation means To give meaning to.
Governmental power has been divided into three wings namely the legislature, the
executive and the judiciary.
Interpretation of statues to render justice is the primary function of the judiciary. It
is the duty of the Court to interpret the Act and give meaning to each word of the
Statute.
The most common rule of interpretation is that every part of the statute must be
understood in a harmonious manner by reading and
construing every part of it together.
A Verbis legis non est recedendum :
The maxim A Verbis legis non est recedendum means that you must not vary
the words of the statute while interpreting it.
The object of interpretation of statutes is to determine the intention of the

https://www.facebook.com/LLB.GujUni/ http://duralex.bhatt.net.in/
https://www.facebook.com/groups/LLB.GujUni/ Page - 32 of 119

legislature conveyed expressly or impliedly in the language used.


In Santi swarup Sarkar v pradeep kumar sarkar, the Supreme Court held that,
if two interpretations are possible of the same statute, the one which validates
the statute must be preferred.
Need for interpretation :
Interpretation is necessary when case involves subtle or ambiguous aspects of a
statute. Generally, the words of a statute have a plain and straightforward
meaning. But in some cases, there may be ambiguity or vagueness in the words of
the statute that must be resolved by the judge.
It is not always possible to precisely transform the intention of the legislature into
written words.
Interpreting a statute to determine whether it applies to a given set of facts often
boils down to analyzing whether a single word or short phrase covers some
element of the factual situation before the judge.
The expansiveness of language necessarily means that there will often be equally
good or equally unconvincing arguments for two competing interpretations.
A judge is then forced to resort to documentation of legislative intent, which may
also be unhelpful, and then finally to his or her own judgment of what outcome is
ultimately fair and logical under the totality of the circumstances.
To find the meanings of statutes, judges use various tools and methods of statutory
interpretation, including traditional canons of statutory interpretation, legislative
history, and purpose.
In common law jurisdictions, the judiciary may apply rules of statutory
interpretation to legislation enacted by the legislature or to delegated legislation
such as administrative agency regulations.
Outline of classification of Rules of Statutory Interpretation :
(1) Primary Rules :
(1.1) Literal Rule (aka Plain Meaning Rule),
(1.2) Golden rule,
(1.3) Mischief rule,
Purposive approach/ interpretation : Subjective and Objective Interpretation,
(1.4) Rule of Harmonious Construction,
(2) Secondary Rules (aka Rules of Language) :
(2.1) Noscitur a sociis,
(2.2) Ejusdem Generis,
(2.3) Reddendo Singula Singulis,

https://www.facebook.com/LLB.GujUni/ http://duralex.bhatt.net.in/
https://www.facebook.com/groups/LLB.GujUni/ Page - 33 of 119

(1) Primary Rules :


(1.1) Literal Rule :
The language of statute must be construed in its grammatical and literal sense and
hence it is termed as litera legis or litera script. It is the duty of the court not to
modify the language of the Act and if such meaning is clear and unambiguous,
effect should be given to the provisions of a statute, irrespective of consequences.
The idea behind such a principle is that the legislature, being the supreme law
making body must know what it intends in the words of the statute, and one need
not probe into the intention of the legislature.. Literal interpretation has been
called the safest rule because the legislatures intention can be deduced only from
the language through which it has expressed itself.
The literal rule is what the law says instead of what the law means. This is the
oldest of the rules of construction and is still used today, primarily because judges
are not supposed to legislate. As there is always the danger that a particular
interpretation may be the equivalent of making law, some judges prefer to adhere
to the law's literal wording.
A statues often contains a "definitions" section, which explicitly defines the most
important terms used in that statute. However, some statutes omit a definitions
section entirely, or fail to define a particular term.
The literal rule, which is also known as the plain meaning rule, attempts to guide
courts faced with litigation that turns on the meaning of a term not defined by the
statute, or on that of a word found within a definition itself.
According to this rule, when a word does not contain any definition in a statute, it
must be given its plain, ordinary, and literal meaning.
If the word is clear, it must be applied, even though the intention of the legislature
may have been different or the result is harsh or undesirable.
Case law :
Crawford vs Spooner, 1846 : Privy council noted that the courts cannot aid the
legislature's defective phrasing of an Act, they cannot add or mend, and by
construction make up for deficiencies which are left there.
In M V Joshi vs M V Shimpi, AIR 1961, relating to Food and Adulteration Act, it
was contented that the act does not apply to butter made from curd. However,
SC held that the word butter in the said act is plain and clear and there is no
need to interpret it differently. Butter is butter whether made from milk or curd.
In J.P. Bansal v. State of Rajasthan 2003, SC observed that the intention of the
legislature is primarily to be gathered from the language used. ie attention
should be paid to what has been said as also to what has not been said.
As a consequence, a construction which requires for its support, addition,

https://www.facebook.com/LLB.GujUni/ http://duralex.bhatt.net.in/
https://www.facebook.com/groups/LLB.GujUni/ Page - 34 of 119

substitution, or removal of words or which results in rejection of words as


meaningless has to be avoided.
In Kannailala Sur vs Parammindhi Sadhu Khan 1957, J Gajendragadkar says
that if the words used in statute are capable of only one construction then it is
not open to the courts to adopt any other hypothetical construction on the
ground that such construction is more consistent with the alleged objective and
policy of the act..
Thus, when the language of a provision is plain and clear, court cannot enlarge the
scope of the provision by interpretive process. Further, a construction which
requires for its support addition of words or which results in rejection of words as
meaningless has to be avoided
Advantages of literal rule :
It prevents courts from taking sides in legislative or political issues.
Ordinary people and lawyers do not have extensive access to secondary sources
and thus depending on the ordinary meaning of the words is the safest route.
It encourages precision in drafting.
Disadvantages :
The rule rests on the erroneous assumption that words have a fixed meaning.
Words are imprecise, leading justices to impose their own prejudices to
determine the meaning of a statute.
Sometimes the use of the literal rule may defeat the intention of Parliament.
For instance, in the case of Whiteley vs Chappel (1868; LR 4 QB 147), the
court came to the reluctant conclusion that Whiteley could not be convicted of
impersonating "any person entitled to vote" at an election, because the
person he impersonated was dead.
Using a literal construction of the relevant statutory provision, the deceased
was not "a person entitled to vote." This, surely, could not have been the
intention of Parliament. However, the literal rule does not take into account
the consequences of a literal interpretation.
If Parliament does not like the literal interpretation, then it must amend the
legislation.
Literal rule obliges the courts to fall back on standard common law principles of
statutory interpretation. Legislation is drawn up with these principles in mind.
One judges literal interpretation might be very different from anothers. Casey
says: What may seem plain to one judge may seem perverse and unreal to
another.
It ignores the limitations of language.

https://www.facebook.com/LLB.GujUni/ http://duralex.bhatt.net.in/
https://www.facebook.com/groups/LLB.GujUni/ Page - 35 of 119

To place undue emphasis on the literal meaning of the words is to assume an


unattainable perfection in draftsmanship.
Judges have tended excessively to emphasis the literal meaning of statutory
provisions without giving due weight to their meaning in wider contexts.
Advantages :
Proponents of the plain meaning rule claim that it prevents courts from taking
sides in legislative or political issues.
They also point out that ordinary people and lawyers do not have extensive
access to secondary sources and thus depending on the ordinary meaning of the
words is the safest route.
It encourages precision in drafting.
Disadvantages :
Opponents of the plain meaning rule claim that the rule rests on the erroneous
assumption that words have a fixed meaning. Words are imprecise, leading
justices to impose their own prejudices to determine the meaning of a statute.
However, since little else is offered as an alternative discretion-confining theory,
plain meaning survives.
Sometimes the use of the literal rule may defeat the intention of Parliament.
For instance, in the case of Whiteley vs Chappel (1868; LR 4 QB 147), the
court came to the reluctant conclusion that Whiteley could not be convicted of
impersonating any person entitled to vote at an election, because the
person he impersonated was dead. Using a literal construction of the relevant
statutory provision, the deceased was not a person entitled to vote.
This, surely, could not have been the intention of Parliament. However, the
literal rule does not take into account the consequences of a literal
interpretation, only whether words have a clear meaning that makes sense
within that context. If Parliament does not like the literal interpretation, then
it must amend the legislation.
It obliges the courts to fall back on standard common law principles of statutory
interpretation. Legislation is drawn up with these principles in mind. However,
these principles may not be appropriate to constitutional interpretation, which by
its nature tends to lay down general principles. It is said that it seems wrong to
parcel the Constitution as if it were a Finance Act.
One judges literal interpretation might be very different from anothers.
It ignores the limitations of language.
To place undue emphasis on the literal meaning of the words is to assume an
unattainable perfection in draftsmanship.

https://www.facebook.com/LLB.GujUni/ http://duralex.bhatt.net.in/
https://www.facebook.com/groups/LLB.GujUni/ Page - 36 of 119

Judges have tended excessively to emphasise the literal meaning of statutory


provisions without giving due weight to their meaning in wider contexts.

GO TO MODULE-1 QUESTIONS.
GO TO CONTENTS.

(1.2) Golden Rule :


Adhere to the ordinary meaning of the words used unless that is at variance with
the intention of the legislature (to be collected from the statute itself), or leads to
any manifest absurdity or repugnance, in which case it allows the language to be
varied or modified so as to avoid such inconvenience.
A further corollary to this rule is Rule of Reasonable Construction, that in case
there are multiple constructions to effect the Golden rule the one which favors
the assessee should always be taken.
This rule of statutory interpretation allows a shift from the ordinary sense of a
word(s) if the overall content of the document demands it.
This rule is a modification of the literal rule. It states that if the literal rule produces
an absurdity, then the court should look for another meaning of the words to avoid
that absurd result.
The rule was evolved by Parke B in Becke v Smith, 1836 and in Grey v Pearson,
1857, who stated,
"The grammatical and ordinary sense of the words is to be adhered to unless
that would lead to some absurdity or some repugnance or inconsistency with the
rest of the instrument in which case the grammatical and ordinary sense of the
words may be modified so as to avoid the absurdity and inconsistency, but no
farther."
This rule may be used in two ways :
1. It is applied most frequently in a narrow sense where there is some ambiguity
or absurdity in the words themselves.
Case of Lee vs Knapp 1967 QB where the interpretation of the word "stop"
was involved. Under Road Traffic Act, 1960, a person causing an accident
"shall stop" after the accident.
Now, in this case, the driver stopped after causing the accident and then
drove off. It was held that the literal interpretation of the word stop is
absurd and that the requirement under the act was not fulfilled because the
driver did not stop for a reasonable time so that interested parties can
make inquiries from him about the accident.
2. To avoid a result that is obnoxious to principles of public policy, even where

https://www.facebook.com/LLB.GujUni/ http://duralex.bhatt.net.in/
https://www.facebook.com/groups/LLB.GujUni/ Page - 37 of 119

words have only one meaning.


Case of Bedford vs Bedford, 1935, is another interesting case that highlighted
the use of this rule. It concerned a case where a son murdered his mother
and committed suicide.
The courts were required to rule on who then inherited the estate, the
mother's family, or the son's descendants.
The mother had not made a will and under the Administration of Justice Act
1925 her estate would be inherited by her next of kin, i.e. her son.
There was no ambiguity in the words of the Act, but the court was not
prepared to let the son who had murdered his mother benefit from his
crime.
It was held that the literal rule should not apply and that the golden rule
should be used to prevent the repugnant situation of the son inheriting. The
court held that if the son inherits the estate that would amount to profiting
from a crime and that would be repugnant to the act.
Advantages :
This rule prevents absurd results in some cases containing situations that are
completely unimagined by the law makers.
It focuses on imparting justice instead of blindly enforcing the law.
Disadvantages :
The golden rule provides no clear means to test the existence or extent of an
absurdity. It seems to depend on the result of each individual case.
No test exists to determine what is an absurdity.
This rule tends to let the judiciary overpower the legislature by applying its own
standards of what is absurd and what it not.
When the Golden Rule can be ignored ?
Golden rule can be ignored when words of the statute does NOT produce
absurdity.

GO TO MODULE-1 QUESTIONS.
GO TO CONTENTS.

(1.3) Mischief Rule (Heydons Case) :


The Mischief Rule is used by judges in statutory interpretation in order to discover
legislature's intention, as determined by examining secondary sources, such as
committee reports, treatises, law review articles and corresponding statutes.

https://www.facebook.com/LLB.GujUni/ http://duralex.bhatt.net.in/
https://www.facebook.com/groups/LLB.GujUni/ Page - 38 of 119

It essentially asks the question : By creating an Act of Parliament what was the
"mischief" that, (i) the previous or existing law did not cover, and (ii) this act
covers.
This rule was developed by Lord Coke in Sir John Heydon's Case, 1584, where it
was stated that there were four points to be taken into consideration when
interpreting a statute :
What was the common law before the making of the act?
What was the "mischief or defect" for which the common law did not provide?
What remedy the parliament hath resolved and appointed to cure the disease of
the commonwealth?
What is the true reason of the remedy?
The application of this rule gives the judge more discretion than the literal
and the golden rule as it allows him to effectively decide on Parliament's intent.
Case law :
Smith v Hughes, 1960 : Under the Street Offences Act 1959, it was a crime for
prostitutes to "loiter or solicit in the street for the purposes of prostitution".
Now, the defendants were calling to men in the street from balconies and
tapping on windows. They claimed they were not guilty as they were not in
the "street."
The judge applied the mischief rule to come to the conclusion that they were
guilty as the intention of the Act was to cover the mischief of harassment from
prostitutes.
Mischief rule is of narrower application than the golden rule or the plain meaning
rule, in that it can only be used to interpret a statute and only when the
statute was passed to remedy a defect in the common law.
This rule has often been used to resolve ambiguities in cases in which the literal
rule cannot be applied. As seen In Smith v Hughes, the mischief approach gave a
more sensible outcome than that of the literal approach.
Advantages :
The Law Commission sees it as a far more satisfactory way of interpreting acts
as opposed to the Golden or Literal rules.
It usually avoids unjust or absurd results in sentencing.
Disadvantages :
It is seen to be out of date as it has been in use since the 16th century, when
common law was the primary source of law and parliamentary supremacy was
not established.
It gives too much power to the unelected judiciary which is argued to be

https://www.facebook.com/LLB.GujUni/ http://duralex.bhatt.net.in/
https://www.facebook.com/groups/LLB.GujUni/ Page - 39 of 119

undemocratic.
In the 16th century, the judiciary would often draft acts on behalf of the king
and were therefore well qualified in what mischief the act was meant to remedy,
however, such is not the case any more.
Advantages :
The Law Commission sees it as a far more satisfactory way of interpreting acts
as opposed to the Golden or Literal rules.
It usually avoids unjust or absurd results in sentencing
Disadvantages :
It is seen to be out of date as it has been in use since the 16th century, when
common law was the primary source of law and parliamentary supremacy was
not established.
It gives too much power to the unelected judiciary which is argued to be
undemocratic.
In the 16th century, the judiciary would often draft acts on behalf of the king
and were therefore well qualified in what mischief the act was meant to remedy,
however, such is not the case any more.

GO TO MODULE-1 QUESTIONS.
GO TO CONTENTS.

The purposive approach/ interpretation : Subjective and Objective


Interpretation :
This approach has emerged in more recent times. Here the court is not just
looking to see what the gap was in the old law, it is making a decision as to what
they felt Parliament meant to achieve.
Purposive interpretation is an approach to statutory and constitutional
interpretation under which common law courts interpret an enactment within the
context of the law's purpose.
Purposive interpretation is a derivation of mischief rule set in Heydon's Case, and
intended to replace the mischief rule, the plain meaning rule and the golden
rule. Purposive interpretation is used when the courts use extraneous materials
from the pre-enactment phase of legislation, including early drafts, hansards,
committee reports, and white papers. The purposive interpretation involves a
rejection of the exclusionary rule.
Israeli jurist Aharon Barak views purposive interpretation as a legal construction

https://www.facebook.com/LLB.GujUni/ http://duralex.bhatt.net.in/
https://www.facebook.com/groups/LLB.GujUni/ Page - 40 of 119

that combines elements of the subjective and objective interpretation.1


Barak states that the subjective elements include the intention of the author of
the text, whereas the objective elements include the intent of the reasonable
author and the legal systems fundamental values.
Critics of purposivism argue it fails to separate the powers between the legislator
and the judiciary, since it allows more freedom in interpretation by way of
extraneous materials in interpreting the law
Lord Denning in the Court of Appeal stated in Magor and St. Mellons Rural
District Council v Newport Corporation, 1950,
"we sit here to find out the intention of Parliament and of ministers and carry
it out, and we do this better by filling in the gaps and making sense of the
enactment by opening it up to destructive analysis".
This attitude was criticised on appeal by the House of Lords.
Lord Simmons called this approach "a naked usurpation of the legislative
function under the thin disguise of interpretation". He went on to say that if a
gap is disclosed, the remedy lies in an amending Act..
The purposive approach is one used by most continental European countries
when interpreting their own legislation. It is also the approach which is taken by
the European Court of Justice in interpreting EU law.
In young, specific, and rules-based statutes, subjective purpose would prevail,
while for old, general, and standards-based statutes, objective purpose would
weigh more heavily.

1 What is Subjective and what is Objective ?


Subjective is a statement that has been colored by the character of the speaker or writer. It
often has a basis in reality, but reflects the perspective through with the speaker views reality.
It cannot be verified using concrete facts and figures.
Objective is a statement that is completely unbiased. It is not touched by the speakers
previous experiences or tastes. It is verifiable by looking up facts or performing mathematical
calculations.
When to Be Objective and Subjective ?

Subjective : can be used when nothing tangible is at stake. When you are watching a movie
or reading a book for pleasure, being subjective and getting caught up in the world of the
characters makes your experience more enjoyable. If you are discussing any type of art,
you have to keep in mind that everyones opinions on a particular piece are subjective.
Objective : it is important to be objective when you are making any kind of a rational
decision. It might involve purchasing something or deciding which job offer to take. You
should also be objective when you are reading, especially news sources. Being objective
when you are meeting and having discussions with new people helps you to keep your
concentration focused on your goal, rather than on any emotions your meeting might
trigger.

https://www.facebook.com/LLB.GujUni/ http://duralex.bhatt.net.in/
https://www.facebook.com/groups/LLB.GujUni/ Page - 41 of 119

GO TO MODULE-1 QUESTIONS.
GO TO CONTENTS.

(1.4) Rule of Harmonious Construction : Statute must be read as a whole :


Intro :
Interpretation of statutes means the art of finding out the true sense of an
enactment by giving the words of the enactment their natural and ordinary
meaning.
It is the process of ascertaining the true meaning of the words used in a statute.
However, the Court is not expected to interpret arbitrarily and therefore there
have been certain principles which have evolved out of the continuous exercise
by the Courts. These principles are called rules of interpretation.
The object of interpretation of statutes is to determine the intention of the
legislature conveyed expressly or impliedly in the language used.
The principle of harmonious interpretation is similar to the idea of broad or
purposive approach. The key to this method of constitutional interpretation is
that provisions of the Constitution should be harmoniously interpreted.
As per Kelly : Constitutional provisions should not be construed in isolation from
all other parts of the Constitution, but should be construed as to harmonize with
those other parts.
As stated by SALMOND, "by interpretation or construction is meant, the process
by which the courts seek to ascertain the meaning of the legislature through the
medium of authoritative forms in which it is expressed."
Rule of Harmonious Construction :
The rule of harmonious construction is the thumb rule to interpretation of any
statute.
When there is a conflict between two or more statues or two or more parts of a
statute then the rule of harmonious construction needs to be adopted.
The rule follows a very simple premise that every statute has a purpose and
intent as per law and should be read as a whole. The interpretation consistent of
all the provisions of the statute should be adopted.
An interpretation which makes the enactment a consistent whole, should be the
aim of the Courts and a construction which avoids inconsistency or repugnancy
between the various sections or parts of the statute should be adopted.
The principle : Upon looking at various cases, the following important aspects
of this principle are evident

https://www.facebook.com/LLB.GujUni/ http://duralex.bhatt.net.in/
https://www.facebook.com/groups/LLB.GujUni/ Page - 42 of 119

The courts must avoid a head on clash of seemingly contradicting provisions


and they must construe the contradictory provisions so as to harmonize them.
The provision of one section cannot be used to defeat the provision contained
in another unless the court, despite all its effort, is unable to find a way to
reconcile their differences.
When it is impossible to completely reconcile the differences in contradictory
provisions, the courts must interpret them in such as way so that effect is
given to both the provisions as much as possible.
Courts must also keep in mind that interpretation that reduces one provision
to a useless number or a dead lumbar, is not harmonious construction.
To harmonize is not to destroy any statutory provision or to render it otiose.
The normal presumption should be consistency. It should *NOT* happen that
what is given with one hand (legislature) is taken away by the other (court).
The rule of harmonious construction has been tersely explained by the Supreme
Court thus,
When there are, in an enactment two provisions which cannot be
reconciled with each other, they should be so interpreted, that if possible,
effect should be given to both.
A construction which makes one portion of the enactment a dead letter
should be avoided, because harmonization is not equivalent to destruction.
Harmonious Construction should be applied to statutory rules and courts should
avoid absurd or unintended results.
It should be resorted to making the provision meaningful in the context.
It should be in consonance with the intention of Rule makers.
Note that, the Rule of Harmonious construction is even applicable to subordinate
legislature.
Case laws :
In the case of Raj Krishna vs Binod AIR 1954. In this case, two provisions of
Representation of People Act, 1951, which were in apparent conflict were
brought forth.
Section 33 (2) says that a Government Servant can nominate or second
a person in election,
but section 123(8) says that a Government Servant cannot assist any
candidate in election except by casting his vote.
The Supreme Court observed that both these provisions should be
harmoniously interpreted and held that
a Government Servant was entitled to nominate or second a candidate

https://www.facebook.com/LLB.GujUni/ http://duralex.bhatt.net.in/
https://www.facebook.com/groups/LLB.GujUni/ Page - 43 of 119

seeking election in State Legislative assembly. This harmony can only be


achieved if Section 123(8) is interpreted as giving the govt. servant the
right to vote as well as to nominate or second a candidate and forbidding
him to assist the candidate it any other manner.
Landmark case of CIT v Hindustan Bulk Carriers : Here the Supreme Court
laid down principles of rule of Harmonious Construction,
1. The courts must avoid a head on clash of seemingly contradicting
provisions and they must construe the contradictory provisions so as to
harmonize them.
2. The provision of one section cannot be used to defeat the provision
contained in another unless the court, despite all its effort, is unable to find
a way to reconcile their differences.
3. When it is impossible to completely reconcile the differences in
contradictory provisions, the courts must interpret them in such as way so
that effect is given to both the provisions as much as possible.
4. Courts must also keep in mind that interpretation that reduces one
provision to a useless number or dead is not harmonious construction.
Commissioner of Sales Tax, MP v Radha Krishna
Under section 46 (1) c of the Madhya Pradesh General Sales Tax Act, 1958,
criminal prosecution of the respondent partners was sanctioned in this case
by the Commissioner when even after repeated demands the assesse did
not pay the sales tax.
The respondent challenged this provision on the ground that there were
two separate provisions under the Act, namely, section 22 (4 A) and
section 46 (1) c under which two different procedures were prescribed to
realize the amount due but there was no provision of law which could tell
that which provision should be applied in which case.
According to the Supreme Court, the provision prescribed u/s 46 (1) c was
more drastic. It was held that by harmonious construction of these two
provisions, the conclusion drawn is that the Commissioner had a judicial
discretion to decide as to which procedure to be followed in which case.
Whenever the Commissioner will fail to act judicially, the court will have the
right to intervene.
However, in this case, the Commissioner had correctly decided that the
more drastic procedure under section 46 (1) c deserved to be followed
because of the failure of the assesse firm in paying sales tax despite the
repeated demands by the sales tax officer.
Sirsilk Ltd. v Govt. of Andhra Pradesh

https://www.facebook.com/LLB.GujUni/ http://duralex.bhatt.net.in/
https://www.facebook.com/groups/LLB.GujUni/ Page - 44 of 119

An interesting question relating to a conflict between two equally


mandatory provisions, viz., ss 17(1) and 18(1) of the Industrial Disputes
Act, 1947, is a good illustration of the importance of the principle that
every effort should be made to give effect to all the provisions of an act by
harmonizing any apparent conflict between two or more of its provisions.
Section 17(1) of the Act requires the government to publish every award of
a Labour Tribunal within thirty days of its receipt and by sub section (2)
of section 17 the award on its publication becomes final. Section 18(1) of
the Act provides that a settlement between employer and workmen shall be
binding on the parties to the agreement.
In a case where a settlement was arrived at after the receipt of the award
of a Labour Tribunal by the Government but before its publication, the
question was whether the Government was still required u/s 17(1) to
publish the award.
In construing these two equally mandatory provisions, the Supreme Court
held that the only way to resolve the conflict was to hold that by the
settlement, which becomes effective from the date of signing, the industrial
dispute comes to an end and the award becomes infructuous and the
Government cannot publish it.

GO TO MODULE-1 QUESTIONS.
GO TO CONTENTS.

(2) Secondary Rules (aka Rules of Language) :


(2.1) Noscitur a sociis :
Noscitur a Sociis (knowing from association) :
Noscere means to know and sociis means association. Thus, Noscitur a Sociis
means knowing from association.
Under the doctrine of "noscitur a sociis" the questionable meaning of a word or
doubtful words can be derived from its association with other words within the
context of the phrase.
This means that words in a list within a statute have meanings that are related to
each other. If multiple words having similar meaning are put together, they are to
be understood in their collective meaning.
According to Maxwell, "this rule means that when two or more words susceptible to
analogous meaning are clubbed together, they are understood to be used in their
cognate sense.
Rule of Noscitur a Sociis puts the words in context of the whole phrase.

https://www.facebook.com/LLB.GujUni/ http://duralex.bhatt.net.in/
https://www.facebook.com/groups/LLB.GujUni/ Page - 45 of 119

Rule of ejusdem generis puts the words just in relation to the nearby words. ie Rule
of Noscitur A Sociis is wider than the rule of Ejusdem Generis, which is only an
application of the former.
The language of the phrase can be used as a guide to arrive at the true meaning of
the word.
Note that,
Noscitur a sociis is only a rule of construction and it cannot be used when it is
clear that the word with wider meaning is deliberately used in order to increase
the scope.
It can only be used when the intention of the legislature in using a word with
wider sense along with the words with narrower meaning is not clear.
Further, this rule can only be used when the associated words have similar
meaning. It cannot be used when the words have disjoint meanings. For
example, in the case of Lokmat Newspapers vs Shankarprasad AIR 1999, it was
held that the words "discharge" and "dismissal" do not have the same analogous
meaning and so this rule cannot be applied.
Words that are coupled together take their colour from each other. The more
general is restricted to the less general.
Illustration :
cows, bulls, goats, sheep and other animals. Here, the expression other
animals cannot refer to lions and tigers; it refers only to domestic animals.
old, discarded, unserviceable or obsolete machinery stores or vehicles including
waste products. Here, the expression old was construed to refer to machinery
that had become non-functional or non-usable.
Case law :
In Foster v Diphwys Casson (1887) 18 QBD 428, involving a statute which stated
that explosives taken into a mine must be in a "case or canister".
Here the defendant used a cloth bag. The courts had to consider whether a
cloth bag was within the definition.
Under the rule of Noscitur a sociis, it was held that the bag could not have
been within the statutory definition, because parliament's intention was
referring to a case or container of the same strength as a canister.
In State of Assam vs R Muhammad AIR 1967, SC made use of this rule to arrive
at the meaning of the word "posting" used in Article 233 (1) of the Constitution.
It held that since the word "posting" occurs in association with the words
"appointment" and "promotion", it took its colour from them and so it means
"assignment of an appointee or a promotee to a position" and does not mean

https://www.facebook.com/LLB.GujUni/ http://duralex.bhatt.net.in/
https://www.facebook.com/groups/LLB.GujUni/ Page - 46 of 119

transfer of a person from one station to another.

GO TO MODULE-1 QUESTIONS.
GO TO CONTENTS.

(2.2) Ejusdem Generis :


Ejusdem generis means of the same kind. Every clause of a statute must be
construed with reference to other clauses of the Act.
The ejusdem generis, or of the same genus rule, is similar though narrower than
the more general rule of noscitur a sociis.
Generally particular words are given their natural meaning, provided the context
does not require otherwise.
If general words follow particular words pertaining to a class, category or genus
then it is construed that general words are limited to mean the person or thing of
the same general class, category or genus as those particularly exposed.
Eg: if the husband asks the wife to buy bread, milk and cake and if the wife buys
jam along with them, it is not invalidated merely because of not specifying it but
is valid because it is of the same kind.
The basic rule is that if the legislature intended general words to be used in
unrestricted sense, then it need not have used particular words at all. In such a
case the general words are construed as limited to things of the same kind as those
specified.
The rule operates where a broad or open-ended term appears following a
series of more restrictive terms in the text of a statute. Where the terms listed
are similar enough to constitute a class or genus, the courts will presume, in
interpreting the general words that follow, that they are intended to apply only to
things of the same genus as the particular items listed.
However, for this rule to apply, the preceding words must for a specific class or
genus.
Further, this rule cannot be applied in the words with a wider meaning appear
before the words with specific or narrow meaning.
Case law :
In Devendra Surti v State of Gujarat, under s2 (4) of the Bombay shops and
Establishments Act, 1948 the term commercial establishment means an
establishments which carries any trade, business or profession.
Here the word profession is associated to business or trade and hence a
private doctors clinic cannot be included in the above definitions as under the

https://www.facebook.com/LLB.GujUni/ http://duralex.bhatt.net.in/
https://www.facebook.com/groups/LLB.GujUni/ Page - 47 of 119

rule of Ejusdem Generis.


In UP State Electricity Board vs Harishankar, AIR 1979, SC held that the
following conditions must exist for the application of this rule -
1. The statue contains an enumeration of specific words
2. The subject of the enumeration constitute a class or a category
3. The class or category is not exhausted by the enumeration
4. A general term is present at the end of the enumeration
5. There is no indication of a different legislative intent
Justice Hidayatullah explained the principles of this rule through the following
examples
In the expression, "books, pamphlets, newspapers, and other documents",
private letters may not be held included if "other documents" be interpreted
ejusdem generis with what goes before.
But in a provision which reads, "newspapers or other documents likely to
convey secrets to the enemy", the words "other documents" would include
documents of any kind and would not take their meaning from newspaper.
In the case of Ishwar Singh Bagga vs State of Rajasthan 1987, where the words
"other person", in the expression "any police officer authorized in this behalf or
any other person authorized in this behalf by the State government" in Section
129 of Motor Vehicles Act, were held not to be interpreted ejusdem generis
because the mention of a single species of "police officers" does not constitute a
genus.
It can be seen that this rule is an exception to the rule of construction that general
words should be given their full and natural meaning. It is a canon of construction
like many other rules that are used to understand the intention of the legislature.
Rule of Ejusdem generis also covers The rank principle, which goes as follows -
Where a string of items of a certain rank or level is followed by general
residuary words, it is presumed that the residuary words are not intended to
include items of a higher rank than those specified.
By specifying only items of lower rank the impression is created that higher
ranks are not intended to be covered.
For example,
the phrase "tradesman, artificer, workman, labourer, or other person
whatsoever" was held not to include persons above the artisan class.
Similarly, the phrase "copper, brass, pewter, and tin, and all other metals" in a
local Act of 1825 was held not to include precious metals such as gold and
silver.

https://www.facebook.com/LLB.GujUni/ http://duralex.bhatt.net.in/
https://www.facebook.com/groups/LLB.GujUni/ Page - 48 of 119

Order banning arms, ammunition or gun powder or any other goods. Here,
in this order any other goods was construed to refer to goods similar to
arms, ammunition or gun powder.
Any person who entered into or works under a contract with an employer
whether the contract be any way of manual labour, clerical work or otherwise
. ---> Held that ejusdem generis is not applicable since manual labour and
clerical work do not belong to a single limited genus.

GO TO MODULE-1 QUESTIONS.
GO TO CONTENTS.

(2.3) Reddendo Singula Singulis :


The reddendo singula singulis principle concerns the use of words distributively.
Where a complex sentence has more than one subject, and more than one object,
it may be the right construction to render each to each, by reading the provision
distributively and applying each object to its appropriate subject.
A similar principle applies to verbs and their subjects, and to other parts of speech.
A typical application of this principle is where a testator A says 'I devise and
bequeath all my real and personal property to B'.
The term devise is appropriate only to real property. The term bequeath is
appropriate only to personal property. Accordingly, by the application of the
principle reddendo singula singulis, the testamentary disposition is read as if it
were worded 'I devise all my real property, and bequeath all my personal
property, to B'.
Case law :
In the case of Koteshwar Vittal Kamat vs K Rangappa Baliga, AIR 1969, in the
construction of the Proviso to Article 304 of the Constitution which reads,
"Provided that no bill or amendment for the purpose of clause (b), shall be
introduced or moved in the legislature of a state without the previous sanction of
the President".
It was held that the word introduced applies to bill and moved applies to
amendment.

GO TO MODULE-1 QUESTIONS.
GO TO CONTENTS.

What are the subsidiary rules for Interpretation ? Explain in detail the rules with

https://www.facebook.com/LLB.GujUni/ http://duralex.bhatt.net.in/
https://www.facebook.com/groups/LLB.GujUni/ Page - 49 of 119

respect to mandatory and directory provisions of any statute. (Apr-2012, Mar-


2015)
Write short note : Mandatory and directory provision of the statute with case laws.
(Oct-2012, Apr2013, Mar-2014, Mar-2015)
Write short note : Same word same meaning. (Oct-2012)
ANS :
Refer :
https://en.wikipedia.org/wiki/Last_antecedent_rule
https://www.bcasonline.org/Referencer2015-16/Taxation/Income
%20Tax/interpretation_of_taxing_statutes.html
https://www.scribd.com/doc/7118910/Interpretation-of-Statutes
https://www.scribd.com/doc/7118910/Interpretation-of-Statutes
http://www.legalservicesindia.com/article/article/interpretation-of-statutes-1152-
1.html
http://www.indialegalhelp.com/files/interpretation.pptx
Intro :
Subsidiary Rules (or Secondary Rules or Rules of Languge) of interpretation help
application of Primary Rules in interpretation of Statutes.
In Kanai Lal v. Paramnidh the court said,
it must always be borne in mind that the first and primary rule of construction
is that the intention of the Legislature must be found in the words used by the
legislature.
It also added When the material words are capable of two constructions, one of
which is likely to defeat or impair the policy of which is likely to defeat or impair
the policy of the Act whilst the other construction is likely to assist the
achievement of the said policy, then the courts would prefer to adopt the latter
construction.
Here follows that partial list of these Subsidiary Rules :
Mandatory and Directory Provisions
Same word, same meaning ; Different Word Different Meanings
Rule of Last Antecedent
Stare Decisis
Non Obstante Clause
Legal Fiction
Use of Or and And

https://www.facebook.com/LLB.GujUni/ http://duralex.bhatt.net.in/
https://www.facebook.com/groups/LLB.GujUni/ Page - 50 of 119

Construction of General Words


Noscitur a sociis (discussed elsewhere in this doc)
Ejusdem generis (discussed elsewhere in this doc)
Reddendo singula singulis (discussed elsewhere in this doc)
Casus Omissus
Expressio unius est excusio alterius (the express reference of one member of a
class may exclude other members of the class not so expressed, eg express
reference to coal mines may exclude reference to other types of mine)
Mandatory and Directory Provisions :
Summary :
A mandatory enactment must be obeyed or fulfilled exactly but it is sufficient if a
directory enactment be obeyed or fulfilled substantially.
When consequences of nullification or failure provided in the statute, the statute
is mandatory.
Use of negative words usually mandatory. Not less than three months notice
indicates a clear mandatory provision.
Affirmative words can be used to exclude all that is not covered.
The use of word shall generally indicates an imperative unless context shows
otherwise.
Use of word may indicates directory provision. Enabling provision.
must is a stronger version of shall.
Detailed discussion :
Whether an enactment is mandatory or directory depends on the scope and the
object of the statute.
Where the enactment demands the performance of certain provision without any
option or discretion it will be called peremptory or mandatory.
On the other hand if the acting authority is vested with discretion, choice or
judgment the enactment is directory.
In deciding whether the provision is directory or mandatory, one has to ascertain
whether the power is coupled with a duty of the person to whom it is given to
exercise it. If so, then it is imperative, mandatory.
Generally the intention of the legislature is expressed by mandatory and
directory verbs such as may, shall and must.
However, sometimes the legislature uses such words interchangeably. In such
cases, the interpreter of the law has to consider the intention of the legislature.

https://www.facebook.com/LLB.GujUni/ http://duralex.bhatt.net.in/
https://www.facebook.com/groups/LLB.GujUni/ Page - 51 of 119

If two interpretations are possible then the one which preserves the
constitutionality of the particular statutory provisions should be adopted and the
one which renders it unconstitutional and void should be rejected.
Non-compliance of mandatory provisions has penal consequences where as non-
compliance of directory provisions would not furnish any cause of action or
ground of challenge.
Maxwell says
that it is impossible to lay down any general rule for determining whether a
provision is mandatory or directory.
The supreme court of India is stressing time and again that
the question whether a statute is mandatory or directory, is not capable of
generalization and that in each case the court should try and get at the real
intention of the legislature by analyzing the entire provisions of the enactment
and the scheme underlying it.
In other words it depends on the intent of the legislature and not upon the
language in which the intent is clothed.
The intent of the legislature must be ascertained not only from the
phraseology of the provision, but also from its nature, design and
consequences which would follow from construing it in one form or another.
Same word, same meaning ; Different Word Different Meanings :
Same Word Same Meaning, Different Words Different Meanings fall within the
broad category of linguistic canons of construction and aid in the application and
interpretation of statutes.
Same Word Same Meaning, Different Words Different Meanings are important tools
in statutory interpretation which can resolve various problems while interpreting
statutes and thus ensure a just, fair and equitable result.
Presumption that words are used in the same sense throughout a statute.
When different words are used in the same statute, presumption that they are not
used in the same sense.
According to the rule of Same Word Same Meaning it is presumed that the same
word used in the same section or in different parts of the same statute bears the
same meaning and effect throughout the entire statute. The rule of Different Words
Different Meanings is a corollary to the rule of Same Word Same Meaning.
It must be noted that both the rules are flexible and are not hard and fast rules.
The presumptions thus are not very strong and can be rebutted by taking into
account the context in which the words are used, the object and scheme of the Act
etc

https://www.facebook.com/LLB.GujUni/ http://duralex.bhatt.net.in/
https://www.facebook.com/groups/LLB.GujUni/ Page - 52 of 119

Limitations : Weak presumption, readily displaced by the context.


Presumption is at its weakest when the word in question is of the kind that readily
draws its precise import, its range of meanings from its immediate setting or the
nature of the subject with regard to which it is employed.
Example at the end of the previous year and in the course of such previous
year appearing in section 23A of Income Tax Act, 1922 were interpreted
differently.
Rule of Last Antecedent :
Summary :
Relative and qualifying words, phrases and clauses are applied to the antecedent
immediately preceding.
Definition of premises in the Bombay Land Requisition Act, 1948 stated
any building or part of a building let or intended to be let separately.
It was held that the words, let or intended to be let separately did not
qualify the word building but only the words part of a building.
This subsidiary Rule is subordinate to the context.
Section 1(3)(a) of the Employees Provident Funds Act, 1952 reads,
subject to the provisions contained in section 16, it (the Act) applies to every
establishment which is a factory engaged in any industry specified in Schedule
I and in which fifty or more persons are employed.
Argued that 50 or more persons applied to industry and not to establishment.
Held otherwise.
Detailed discussion :
The last antecedent rule is a doctrine of interpretation of a statute, by which
"Referential and qualifying phrases, where no contrary intention appears, refer
solely to the last antecedent."
The rule is typically bound by "common sense" and is flexible enough to avoid
application that "would involve an absurdity, do violence to the plain intent of the
language, or if the context for other reason requires a deviation from the rule.".
A more formulaic approach to the rule requires, "Evidence that a qualifying
phrase is supposed to apply to all antecedents instead of only to the immediately
preceding one may be found in the fact that it is separated from the antecedents
by a comma."
Kenneth A. Adams, author of A Manual of Style for Contract Drafting, has
criticized this canon of construction as being applied inconsistently and contrary
to the guidance of many manuals of style :

https://www.facebook.com/LLB.GujUni/ http://duralex.bhatt.net.in/
https://www.facebook.com/groups/LLB.GujUni/ Page - 53 of 119

Manuals of style recognize that the comma is used to indicate a slight break in
a sentence. But according to the Rule of the Last Antecedent, adding a comma
after a series of antecedents not only doesn't sever the modifier from the last
noun or phrase in the series, it in fact operates remotely on all the
antecedents, binding them to the modifier. Nothing in the general literature on
punctuation suggests such a mechanism.
The last antecedent rule is also applied to contract interpretation.
Stare Decisis :
MEANS - to stand by decisions and not to disturb what is settled.
A principle of law which has become settled by a series of decisions is generally
binding on the courts and should be followed in similar cases.
This doctrine is the basis of common law.
The older the decision, the greater is its authority and the more truly it is
accepted as stating correct law.
A precedent by long recognition may mature into Stare Decisis.
This maxim has less relevance in constitutional cases.
Non Obstante Clause :
Summary :
Notwithstanding anything contained in this Act or in some particular provision in
the Act or in some particular Act or in any law for the time being in force
Later enactment prevails over the former.
Special enactment prevails over the general one.
Resolves conflicts between laws.
Detailed discussion :
The expression "non obstante means notwithstanding.
It is a legislative device to give such a clause an overriding effect over the law or
provision that qualifies such clause.
When a clause begins with notwithstanding anything contained in the Act or in
some particular provision/provisions in the Act, it is with a view to give the
enacting part of the section, in case of conflict, an overriding effect over the Act
or provision mentioned in the non obstante clause. It conveys that in spite of the
provisions or the Act mentioned in the non obstante clause, the enactment
following such expression shall have full operation.
The clause is used to override the mentioned law/ provision in specified
circumstances.
Parasuramaiah vs. Lakshamma AIR 1965 AP 220

https://www.facebook.com/LLB.GujUni/ http://duralex.bhatt.net.in/
https://www.facebook.com/groups/LLB.GujUni/ Page - 54 of 119

A non-obstante clause is usually used in a provision to indicate that the


provision should prevail despite anything to the contrary in the provision
mentioned in such non-obstante clause. In case there is any inconsistency or
a departure between the non-obstante clause and another provision, one of
the objects of such a clause is to indicate that it is the non-obstante clause
which would prevail over the other clause.
It is fairly common in different enactments to use the expression
"notwithstanding anything contained in this Act or other Acts" in order to make
such provision as superseding to the other provisions.
The Supreme Court in Maharashtra Tubes Ltd. vs. State Industrial and
Investment Corporation of Maharashtra Ltd. (1993) 78 Comp Case 803 has held
in no uncertain terms that
where two statutes contain similar non obstante clauses, it is the latter which
is to prevail over the former, for, the Legislature is supposed to be aware of
the fact that the statute already in force contains a non obstante clause but
still incorporates such non obstante clause in order to obliterate the effect of
the non obstante clause contained in the former statute.
Legal Fiction :
Summary :
when a statute enacts that something shall be deemed to have been done, which
in fact and in truth was not done, the court is entitled and bound to ascertain for
what purposes and between what persons the statutory fiction is to be resorted
to.
Full effect must be given to the statutory fiction and it should be carried to its
logical conclusion.
eg A legal fiction created in terms enacted for the purposes of this Act is
normally restricted to that Act and cannot be extended to cover another Act.
Detailed discussion :
Legal fiction is an important subsidiary rule of interpretation of Statute. It is
useful in deciding case where certain things are presumed to exist in fact of their
non-existences.
Legal fiction is defined as :-
1. A legal assumption that a thing is true which is either not true, or which is
probably false.
2. An assumption of law that something which is false is true.
3. A state of facts exists which has never really existed.
A legal fiction is a devise by which the law deliberately departs from the truth of

https://www.facebook.com/LLB.GujUni/ http://duralex.bhatt.net.in/
https://www.facebook.com/groups/LLB.GujUni/ Page - 55 of 119

things for some reason. E.g. A foreigner was treated to be a Roman citizen for
the purpose of jurisdiction.
Legal fiction is treated in the provisions of an enactment by using the term is
deemed.The deeming provision is for the purpose of assuming the existence of
fact does not really exist.
In New India Assurance Co. Ltd v Complete Insulation Pvt Ltd, the Supreme
Court held that
legal fiction created under Section 157 of the Motor Vehicles Act, 1988, the
transfer of 3rd party insurance is deemed to have effect from buyer to seller.
In Bengal Immunity Co Ltd v State of Bihar, The Supreme Court held that
the legal fiction should not be extended beyond its legitimate limits.
In Pandurang Vinayak v State of Bombay, the Supreme Court held that
for the purpose of legal fiction, the word ordinance is to be read as
enactment.
In Bombay corporation v CIT Bombay, Section 43 of the Income Tax Act provided
that under certain circumstances, an agent is for all the purpose of this Act,
deemed to be an agent of a non-resident person. Such agent is deemed to be an
assessee.
In Avatar Singh v State of Punjab, it was held that
rules framed in contravention of the Electricity Act, 1910 are separate and
hence theft of electricity is not an offence under the IPC.
Use of Or and And :
Summary :
Or is normally disjunctive and And is conjunctive, but at times they are read
as vice versa.
Owner and Master to be guilty Does a person have to be both owner and
master of a ship to be held guilty? Or are both person guilty?
Section 7 of the Official Secrets Act, 1920, reads :
Any person who attempts to commit any offence under the principal Act or
this Act, or solicits or incites or endeavours to persuade another person to
commit an offence, or aids or abets *and* does any act preparatory to the
commission of an offence.
Here, the word and printed in Italics was read as or, because by reading
and as and the result produced was unintelligible and absurd and against
the clear intention of the Legislature.
Detailed discussion :

https://www.facebook.com/LLB.GujUni/ http://duralex.bhatt.net.in/
https://www.facebook.com/groups/LLB.GujUni/ Page - 56 of 119

Prima Facie it may seem that interpretation of the words and and or need not
been considered essential, are be subsidiary and do not need much attention.
However, several times, it is just through the interpretation of the words and
and or that the whole meaning of the Statute has been changed and the
Judicature has evolved a new principle altogether which was never expected.
The aim of this article is to lay light on the importance and the need for correct
interpretation of the words and and or, as an aid to interpretation to ensure
that effect is given to the true intent of the Legislature.
The word or is normally disjunctive and and is normally conjunctive but at
times they are read as vice versa to effectuate the manifest intent of the
legislature as disclosed from the context. As stated by SCRUTTON L.J, You do
sometimes read or as and in a statute. But you do not do it unless you are
obliged because or does not generally mean and and and does not
generally mean or.
However, the rule is that or is normally disjunctive and and is normally
conjunctive and a departure from the same is not available unless the very aim
and purpose of the Statute so requires. The rationale being that if the
Legislature wishes to use and in a particular statutory provision, then it has
every right to do and nothing prevents them for doing so. So if the word and
has not been used and instead the word or has been used, it is obvious that
the Legislature has purposively used the word or. Unless, it is not proved, that
there was some reason or difficulty that prevented the Legislature from using
the and, literal interpretation has to be applied to the statutory provision and
the rule - or is normally disjunctive and and is normally conjunctive has to be
given effect to
The rule which permits deviation from the usual and ordinary interpretation of
the words and and or, is an extension of the Purposive Theory wherein
Courts have conferred upon themselves the power to extend the meaning of the
and and or and give them a meaning and interpretation, which though not
directly stated by the Legislature, yet, aims at achieving the real purpose of
Legislature.
Construction of General Words :
Examples of general words in any legal proceeding whatsoever, any connection
whatever, every place, in all its form.
Normal rule general words must receive a general construction unless there is
something in the Act itself such as the subject-matter with which the Act is dealing
or the context in which the said words are used to show the intention of the
Legislature that they must be given a restrictive meaning.
Words and particularly general words cannot be read in isolation; their colour and
content are derived from their context.

https://www.facebook.com/LLB.GujUni/ http://duralex.bhatt.net.in/
https://www.facebook.com/groups/LLB.GujUni/ Page - 57 of 119

General words and phrases, however wide and comprehensive they may be, in
their literal sense must usually be construed as being limited to the actual object of
the Act.
Casus Omissus :
The Casus Omissus rule provides that omissions in a statute cannot be supplied by
judicial construction.
This rule signifies that omissions in a statute cannot as a general rule be supplied
by interpretation. The Courts have the liberty only to remedy the logical defects in
words and phrases used in the statute and the intention of the legislature. The
court prefers the interpretation in accordance with the words used without adding a
new word.
In Parkinson v. Plunton while interpreting catering establishment in Wages Act,
1943 the House of Lords preferred the interpretation in accordance with the
language used therein and did not extend to cover the boarding and lodging.
The approach of the court is not to apply certain words which are not found in
the statute.
However, if the intention of the legislature is faulty, either too broad or too narrow,
the Courts are bound to accept them as they are given and they cannot either add,
alter, modify, deduct or amend from the given Statute, as such an action would
amount to legislation rather than construction or interpretation.
There is no scope for importing into the statute words which are not there. Such
importation would be, not to construe, but to amend the statute. Even if there be a
casus omissus, the defect can be remedied only by Legislation and not by judicial
interpretation.
The duty of the Court to try and harmonise the various provisions of an Act passed
by the legislature, but not to amend the words used by legislature. It is certainly
not the duty of the Court to stretch the words used by the legislature to fill the
gaps or omissions in the provisions of an Act, as given in Hiradevi v. District Board.
In the case of Commissioner of Income Tax, Central Calcutta v. National Taj Traders
The purpose of the Legislature has to be established from the exact words of the
Statute, where they arise in their accurate and precise form. But if the same is
implied in vague and ambiguous language, the Courts may seek the aid of every
reasonable and permissible aids to interpretation. This principle of Casus
Omissus cannot be supplied by the Court except in case of clear necessity and
when the reasons for it are found in the four corners of the Statute itself.
Expressio unius est excusio alterius :
The express mention of one person or thing is the exclusion of another.
Where the statutory language is plain and the meaning clear, there is no scope for

https://www.facebook.com/LLB.GujUni/ http://duralex.bhatt.net.in/
https://www.facebook.com/groups/LLB.GujUni/ Page - 58 of 119

applying the rule.


If a given word or phrase is competent of two interpretations, the express mention
of one of the possibilities on a similar context excludes the other possibility.
This rule may be used to denote the aim or intention of the Legislature, although it
would not be safe to regard it as an obligatory rule of law.
In the words of Lopes, L.J this maxim means
a valuable servant but a dangerous master.
Section 5 of the Transfer of Property Act,1882 defines transfer of property, which
means,
an act by which living persons conveys property, in present or future, to one or
more other living persons or to himself in and one or more other living persons
and to transfer property or to himself is to perform such act.
The next paragraph provides that in this section living person includes a
company or association or body of individuals whether incorporated or not .
This clearly provides that living person not only means an individual or human
being but can also refers to a company or association or body of individuals
whether incorporated or not.
However this rule may not always provide the answer to problems of construction.
It is often the result of inadvertence or accident that this principle is applied and
the maxim ought not to be applied when its application, having regard to the
subject matter to which it is to be applied, leads to inconsistency or injustice.
This maxim is also not used to extend the operation of a statute beyond the
provision that it actually makes, e.g. a law enacted by Parliament for A. While
there already is a law for A and others, the new law will not change the law for
others.

GO TO MODULE-1 QUESTIONS.
GO TO CONTENTS.

https://www.facebook.com/LLB.GujUni/ http://duralex.bhatt.net.in/
https://www.facebook.com/groups/LLB.GujUni/ Page - 59 of 119

Module-2) AIDS TO INTERPRETATION :

2.1) Internal aids


2.1.1) Title, Preamble, Headings and marginal notes
2.1.2) Sections and sub-sections, Punctuation marks
2.1.3) Illustrations, exceptions, provisos and saving clauses
2.1.4) Schedules, Non-obstante clause
2.2) External aids
2.2.1) Dictionaries
2.2.2) Translations
2.2.3) Travaux Preparations
2.2.4) Statutes in pari material
2.2.5) Contemporanea Exposito
2.2.6) Debates, inquiry commission report and Law Commission reports

GO TO CONTENTS.

MODULE-2 QUESTIONS :

What are the internal aids of interpretation of Statutes ? Explain any four in detail.
(Oct-2012, Apr2013, Mar-2015)
State the Internal Aids for Interpretation and discuss "Heading", "Punctuations and
"Definitions'. (Apr-2012, Apr-2016)
Explain briefly internal aid to Construction. Discuss Title, Preamble, Headings,
Marginal Notes, Punctuation and illustration in detail. (Mar-2014)
Write short notes : Marginal notes (Apr2013, Mar-2014)
Explain : Sections and sub-sections, exceptions, saving clauses, Schedules,
Non-obstante clause.
Write short note : Importance of illustration to section in the interpretation of
section. (Oct-2012, Apr2013)
Write short note : Proviso. (Oct-2012, Apr2013)
Explain in detail the external aids for the Interpretation. (Mar-2015, Apr-2016)
Discuss : Following external aids : (i) Translations, (ii) Travaux Preparations, (iii)
Statutes in pari material, (iv) Contemporanea Exposito, (v) Debates, inquiry
commission report and Law Commission reports.

https://www.facebook.com/LLB.GujUni/ http://duralex.bhatt.net.in/
https://www.facebook.com/groups/LLB.GujUni/ Page - 60 of 119

When can the External Aids be taken for Interpretation? Discuss when and how
"Later Political, Social, Economic Developments and Scientific Inventions",
"Dictionaries" and "Foreign Judgment", can be used for Interpretation ? (Apr-
2012)
When can the External Aids be taken for Interpretation ? Discuss when and how
"Parliamentary history", Dictionaries and "Foreign Judgments" be used for
interpretation ? (Oct-2012)
Discuss External aids of interpretation of statutes and explain "precedents', decision
of foreign courts and parliamentary histories. (Apr2013, Mar-2014)
Explain in detail : Importance of Foreign Judgments for Interpretation (Mar-2015)

GO TO CONTENTS.

MODULE-2 ANSWERS :

What are the internal aids of interpretation of Statutes ? Explain any four in detail.
(Oct-2012, Apr2013, Mar-2015)
State the Internal Aids for Interpretation and discuss "Heading", "Punctuations and
"Definitions'. (Apr-2012, Apr-2016)
Explain briefly internal aid to Construction. Discuss Title, Preamble, Headings,
Marginal Notes, Punctuation and illustration in detail. (Mar-2014)
Write short notes : Marginal notes (Apr2013, Mar-2014)
Explain : Sections and sub-sections, exceptions, saving clauses, Schedules,
Non-obstante clause.
Write short note : Importance of illustration to section in the interpretation of
section. (Oct-2012, Apr2013)
Write short note : Proviso. (Oct-2012, Apr2013)
ANS :
Refer :
http://www.caaa.in/Image/Interpretation%20of%20Statutes.pdf by CA. Rajkumar
S. Adukia
https://www.bcasonline.org/Referencer2015-16/Taxation/Income
%20Tax/interpretation_of_taxing_statutes.html
www.indialegalhelp.com/files/interpretation.pptx
Intro :
An Aid, on the other hand is a device that helps or assists. For the purpose of

https://www.facebook.com/LLB.GujUni/ http://duralex.bhatt.net.in/
https://www.facebook.com/groups/LLB.GujUni/ Page - 61 of 119

construction or interpretation, the court has to take recourse to various internal


and external aids.
Internal aids mean those materials which are available in the statute itself, though
they may not be part of enactment. These internal aids include, long title,
preamble, headings, marginal notes, illustrations, punctuation, proviso, schedule,
transitory provisions, etc.
When internal aids are not adequate, court has to take recourse to External aids.
External Aids may be parliamentary material, historical background, reports of a
committee or a commission, official statement, dictionary meanings, foreign
decisions, etc.
B. Prabhakar Rao and others v State of A.P. and others , AIR 1986 SC 120
O.Chennappa, Reddy J. has observed :
Where internal aids are not forthcoming, we can always have recourse to
external aids to discover the object of the legislation. External aids are not ruled
out. This is now a well settled principle of modern statutory construction.
Outline of internal-aids :
Title,
Preamble,
Headings
marginal notes
Definitional Sections/ Clauses
Sections and sub-sections,
Punctuation marks
Illustrations,
Exceptions and saving clause
provisos
Explanations
Schedules,
Non-obstante clause
Transitional provisions
Title :
Long title
The Long Title of a Statute is an internal part of the statute and is admissible as
an aid to its construction. In the past, the long title was not considered a part of
the statute and therefore, it was not put in the category of internal aids to

https://www.facebook.com/LLB.GujUni/ http://duralex.bhatt.net.in/
https://www.facebook.com/groups/LLB.GujUni/ Page - 62 of 119

construction.
Statute is headed by a long title and it gives the description about the object,
scope or purpose of an Act. It begins with the words- An Act to ............. For
e.g. The long title of the Criminal Procedure Code, 1973 is An Act to
consolidate and amend the law relating to criminal procedure.
However, its useful only to the extent of removing the ambiguity and confusions
and is not a conclusive aid to interpret the provision of the statute.
A long title of a Legislation may not control, circumscribe or widen the scope of
the legislation, if the provisions thereof are otherwise clear and unambiguous,
but if the terms of the legislation are capable of both a wider and a narrower
construction, that construction which would be in tune with the avowed object
manifested in the preamble or declared in the long title, ought to be accepted.
In Re Kerala Education bill, the Supreme Court held that
the policy and purpose may be deduced from the long title and the preamble.
In Manohar Lal v State of Punjab, Long title of the Act is relied as a guide to
decide the scope of the Act.
In Kedar Nath v. State of W Bengal, s 4 of the West Bengal Criminal Law
Amendment (Special Courts) Act, 1949, was interepted.
This section, under which, the start Government was empowered to choose as
to which particular case should go for reference to the Special Courtand be
tried under a special procedure, was challenged as violative of Art 14 of the
Constitution.
Rejecting the contention, the Supreme Court held that
the long title of the Act which said as Act to provide for the more speedy
trial and more effective punishment of certain offences was clear enough to
give the state government discretion as to which offences deserved tried by
the special courts under a special procedure.
Although the title is a part of the Act, it is in itself not an enacting provision and
though useful in case of ambiguity of the enacting provisions, is ineffective to
control their clear meaning.
Short Title -
The short title of an Act is for the purpose of reference & for its identification.
It ends with the year of passing of the Act. E.g. The Indian Penal Code, 1860;
The Indian Evidence Act, 1872. The Short Title is generally given at the
beginning with the words- This Act may be called...............
For e.g Section 1 of The Indian Evidence Act, 1872, says This Act may be
called, The Indian Evidence Act, 1872.

https://www.facebook.com/LLB.GujUni/ http://duralex.bhatt.net.in/
https://www.facebook.com/groups/LLB.GujUni/ Page - 63 of 119

Even though short title is the part of the statute, it does not have any role in the
interpretation of the provisions of an Act.
Preamble :
Expresses the scope, object and purpose of the Act more comprehensively than the
Long Title.
Preamble is the Act in a nutshell. It is a preparatory statement. It contains the
recitals showing the reason for enactment of the Act.
There may be no exact correspondence between preamble and enactment. The
enactment may go beyond, or it may fall short of, the indications gathered from the
Preamble.
It is one of the cardinal principles of construction that where the language of the
Act is clear, the preamble must be disregarded.
But where the object or meaning of the enactment is not clear, the preamble may
be resorted to explain it. Again, where very general language is used in an
enactment which, it is clear must be intended to have a limited application, the
preamble may be used to indicate to what particular instances, the enactment is
intended to apply.
We cannot, therefore, start with the preamble for construing the provisions of an
Act, though we could be justified in resorting to it, and we will be required to do so,
if we find that language used by the Parliament is ambiguous or is too general.
If the Preamble is clear one way and the enacting part is clear the other way, the
latter shall prevail.
Preamble of Constitution used for interpreting all laws in India
The preamble is an intrinsic aid in the interpretation of an ambiguous act.
If any doubts arise from the terms employed by the Legislature, it has always been
held a safe means of collecting the intention to call in aid the ground and cause of
making the statute and to have recourse to the preamble.
In Kashi Prasad v State, the court held that
even though the preamble cannot be used to defeat the enacting clauses of a
statute, it can be treated as a key for the interpretation of the statute.
Headings : marginal notes :
Headings are of two kinds
one prefixed to a section and
other prefixed to a group or set of sections.
Heading is to be regarded as giving the key to the interpretation and the heading
may be treated as preambles to the provisions following them.

https://www.facebook.com/LLB.GujUni/ http://duralex.bhatt.net.in/
https://www.facebook.com/groups/LLB.GujUni/ Page - 64 of 119

A heading is to be regarded as giving the key to the interpretation of clauses


ranged under it, unless the wording of section(s) is inconsistent with such
interpretation.
There are conflicting views on utility of headings. In general, It is well settled
that the headings prefixed to sections or entries (of a Tariff Schedule) cannot
control the plain words of the provision.
Useful to determine the sense of a doubtful expression; Indicates the general
drift of clause.
In Krishnaih V. State of (A.P. AIR 2005 AP 10) it was held that
headings prefixed to sections cannot control the plain words of the provisions.
Only in the case of ambiguity or doubt, heading or sub-heading may be
referred to as an aid in construing provision.
In Durga Thathera v Narain Thathera, the court held that
the headings are like a preamble which helps as a key to the mind of the
legislature but do not control the substantive section of the enactment.
Marginal notes are the notes which are inserted at the side of the sections in an Act
and express the effect of the sections stated.
Marginal notes appended to the Articles of the Constitution have been held to
constitute part of the constitution as passed by the constituent assembly and
therefore they have been made use of in construing the articles.
In Wilkes v Goodwin, the Court held that
the side notes are not part of the Act and hence marginal notes cannot be
referred.
It has been held by Courts that Headings & Marginal Notes do not decide the
construction of the section, but they are indicative of the meaning and purpose of
the section.
Definitional Sections/ Clauses :
The object of a definition is to avoid the necessity of frequent repetitions in
describing the subject matter to which the word or expression defined is intended
to apply.
A definition contained in the definition clause of a particular statute should be used
for the purpose of that Act. Definition from any other statute cannot be borrowed
and used ignoring the definition contained in the statute itself.
Definitions are used to extend the meaning of a word to include or cover something
which is not normally covered or included.
Definitions help in interpreting ambiguous words.
Definitions may be restrictive (means) or extensive (includes).

https://www.facebook.com/LLB.GujUni/ http://duralex.bhatt.net.in/
https://www.facebook.com/groups/LLB.GujUni/ Page - 65 of 119

All statutory definitions must be read subject to the qualifications variously


expressed in the provisions were they are used.
In the absence of a definition, the General Clauses Act should be referred. If not
defined there also, rules of interpretation shall come into play.
Sections and sub-sections :

Punctuation marks :
These are minor elements in the construction of statute and very little attention is
paid to them.
Only when a statute is carefully punctuated and there is no doubt about its
meaning can weight be given to punctuation.
It cannot, however, be regarded as a controlling element for determining the
meaning of a statute.
Have relevance only when the statute is properly punctuated.
If the section as punctuated leads to a conflict the punctuation must be ignored.
Example Section 27 of the Drugs and Cosmetics Act, 1940 provides that
whoever manufactures for sale, sells, stocks or exhibits for sale or distributes a
drug without license, is liable for punishment.
Does only stocking (as against stocking for sale) call for punishment?
Held that since there is no comma after stocks, it is not an offence to only stock
but is an offence to stock for sale.
Illustrations :
Illustrations in enactment provided by the legislature are valuable aids in the
understanding the real scope.
Illustrations form a part of the statute and though do not form a part of the
section, are relevant for interpretation.
Illustrations should not be readily rejected as repugnant to the section.
It shows the intention of framers of the Act and are useful for finding out the
intention of the legislature.
Example :
Is a diary maintained by a deceased father containing date of births, death and
marriages in his family a proof of age of his son under Indian Evidence Act?
Illustration to the relevant section provides that a letter from As deceased father
announcing the birth of A on a given day is a relevant fact. Held diary is relevant
fact since diary similar to letter.

https://www.facebook.com/LLB.GujUni/ http://duralex.bhatt.net.in/
https://www.facebook.com/groups/LLB.GujUni/ Page - 66 of 119

In Mahesh Chandra Sharma V.Raj Kumari Sharma, (AIR 1996 2 6SC 869), it was
held that
illustrations are parts of the Section and help to elucidate the principles of the
section.
Exceptions and saving clause :
Exception restrains the enacting clause to particular cases.
The exception operates to affirm the operation of the statute to all cases not
excepted and excludes all other exceptions.
Saving Clause are generally added when a statute is repealed and re-enacted.
Saving Clause is inserted to safeguard rights which would be lost by the general
provision.
A saving clause repugnant to the body of the Act is void.
It does not give a further right to the party.
Ex: Principle of Consideration has exceptions laid down under section 25 of the
Indian contract Act.
Provisos :
The normal function of a proviso is to except and deal with a case which would
otherwise fall within the general language of the main enactment, and its effect is
confined to that case.
It is added to the enactment to qualify or create an exception.
It does not state a general rule.
It must be construed with reference to the preceding parts of the clause to which it
is appended.
Differs from exception and saving clause . <----
It is subordinate to the main section.
It is used to remove special cases from the general enactment and provide for
them specifically.
There may be cases in which the language of the statute may be so clear that a
proviso may be construed as a substantive clause. But whether a proviso is
construed as restricting the main provision or as a substantive clause, it cannot be
divorced from the provision to which it stands as a proviso.
CIT vs. Ajax Products Ltd. (1964) 55 ITR 741 (SC)
It must be construed harmoniously with the main enactment.
Explanations :
An Explanation is added to a section to elaborate upon and explain the meaning of

https://www.facebook.com/LLB.GujUni/ http://duralex.bhatt.net.in/
https://www.facebook.com/groups/LLB.GujUni/ Page - 67 of 119

the words appearing in the section.


An Explanation to a statutory provision has to be read with the main provision to
which it is added as an Explanation.
An Explanation appended to a section or a sub-section becomes an integral part of
it and has no independent existence apart from it.
The purpose of an Explanation is not to limit the scope of the main section.
An Explanation is quite different in nature from a proviso; the latter excludes,
excepts and restricts while the former explains, clarifies or subtracts or includes
something by introducing a legal fiction.
It makes the meaning clear beyond dispute.
It helps in interpreting the true intent of the enactment.
It is not a substantive provision by itself.
Does an explanation is given at the end of the section apply to all the provisions of
the section ?
Held- it should be seen as to which clause it applies.
Explanation may be a negative one too.
Schedules :
Schedules form part of a statute.
They are at the end and contain minute details for working out the provisions of
the express enactment.
The expression in the schedule cannot override the provisions of the express
enactment.
Division of a statute into sections and schedules is only for convenience.
Schedule may contain substantive part of the Act.
Schedules often contains minute detail for working out the provisions of the
enactment.
Non-obstante clause :
Summary :
Notwithstanding anything contained in this Act or in some particular provision in
the Act or in some particular Act or in any law for the time being in force
Later enactment prevails over the former.
Special enactment prevails over the general one.
Resolves conflicts between laws.
Detailed discussion :

https://www.facebook.com/LLB.GujUni/ http://duralex.bhatt.net.in/
https://www.facebook.com/groups/LLB.GujUni/ Page - 68 of 119

The expression "non obstante means notwithstanding.


It is a legislative device to give such a clause an overriding effect over the law or
provision that qualifies such clause.
When a clause begins with notwithstanding anything contained in the Act or in
some particular provision/provisions in the Act, it is with a view to give the
enacting part of the section, in case of conflict, an overriding effect over the Act
or provision mentioned in the non obstante clause. It conveys that in spite of the
provisions or the Act mentioned in the non obstante clause, the enactment
following such expression shall have full operation.
The clause is used to override the mentioned law/ provision in specified
circumstances.
Parasuramaiah vs. Lakshamma AIR 1965 AP 220
A non-obstante clause is usually used in a provision to indicate that the
provision should prevail despite anything to the contrary in the provision
mentioned in such non-obstante clause. In case there is any inconsistency or
a departure between the non-obstante clause and another provision, one of
the objects of such a clause is to indicate that it is the non-obstante clause
which would prevail over the other clause.
It is fairly common in different enactments to use the expression
"notwithstanding anything contained in this Act or other Acts" in order to make
such provision as superseding to the other provisions.
The Supreme Court in Maharashtra Tubes Ltd. vs. State Industrial and
Investment Corporation of Maharashtra Ltd. (1993) 78 Comp Case 803 has held
in no uncertain terms that
where two statutes contain similar non obstante clauses, it is the latter which
is to prevail over the former, for, the Legislature is supposed to be aware of
the fact that the statute already in force contains a non obstante clause but
still incorporates such non obstante clause in order to obliterate the effect of
the non obstante clause contained in the former statute.
Transitional provisions :
Transitional provisions are temperorary provisions which are enacted for time
being. In order to fill the gap or lacuna which gets created when a provision gets
repealed unless new provision is inserted, transitional provisions are created by the
legislature.
Sometimes in the absence of a Law or an Act, Courts frame guidelines or rules
which act as transitional provisions.
Ex: guidelines laid down by the SC in Vishaka v/s State of Rajasthan is applied as a
law in matters dealing with sexual harassment of working women in work places.

https://www.facebook.com/LLB.GujUni/ http://duralex.bhatt.net.in/
https://www.facebook.com/groups/LLB.GujUni/ Page - 69 of 119

GO TO MODULE-2 QUESTIONS.
GO TO CONTENTS.

Explain in detail the external aids for the Interpretation. (Mar-2015, Apr-2016)
Discuss : Following external aids : (i) Translations, (ii) Travaux Preparations, (iii)
Statutes in pari material, (iv) Contemporanea Exposito, (v) Debates, inquiry
commission report and Law Commission reports.
When can the External Aids be taken for Interpretation? Discuss when and how
"Later Political, Social, Economic Developments and Scientific Inventions",
"Dictionaries" and "Foreign Judgement", can be used for Interpretation ? (Apr-
2012)
When can the External Aids be taken for Interpretation ? Discuss when and how
"Parliamentary history", Dictionaries and "Foreign Judgements" be used for
interpretation ? (Oct-2012)
Discuss External aids of interpretation of statutes and explain "precedents', decision
of foreign courts and parliamentary histories. (Apr2013, Mar-2014)
Explain in detail : Importance of Foreign Judgements for Interpretation (Mar-2015)
ANS :
Refer :
http://www.caaa.in/Image/Interpretation%20of%20Statutes.pdf by CA. Rajkumar
S. Adukia
https://www.bcasonline.org/Referencer2015-16/Taxation/Income
%20Tax/interpretation_of_taxing_statutes.html
www.indialegalhelp.com/files/interpretation.pptx
http://interpretationofstatutes.blogspot.in/2016/01/external-aids-to-construction-
of.html
Intro :
When internal aids are not adequate, court has to take recourse to external aids.
The external aids are very useful tools for the interpretation or construction of
statutory provisions.
As opposed to internal aids to construction there are certain aids which are external
to the statute. Such aids will include,
Dictionaries
Translations
Travaux Preparations : Parliamentary Debates : Parliamentary history

https://www.facebook.com/LLB.GujUni/ http://duralex.bhatt.net.in/
https://www.facebook.com/groups/LLB.GujUni/ Page - 70 of 119

Statutes in pari materia : Reference to Other Statutes


Inquiry commission reports
Law Commission reports
Social, Political and Economic Developments and Scientific Inventions
Precedents set by judgments of Indian Courts, Foreign Judgments
Other materials (text books and articles and papers published in the journals)
These external aids used in the interpretation of statutes are diuscussed below :
Dictionaries :
When a word is not defined in the statute itself, it is permissible to refer to
dictionaries to find out the general sense in which that word is understood in
common parlance.
However, in the selection of one out of the various meanings of a word, regard
must always be had to the scheme, context and legislative history.
Note that :
Dictionaries are not to be taken as authoritative exponents.
Definition of a term in one statute, is not a guide for construction of the same
term in another statute.
Titaghur Paper Mills Co. Ltd. vs. State of Orissa (1983) 142 ITR 663 (SC)
The dictionary meaning of a word cannot be looked at where the word has been
statutorily defined or judicially interpreted.
But where there is no such definition or interpretation, the court may take aid of
dictionaries to ascertain the meanings of a word in common parlance, bearing in
mind that a word is used in different senses according to its context and a
dictionary gives all the meanings of a word and the court has, therefore, the
context in which it has to interpret that word.
Punjab General Sales Tax Act, 1948 allows exclusion of turnover of a product on
which purchase tax has been paid from the turnover liable for payment of sales tax
of the same product.
Question arose - Will the turnover of paddy purchased to produce rice be
excluded from the taxable turnover of rice? In other words, are rice and paddy
same product?
Held paddy and rice are identical in terms of dictionary meaning but
commercially paddy and rice are two different products. Thus deduction not
available.
In this case, the court ruled against using the dictionary meaning.
Translations :

https://www.facebook.com/LLB.GujUni/ http://duralex.bhatt.net.in/
https://www.facebook.com/groups/LLB.GujUni/ Page - 71 of 119

Constitution recognizes various languages and official translations may be used as


guide to interpretation.
Contrasting views have been laid down as to validity of translated versions as an
aid to interpretation.
A translator might put his own interpretation while translating. Hence, translated
version is a doubtful tool for interpretation.
Travaux Preparations : Parliamentary Debates : Parliamentary history :
Travaux Preparations is a French word meaning "preparatory works". It means
official records of negotiation and includes statement of objects and reasons,
reports of committees, speeches and surrounding circumstances etc.
Parliamentary debates generally not considered relevant aid as it is ultimately the
statute which prevails.
Speeches made in the course of the debate on a Bill cannot reflect the mental
process lying behind the majority votes which carried the Bill. Therefore cannot
give a meaningful interpretation.
Travaux Preparations and historical setting cannot be used as an aid if the words
are plain and clear. However, if the wordings are ambiguous, then Travaux
Preparations and historical setting may be considered in order to arrive at the
proper construction.
These include parliamentary history, debates, historical facts, statement of objects
and reasons, report of expert committees.
Parliamentary history means the process by which an act is enacted. This includes
conception of an idea, drafting of the bill, the debates made, the amendments
proposed etc. Speech made by mover of the bill, amendments considered during
the progress of the bill are considered in parliamentary history whereas the papers
placed before the cabinet which took the decision for the introduction of the bill are
not relevant since these papers are not placed before the parliament.
The historical facts of the statute that is the external circumstances in which it was
enacted in should also be taken into note so that it can be understood that the
statute in question was intended to alter the law or leave it where it stood.
Statement of objective and reasons as to why the statute is being brought to
enactment can also be a very helpful fact in the research for historical facts, but
the same if done after extensive amendments in statute it may be unsafe to attach
these with the statute in the end.
It is better to use the report of a committee before presenting it in front of the
legislature as they guide us with a legislative intent and place their
recommendations which come in handy while enactment of the bill.
The Supreme Court in a numbers of cases referred to debates in the Constituent

https://www.facebook.com/LLB.GujUni/ http://duralex.bhatt.net.in/
https://www.facebook.com/groups/LLB.GujUni/ Page - 72 of 119

Assembly for interpretation of Constitutional provisions.


Recently, the Supreme Court in S.R. Chaudhuri v State of Punjab and others,
(2001) 7 SCC 126 has stated that
it is a settled position that debates in the Constituent Assembly may be relied
upon as an aid to interpret a Constitutional provision because it is the function of
the Court to find out the intention of the framers of the Constitution. (Para 33)
(K.S. Paripoornan v State of Kerala and others, AIR 1995 SC 1012) As far as
speeches in Parliament are concerned, a distinction is made between speeches of
the mover of the Bill and speeches of other Members. Regarding speeches made by
the Members of the Parliament at the time of consideration of a Bill, it has been
held that they are not admissible as extrinsic aids to the interpretation of the
statutory provision.
However, speeches made by the mover of the Bill or Minister may be referred to
for the purpose of finding out the object intended to be achieved by the Bill.
(Devadoss (dead) by L. Rs, v. Veera Makali Amman Koil Athalur, AIR 1998 SC 750.)
So far as Statement of Objects and Reasons, accompanying a legislative bill is
concerned, it is permissible to refer to it for understanding the background, the
antecedent state of affairs, the surrounding circumstances in relation to the statute
and the evil which the statute sought to remedy.
But, it cannot be used to ascertain the true meaning and effect of the
substantive provision of the statute.
Statutes in pari materia : Reference to Other Statutes :
When can Earlier / Later Acts be referred ?
the subject matter of the statute is same
if the later act is a mere consolidation of the previous acts.
legislature provides for reference to the earlier/ later act
If a general act follows a special act, the special act cannot be said to have been
repealed. (Company Secretaries Act, 1980 vs. Companies Act, 2013)
If two notifications are issued under two Central Acts, the one issued subsequently
shall prevail over the earlier one in case of any inconsistency.
In case where two Acts have to be read together, then each part of every act has to
be construed as if contained in one composite Act. However, if there is some clear
discrepancy then the latter Act would modify the earlier.
Where a single provision of one Act has to be read or added in another, then it has
to be read in the sense in which it was originally construed in the first Act. In this
way the whole of the first Act can be mentioned or referred in the second Act even
though only a provision of the first one was adopted.

https://www.facebook.com/LLB.GujUni/ http://duralex.bhatt.net.in/
https://www.facebook.com/groups/LLB.GujUni/ Page - 73 of 119

In case where an old Act has been repealed, it loses its operative force.
Nevertheless, such a repealed part may still be taken into account for construing
the unrepealed part.
For the purpose of interpretation or construction of a statutory provision, courts
can refer to or can take help of other statutes. It is also known as statutory aids.
For the purpose of interpretation or construction of a statutory provision, courts
can refer to or can take help of other statutes. It is also known as statutory aids.
The General Clauses Act, 1897 is an example of statutory aid.
The application of this rule of construction has the merit of avoiding any
contradiction between a series of statutes dealing with the same subject, it allows
the use of an earlier statute to throw light on the meaning of a phrase used in a
later statute in the same context.
On the same logic when words in an earlier statute have received an authoritative
exposition by a superior court, use of same words in similar context in a later
statute will give rise to a presumption that the legislature intends that the same
interpretation should be followed for construction of those words in the later
statute.
Reference to English Law :
Sometimes, English Law is also referred because they have the same system of
jurisprudence as ours.
Assistance in elucidating general principles and construing acts in pari materia.
Indian statutes should be interpreted with reference to the facts of Indian life.
Example The Company law originated in Great Britain and the Companies Act
of India is modelled on British law and experience.
When the provisions of the Indian law and the English Acts are alike the decision
of the English Courts throw good light and the reasons may be persuasive.
Inquiry commission reports :
<work on this>
Law Commission reports :
Reports of Commissions including Law Commission or Committees including
Parliamentary Committees preceding the introduction of a Bill can also be referred
to in the Court as evidence of historical facts or of surrounding circumstances or of
mischief or evil intended to be remedied.
Law Commissions Reports can also be referred to where a particular enactment or
amendment is the result of recommendations of Law Commission Report.
In Rosy and another v State of Kerala and others, (2000) 2 SCC 230
The Supreme Court considered Law Commission of India, 41st Report for

https://www.facebook.com/LLB.GujUni/ http://duralex.bhatt.net.in/
https://www.facebook.com/groups/LLB.GujUni/ Page - 74 of 119

interpretation of section 200 (2) of the Code of Criminal Procedure, 1898.


Social, Political and Economic Developments and Scientific Inventions :
A Statute must be interpreted to include circumstances or situations which were
unknown or did not exist at the time of enactment of the statute.
Any relevant changes in the social conditions and technology should be given due
weightage.
Courts should take into account all these developments while construing statutory
provisions.
In S.P. Gupta v Union of India, AIR 1982 SC 149, it was stated,
The interpretation of every statutory provision must keep pace with changing
concepts and values and it must, to the extent to which its language permits or
rather does not prohibit, suffer adjustments through judicial interpretation so as
to accord with the requirement of the fast changing society which is undergoing
rapid social and economic transformation ...
It is elementary that law does not operate in a vacuum. It is, therefore, intended
to serve a social purpose and it cannot be interpreted without taking into
account the social, economic and political setting in which it is intended to
operate.
It is here that the Judge is called upon to perform a creative function. He has to
inject flesh and blood in the dry skeleton provided by the legislature and by a
process of dynamic interpretation, invest it with a meaning which will harmonise
the law with the prevailing concepts and values and make it an effective
instrument for delivery of justice. (Para 62)
Therefore, court has to take into account social, political and economic
developments and scientific inventions which take place after enactment of a
statute for proper construction of its provision.
Precedents set by judgments of Indian Courts, Foreign Judgments :
When judicial pronouncements are been taken as reference it should be taken into
note that the decisions referred are Indian,
If they are foreign it should be ensured that such a foreign country follows the
same system of jurisprudence as ours and that these decisions have been taken in
the ground of the same law as ours.
These foreign decisions have persuasive value only and are not binding on Indian
courts and where guidance is available from binding Indian decisions; reference to
foreign decisions is of no use.
Other materials (text books and articles and papers published in the journals) :
In Ramlal v State of Rajasthan, (2001) 1 SCC 175

https://www.facebook.com/LLB.GujUni/ http://duralex.bhatt.net.in/
https://www.facebook.com/groups/LLB.GujUni/ Page - 75 of 119

Supreme Court used information available on internet for the purpose of


interpretation of statutory provision.
Courts also refer passages and materials from text books and articles and papers
published in the journals. These external aids are very useful tools not only for the
proper and correct interpretation or construction of statutory provision, but also for
understanding the object of the statute, the mischief sought to be remedied by it,
circumstances in which it was enacted and many other relevant matters.
In the absence of the admissibility of these external aids, sometimes court may not
be in a position to do justice in a case.

GO TO MODULE-2 QUESTIONS.
GO TO CONTENTS.

https://www.facebook.com/LLB.GujUni/ http://duralex.bhatt.net.in/
https://www.facebook.com/groups/LLB.GujUni/ Page - 76 of 119

Module-3) PRESUMPTIONS IN STATUTORY INTERPRETATION :

3.1) Statutes are valid


3.2) Statutes are territorial in operation
3.3) Presumption as to jurisdiction
3.4) Presumption against what is inconvenient or absurd
3.5) Presumption against intending injustice
3.6) Presumption against impairing obligations or permitting advantage from
ones own Wrong
3.7) Prospective operation of statutes

GO TO CONTENTS.

MODULE-3 QUESTIONS :

Discuss in detail : PRESUMPTIONS IN STATUTORY INTERPRETATION.


Discuss in detail : Statutes are valid
Discuss in detail : Statutes are territorial in operation
Discuss in detail : Presumption as to jurisdiction
Discuss in detail : Presumption against what is inconvenient or absurd
Discuss in detail : Presumption against intending injustice
Discuss in detail : Presumption against impairing obligations or permitting advantage
from ones own Wrong
Discuss in detail : Prospective operation of statutes

GO TO CONTENTS.

MODULE-3 ANSWERS :

Discuss in detail : PRESUMPTIONS IN STATUTORY INTERPRETATION.


ANS :
Refer :
http://www.caaa.in/Image/Interpretation%20of%20Statutes.pdf by CA. Rajkumar
S. Adukia

https://www.facebook.com/LLB.GujUni/ http://duralex.bhatt.net.in/
https://www.facebook.com/groups/LLB.GujUni/ Page - 77 of 119

In construing a doubtful or ambiguous statute, the Courts will presumethat it was the
intention of the legislature to enact a valid, sensible and just law, and one which
should change the prior law no further than maybe necessary to effectuate the
specific purpose of the act in question.
Unless the statute contains express words to the contrary it is assumed that the
following presumptions of statutory interpretation apply, each of which may be
rebutted by contrary evidence.
Presumptions represent the accepted judicial view of a range of circumstances that
have been predetermined to be the way in which every manifestation of those
circumstances will be viewed, until any evidence to the contrary is produced. These
tend to arise from theoretical and practical principles of the law.
Presumptions which are part of the syllabus : <detailed discussion on each given
separately>
Statutes are valid
Statutes are territorial in operation :
Presumption as to jurisdiction
Presumption against what is inconvenient or absurd
Presumption against intending injustice
Presumption against impairing obligations or permitting advantage from ones own
Wrong
Prospective operation of statutes :
Other presumptions :
A statute does not alter the existing common law. If a statute is capable of two
interpretations, one involving alteration of the common law and the other one not,
the latter interpretation is to be preferred.
If a statute deprives a person of his property, say by nationalization, he is to be
compensated for its value.
A statute is not intended to deprive a person of his liberty. If it does so, clear words
must be used. This is relevant in legislation covering, for example, mental health
and immigration.
A statute cannot impose criminal liability without proof of guilty intention. Many
modern statutes rebut this presumption by imposing strict liability; for e.g.
dangerous driving.

GO TO MODULE-3 QUESTIONS.
GO TO CONTENTS.

https://www.facebook.com/LLB.GujUni/ http://duralex.bhatt.net.in/
https://www.facebook.com/groups/LLB.GujUni/ Page - 78 of 119

Discuss in detail : Statutes are valid


ANS :
Refer :
https://www.scribd.com/doc/18178617/Statutory-Construction-Notes
In construing a doubtful or ambiguous statute, the Courts will presumethat it was the
intention of the legislature to enact a valid, sensible and just law, and one which
should change the prior law no further than maybe necessary to effectuate the
specific purpose of the act in question.
Laws are presumed constitutional. To justify nullification of law, theremust be a clear
and unequivocal breach of the constitution. The theory is that, as the joint act of the
legislative and executiveauthorities, a law is supposed to have been carefully studied
anddetermined to be constitutional before it was finally enacted.All laws are presumed
valid and constitutional until or unless otherwiseruled by the Court.
The two laws must be absolutely incompatible, and clear finding thereof must surface,
before the inference of implied repeal may be drawn.In the absence of an express
repeal, a subsequent law cannot beconstrued as repealing a prior law unless an
irreconcilable inconsistencyand repugnancy exists in terms of the new and old laws.
<work on this>

GO TO MODULE-3 QUESTIONS.
GO TO CONTENTS.

Discuss in detail : Statutes are territorial in operation


ANS :
Refer :
http://www.desikanoon.co.in/2014/06/constitutional-law-doctrine-of-territorial-
nexus.html
https://www.scribd.com/doc/18178617/Statutory-Construction-Notes
In construing a doubtful or ambiguous statute, the Courts will presumethat it was
the intention of the legislature to enact a valid, sensible and just law, and one
which should change the prior law no further than maybe necessary to effectuate
the specific purpose of the act in question.
A statute generally has effect only in the country enacted. However a statute does
not run counter to international law and should be interpreted so as to give effect
to international obligations.
<work on this>

https://www.facebook.com/LLB.GujUni/ http://duralex.bhatt.net.in/
https://www.facebook.com/groups/LLB.GujUni/ Page - 79 of 119

GO TO MODULE-3 QUESTIONS.
GO TO CONTENTS.

Discuss in detail : Presumption as to jurisdiction


ANS :
Refer :
https://www.scribd.com/doc/7118910/Interpretation-of-Statutes
https://www.scribd.com/doc/18178617/Statutory-Construction-Notes
Introduction :
In construing a doubtful or ambiguous statute, the Courts will presumethat it was
the intention of the legislature to enact a valid, sensible and just law, and one
which should change the prior law no further than maybe necessary to effectuate
the specific purpose of the act in question.
The general presumption is that ordinary courts of law namely the civil courts,
criminal courts, high courts and Supreme Court have jurisdiction over people.
Any statute which takes away the jurisdiction of ordinary courts must be rarely
resorted to, as people have the right to have free access to all the courts.
Jurisdiction of civil courts :
The basic presumption of law is that all civil courts are empowered to decide all
suits of civil nature. The basis of this presumption is that civil and criminal court
have general jurisdiction over people and they have right to have free access to
both civil and criminal court.
Section 9 of CPC
It was emphasized by the Supreme Court that the rule prescribed by section 9 of
CPC is that the court shall, subject to provisions contained in the code, have
jurisdiction to try al suits of civil nature excepting suits in which their cognizance
is either expressly or impliedly barred.
The law further presumes that a remedy in the ordinary civil courts must always be
available to citizens.
Legal provisions excluding jurisdiction of civil courts and conferring jurisdiction to
tribunals must be strictly interpreted in such a way that as far as possible, the
jurisdiction of civil court are not taken away.
If the statute contains two interpretations, then the one conferring jurisdiction
will prevail.
Exclusion of jurisdiction must be expressed or clearly implied.

https://www.facebook.com/LLB.GujUni/ http://duralex.bhatt.net.in/
https://www.facebook.com/groups/LLB.GujUni/ Page - 80 of 119

It is not possible to curtail jurisdiction of High Court and Supreme Court except by an
amendment to the relevant provisions in the constitution.
Jurisdiction of other courts :
The general presumption is that a statute should not be given such an
interpretation as to take away the jurisdiction of the court unless the language of
the statute is unambiguous and clear.
Since jurisdiction has been given to court by legislation, it is the legislation alone
which can take away the jurisdiction.
If any statute provides for an express bar of jurisdiction of a civil or other court,
then the scheme of the particular Act must provide adequate alternative remedies.
If the constitutionality of any provision is to be challenged, the writ of certiorari is
the only recourse.
There is no sympathy for legislative provisions which oust jurisdiction of courts,
because of the fact that the subjects are deprived of a remedy.
If jurisdiction is conferred to a tribunal, the intention of the parliament is presumed
to have jurisdiction to correct the decision of inferior tribunal.
Finality clause :
Many modern statutes contain provisions which attempt to take away the
jurisdiction of courts by making the decision of the tribunal final or conclusive.
HOWEVER, the remedy by certiorari is never to be taken away by any statute
except by the most explicit and clear words.
The word final means without an appeal. It does not mean without recourse to the
writ of certiorari.
It makes the decision final on fact but not on law.
In Dhulabhai v State of MP, the Supreme court held that
if a statute gives finality to the orders of a special tribunal, the jurisdiction of
civil court must be held to excluded only if there is an adequate alternate
remedy similar to what civil remedy would be.
In R v Medical Appeal Tribunal, Lord Denning said,
the word final only means without appeal and the remedy of certiorari cannot
be taken away because it is not an appeal.
Creating new and enlarging existing jurisdiction :
It is presumed that a statute does not create new jurisdiction or enlarge existing
jurisdiction.
Express language is required if an Act is to be so interpreted, as to create new
jurisdiction or enlarge existing jurisdiction.

https://www.facebook.com/LLB.GujUni/ http://duralex.bhatt.net.in/
https://www.facebook.com/groups/LLB.GujUni/ Page - 81 of 119

In Heathstar properties Ltd,


a statute giving power to grant relief on being satisfied on certain facts, does
not confer on it any power to grant interim relief until such fact had been fully
ascertained.
In State of UP v Mohammed Nooh,
in a departmental enquiry against the constable, the person holding the trial
offered to be a witness and prosecuted the constable. There was a gross
violation of the principles of natural justice. The court held that it can issue a
certiorari.

GO TO MODULE-3 QUESTIONS.
GO TO CONTENTS.

Discuss in detail : Presumption against what is inconvenient or absurd


Discuss in detail : Presumption against intending injustice
ANS :
Refer :
https://www.wcl.american.edu/journal/lawrev/44/dougherty.pdf
https://www.scribd.com/doc/18178617/Statutory-Construction-Notes
http://lawaids.blogspot.in/2010/06/presumption-against-intending-what-is.html
In construing a doubtful or ambiguous statute, the Courts will presumethat it was the
intention of the legislature to enact a valid, sensible and just law, and one which
should change the prior law no further than maybe necessary to effectuate the
specific purpose of the act in question.
Presumption that "a statute can not intend to do injustice" has its application in cases
where literal interpretation of statures results in absurity or injustice.
Statutes must receive a sensible construction such as will give effect to the legislative
intention so as to avoid an unjust and absurd conclusion.
Presumption against undesirable consequences are never intended by any legislative
measure.
Rokanas v. Co-operative Fire and Casualty Company, 1976 CanLII 244 (BC S.C.)
In determining either the general object of the legislature, or the meaning of its
language in any particular passage, it is obvious that the intention which appears
to be most in accord with convenience, reason, justice and legal principles should,
in all cases of doubtful significance, be presumed to be the true one.
An intention to produce an unreasonable result is not to be imputed to a statute

https://www.facebook.com/LLB.GujUni/ http://duralex.bhatt.net.in/
https://www.facebook.com/groups/LLB.GujUni/ Page - 82 of 119

if there is some other construction available.


When application of words literally would defeat the obvious intention of the
legislation and produce a wholly unreasonable result, we must do some violence
to the words so as to achieve that obvious intention and produce a rational
construction.
The question of inconvenience or unreasonableness must be looked at in the
light of the state of affairs at the date of the passing of the statute, not in the
light of subsequent events.
Literal Rule & its criticism :
Rule : If the precise words used are plain and unambiguous, in courts are bound to
construe them in their ordinary sense even though it too leads in our view of the
case to an absurdity or manifest injustice
There are certain defects of the literal rule of interpretation. The defects may be of
two types,
Logical defect which constitutes of ambiguity, inconsistency and incompleteness
and the
second type is absurdity or irrationality.
Ambiguity occurs where a term or an expression used in a statute has not one but
various meanings, and it is not clear which one particular meaning it represents at
which particular context or place. So here the court will have to go beyond the
statute and yet stick to the same literal words of the statute to ascertain its
meaning.
Also the ambiguity sometimes is "syntactic" which means the vagueness arises
from words like "or","and", "all" and other such words.
For example if a punishment for a certain crime is "fine or imprisonment or
both", the court can imprison the accused or impose a fine or impose a fine as
well as imprison him.
If the language of the statute is clear and unambiguous, according to "Literal Rule",
the Court cannot discard the plain meaning, even if it leads to an injustice.
Thus, the strict adherence to this principle may cause injustice and sometimes it
might give results which are quite contrary to general intention of the statute or
common sense.
Remedy :
Remedy for the lakuna in the "Literal Rule lies in the presumption that "a statute
can not intend to do injustice".
Because, legislators never intend to do injustice to the people. In all cases where,
plain reading of the statute results in manifest injustice or absurdity or repugnance
then the Golden Rule of statutory interpretation yields just and correct interpretat

https://www.facebook.com/LLB.GujUni/ http://duralex.bhatt.net.in/
https://www.facebook.com/groups/LLB.GujUni/ Page - 83 of 119

of the status.
Conclusion :
The presumption that "a statute can not intend to do injustice" is in the foundation
of the Golden Rule of statutory interpretation.

GO TO MODULE-3 QUESTIONS.
GO TO CONTENTS.

Discuss in detail : Presumption against impairing obligations or permitting advantage


from ones own Wrong
ANS :
Refer :
http://14.139.60.114:8080/jspui/bitstream/123456789/701/7/Modern%20Rules
%20of%20Statutory%20Interpretation.pdf
https://www.trans-lex.org/101100/_/cheng-bin-general-principles-of-law-as-
applied-by-international-courts-and-tribunals-reprinted-cambridge-1987/
The Latin maxim ex turpi causa non oritur actio [an action (suit) cannot be founded
on an immorai act] is thus applied to statutory interpretation, as it is applied in
relation to matters governed by uncodified law.
Generally, a person will not be allowed to take advantage of his own wrong.
Statutes relating to succession usually do not contain any provision excluding from
their benefit one who has killed the person to whose property succession is claimed.
But courts in most countries have "read" such an exception into such statutes
They will not allow the statute to be so applied as to enable the killer to succeed to
the estate of the person killed. The courts presume that the legislature had
impliedly assumed that certain rules of morality will be applied.
A famous American case (see Dworkin, Taking Rights Seriously, page 22) holds
that a grandson cannot succeed under a will of his grandfather murdered by the
grandson. "No one shall be permitted to profit by his own fraud, or to take advantage
of his own wrong or to found any claim upon his own inequity or to acquire property
by his own crime."
Lily Thomas Vs. Union of India & Ors., AIR 2000 SC 1650
The violators of law cannot be permitted to urge that their offence cannot be
subject matter of inquiry, trial or investigation.
The presumption also holds true in international law.
A State may not invoke its own illegal act to diminish its own liability.

https://www.facebook.com/LLB.GujUni/ http://duralex.bhatt.net.in/
https://www.facebook.com/groups/LLB.GujUni/ Page - 84 of 119

Commissioner Pinkney, in The Betsy Case (1797), called it


"the most exceptionable of all principles, that he who does wrong shall be at
liberty to plead his own illegal conduct on other occasions as a partial excuse."

GO TO MODULE-3 QUESTIONS.
GO TO CONTENTS.

Discuss in detail : Prospective operation of statutes


ANS :
Refer :
page-54 of
http://14.139.60.114:8080/jspui/bitstream/123456789/714/17/Principles%20of
%20Interpretation%20of%20Statutes.pdf
http://www.lawyersclubindia.com/articles/Interpretation-of-Statute-5430.asp
http://www.alameenlaw.in/modelpapers-dec2016.html
Introduction:
Prospective operation means from future date.
A central law passed by the Parliament commences either from the date it receives
Presidential assent or from a future date as specified in the statute. The Law sets
into motion only after the date of its commencement. Prior to this date, the law has
no force.
Retrospective operation means the operation from a past date. The parliament and
state legislature can legislate both prospectively and retrospectively. Where the
legislature intends to enact retrospective law, it should express the same in
unequivocal terms.
The power of retrospective legislation is used for validating the past executive and
legislative act. The defects due to which past acts are invalidated, are cured by way
of retrospective legislation.
Presumption against Retrospectively :
A law is deemed to be prospective in operation unless otherwise is intended either
expressly or by necessary implication. If the legislatures intends to give
retrospective effect to an enactment, such intendment has to be expressly
declared. In absence of such declaration the law is deemed to be prospective.
A retrospective law is one which takes away or impairs the vested or accrued rights
acquired under existing laws or creates new obligation, imposes new duty or
attaches new disability in respect of past events.

https://www.facebook.com/LLB.GujUni/ http://duralex.bhatt.net.in/
https://www.facebook.com/groups/LLB.GujUni/ Page - 85 of 119

The meaning of retrospective law :


I) Affecting in existing contract.
II) Reopening of past, closed and completed transaction.
III) Affecting accrued rights and remedies.
IV) Affecting procedure.
Examples :
Statutes dealing with procedure are presumed to be retrospective :
No person has vested right in course of procedure. He has only the right to
prosecution or defense in the manner prescribed. If the procedure is altered, he
has no other right than to proceed according to altered mode.
In New India Insurance Co. Ltd v. Shanti Misra, it was held that
a new Act requires certain type of original proceedings to be instituted before
a special tribunal to exclusion of civil courts, all proceedings of that type
whether based on old or new cause of action will have to be instituted before
the tribunal.
If the language of the statute is ambiguous and the word bear more than one
meaning and leading to alternative constructions, that interpretation should be
preferred which gives prospective application of law.
Statutes governing Succession are not retrospectively operated :
Statutes regulating succession are not applicable to succession which has
already opened.
By sec 14 of Hindu Succession Act, 1956 any property possessed by female
Hindu, whether acquired before after commencement of this Act, shall be held
by her as full owner thereof & not as limited owner.
The section on its own term is retrospective, but a female should have
possession of estate at the time the Act came into force. Section has not been
given retrospective effect larger than its language permits.
It was , therefore, been held that if a female Hindu had alienated the estate
prior to coming into force of the Act, neither she or her alienee get the right of
full ownership under the section.
Statutes dealing with substantive rights are presumed to be prospective
The statutes with deal with substantive rights are deemed to be prospective
only. They should not be presumed to be retrospective.
An enactment which seeks to take away any vested right cannot be given
retrospective operation unless it is expressly intended.
The civil judge had granted additional benefits over and above the award passed

https://www.facebook.com/LLB.GujUni/ http://duralex.bhatt.net.in/
https://www.facebook.com/groups/LLB.GujUni/ Page - 86 of 119

by collector, as per the amended provisions of the land acquisition act. The said
order was challenged before the high court. The court observed that
the award was passed by the collector on 3.3.1982 i.e. prior to 30.4.1982
when the land acquisition amendment will was introduced. Therefore the
respondents in whose favor award was already passed by the collector prior to
30.4.1982 are not entitled to the benefit of additional component. The benefit
could be given only if the proceedings for acquisition were pending and award
was not passed by the collector.
Likewise,
Statutes regulating transfers and contracts are not retrospective
Penal statutes which create offenses or which have the effect of increasing
penalties for existing offenses will only be prospective by reason of the
constitutional restriction imposed by article 20 of the constitution.
Conclusion :
The law is presumed to be of prospective application unless expressly or by
necessary implication made to have retrospective operation.
Penal, fiscal statutes and such other statutes which impairs vested rights or create
new obligations is usually given prospective operation.
No person has vested right in procedural law, thus it can be given prospective
operation.

GO TO MODULE-3 QUESTIONS.
GO TO CONTENTS.

https://www.facebook.com/LLB.GujUni/ http://duralex.bhatt.net.in/
https://www.facebook.com/groups/LLB.GujUni/ Page - 87 of 119

Module-4) INTERPRETATION WITH REFERENCE TO THE SUBJECT MATTER &


PURPOSE AND PRINCIPLES OF CONSTITUTIONAL INTERPRETATION :

4.1) Interpretation with reference to the subject matter & Purpose


4.1.1) Restrictive and beneficial construction
4.1.2) Taxing statutes
4.1.3) Penal statutes
4.1.4) Welfare legislation
4.2) Principles of Constitutional Interpretation :
4.2.1) Harmonious construction (also see para 1.5.5)
4.2.2) Doctrine of pith and substance
4.2.3) Colorable legislation
4.2.4) Doctrine of repugnancy
4.2.5) Prospective Overruling
4.2.6) Doctrine of Eclipse

GO TO CONTENTS.

MODULE-4 QUESTIONS :

Discuss in detail : Restrictive and beneficial construction.


Write short note : Welfare legislation.
Explain the rules of interpretation of taxing statute. (Apr-2012, Mar-2014, Apr-2016)
What is the difference between remedial and Penal statute? Explain in detail as to
how penal statutes have to be interpreted. (Apr-2012, Apr-2016)
"Remedial statutes require liberal interpretation where as penal statutes is strictly
construed. Explain. (Oct-2012, Apr2013, Mar-2014)
Discuss in detail : Principles of Constitutional Interpretation :
Write short note : Harmonious construction.(Discussed in MODULE-1)
Write short note : Doctrine of Pith and Substance. (Apr-2012, Oct-2012, Apr2013,
Mar-2014, Mar-2015)
Write short note : Doctrin of colourable legislation. (Oct-2012, Mar-2014)
Write short note : Doctrine of repugnancy.
Write short note : Doctrine of Prospective overruling (Apr2013, Mar-2014)

https://www.facebook.com/LLB.GujUni/ http://duralex.bhatt.net.in/
https://www.facebook.com/groups/LLB.GujUni/ Page - 88 of 119

Write short note : Doctrine of Eclipse.

GO TO CONTENTS.

MODULE-4 ANSWERS :

Discuss in detail : Restrictive and beneficial construction.


Write short note : Welfare legislation.
ANS :
Refer :
http://www.legalservicesindia.com/article/article/harmonious-and-beneficial-
construction-1941-1.html
http://hanumant.com/IOS-Unit7-BeneficialAndStrictConstruction.html
https://en.wikipedia.org/wiki/Strict_constructionism
http://legal-dictionary.thefreedictionary.com/strict+construction
http://www.encyclopedia.com/law/encyclopedias-almanacs-transcripts-and-
maps/strict-construction
Intro :
Judges are often called upon to make a construction, or interpretation, of an
unclear term in cases that involve a dispute over the term's legal significance. The
common-law tradition has produced various precepts, maxims, and rules that guide
judges in construing statutes or private written agreements such as contracts.
Eg when text of the statutes is ambiguous or prone to multiple meanings. Then
the court is required to construct the true meaning of the provision in the
context of the statute as well as over all system of law prevalent in the State.
There are two philosophies of construction :
strict or restrictive or narrow construction, and
liberal or beneficial or broad construction.
Welfare legislation :
<work on this>
Restrictive construction :
Restrictive constructionism refers to a particular legal philosophy of judicial
interpretation that limits or restricts judicial interpretation.
Restrictive construction occurs when ambiguous language is given its exact and
technical meaning, and no other equitable considerations or reasonable

https://www.facebook.com/LLB.GujUni/ http://duralex.bhatt.net.in/
https://www.facebook.com/groups/LLB.GujUni/ Page - 89 of 119

implications are made.


Note that, a judge may make a construction only if the language is ambiguous or
unclear. If the language is plain and clear, a judge must apply the plain meaning of
the language and cannot consider other evidence that would change the meaning.
If, however, the judge finds that the words produce absurdity, Ambiguity, or a
literalness never intended, the plain meaning does not apply and a construction
may be made.
In Criminal Law, strict construction must be applied to criminal statutes. This
means that a criminal statute may not be enlarged by implication or intent beyond
the fair meaning of the language used or the meaning that is reasonably justified
by its terms.
Criminal statutes, therefore, will not be held to encompass offenses and individuals
other than those clearly described and provided for in their language.
The strict construction of criminal statutes complements the rule of lenity, which
holds that ambiguity in a criminal statute should be resolved in favor of the
defendant.
Strict construction is the opposite of liberal construction, which permits a term to
be reasonably and fairly evaluated so as to implement the object and purpose of
the document.
Advocates of strict construction believe judges must exercise restraint by refusing
to expand the law through implication.
Critics of strict construction contend that this approach does not always produce
a just or reasonable result.
Narrow construction : Interpreting the Constitution based on a literal and narrow
definition of the language without reference to the differences in conditions when
the Constitution was written and modern conditions, inventions, and societal
changes.
By contrast "broad construction" looks to what someone thinks was the "intent"
of the framers' language and expands and interprets the language extensively to
meet current standards of human conduct and complexity of society.
Restrictive construction may also imply, (http://legal-
dictionary.thefreedictionary.com/strict+construction )
absolutely according to the letter, by chapter and verse, by the rules,
conservative interpretation, exactly as written, in a conservative interpretation,
in an orthodox interpretation, literally as written, plainly within the lannuage,
precisely as written, specific interpretation, strictly read, with a strict
interpretation, with fastidious rigidity, with hard and fast interpretation, with
inflexible interpretaaion, with literal interpretation, with meticulous rigidity, with

https://www.facebook.com/LLB.GujUni/ http://duralex.bhatt.net.in/
https://www.facebook.com/groups/LLB.GujUni/ Page - 90 of 119

narrow interpretation, with punctilious rigidity, with rigid interpretation, with


unyielding interpretation
Restrictive construction requires a judge to apply the text only as it is written. Once
the court has a clear meaning of the text, no further investigation is required.
Judgesin this viewshould avoid drawing inferences from a statute or
constitution and focus only on the text itself.
Justice Hugo Black (18861971) argued that the First Amendment's injunction,
that Congress shall make no law (against certain civil rights), should be construed
strictly: no law, thought Black, admits no exceptions. However, "strict construction"
is not a synonym for textualism or originalism. Antonin Scalia, a major proponent
of textualism, said that "no one ought to be" a strict constructionist.[2]
The term often contrasts with the phrase "judicial activism", used to describe
judges who seek to enact legislation through court rulings, although the two terms
are not actually opposites.
"Strict constructionism" is also used in American political discourse as an umbrella
term for conservative legal philosophies such as originalism and textualism, which
emphasize judicial restraint and fidelity to the original meaning (or originally
intended meaning) of constitutions and laws.

Beneficial construction :
A general rule of interpretation is that if a word used in a statute excludes certain
cases in its common meaning, it should not be constrained unnecessarily to include
those cases.
However, when the objectives of the statute are NOT met by excluding the
cases, then the word may be interpreted extensively so as to include those
cases.
Thus, when a word is ambiguous (ie if it has multiple meanings), which meaning
should be understood by that word?
This is the predicament that is resolved by the principle of Beneficial
Construction.
When a statute is meant for the benefit of a particular class, and if a word in the
statute is capable of two meanings {one which would preserve the benefits and
one which would not} then the meaning that preserves the benefit must be
adopted.
Note : Omissions will not be supplied by the court. Only when multiple meanings
are possible, can the court pick the beneficial one.
Thus, where the court has to choose between a wider mean that carries out the
objective of the legislature better and a narrow meaning, then it usually chooses

https://www.facebook.com/LLB.GujUni/ http://duralex.bhatt.net.in/
https://www.facebook.com/groups/LLB.GujUni/ Page - 91 of 119

the former.
Similarly, when the language used by the legislature fails to achieve the objective
of a statute, an extended meaning could be given to it to achieve that objective,
*IF* the language is fairly susceptible to the extended meaning.
Case law : In the case of B Shah vs Presiding Officer, AIR 1978, (i) Section 5
of Maternity Benefits Act, 1961 was is question, (ii) an expectant mother could take
12 weeks of maternity leave on full salary. In this case, a women who used to work
6 days a week was paid for only 6x12=72 days instead of 7x12=84 days.
SC held that the words 12 weeks were capable of two meanings and one
meaning was beneficial to the woman. Since it is a beneficial legislation, the
meaning that gives more benefit to the woman must be used.
It is said by MAXWELL, that Beneficial Construction is a tendency and not a rule.
The reason is that this principle is based on human tendency to be fair,
accommodating, and just.
Instead of restricting the people from getting the benefit of the statute, Court tends
to include as many classes as it can while remaining faithful to the wordings of the
statute.
Case law : In the case of Alembic Chemical Works vs Workmen AIR 1961,
an industrial tribunal awarded more number of paid leaves to the workers than
what Section 79(1) of Factories Act recommended. This was challenged by the
appellant.
SC held that the enactment being a welfare legislation for the workers, it had
to be beneficially constructed in the favor of worker and thus, if the words are
capable of two meanings, the one that gives benefit to the workers must be
used.
In U Unichoyi vs State of Kerala, 1963, the question was whether setting of a
minimum wage through Minimum Wages Act, 1948 is violative of Article 19 (1)
(g) of the constitution because the act did not define what is minimum wage and
did not take into account the capacity of the employer to pay.
It was held that the act is a beneficial legislation and it must be construed in
favor of the worker. In an under developed country where unemployment is
rampant, it is possible that workers may become ready to work for extremely
low wages but that should not happen.
Limitation On The Application Of Beneficial Construction : If on the application of
the rule of beneficial construction, the court finds that it is doing complete justice
and delivering a fair judgment then there is no question of why should not such
rule is applied? But there are certain restrictions which the court has to take care of
which at the time of application have to be adhered to
1. Where the courts find that by the application of the rule of beneficial

https://www.facebook.com/LLB.GujUni/ http://duralex.bhatt.net.in/
https://www.facebook.com/groups/LLB.GujUni/ Page - 92 of 119

construction, it would be re legislating a provision of statute either by


substituting, adding or altering any provision of the act.
2. Where any word in a statute confers to a single meaning only. Then the courts
should refrain from applying the rule of benevolent construction to the statute.
3. When there is no ambiguity in a provision of a statute so construed. If the
provision is plain, unambiguous and does not give rise to any doubt, the rule of
beneficial construction cannot be applied.
Difference between beneficial construction and harmonious construction :
Harmonious construction is only applied where there are a conflict between the
meaning coming out of two different sections and the meaning land the courts in
dubious situation of which section to apply ?
Whereas, when the rule of beneficial construction is applier, there is no conflict
between the meanings of any two sections and meanings attributed to them.
The rule of Beneficial Construction is applied in the cases where any construction
may do any benefit to the society or any group of people and are basically
applied in the socio economic legislations..

GO TO MODULE-4 QUESTIONS.
GO TO CONTENTS.

Explain the rules of interpretation of taxing statute. (Apr-2012, Mar-2014, Apr-2016)


ANS :
Refer :
http://lawtimesjournal.in/interpretation-of-statutes-important-questions/
http://hanumant.com/IOS-Unit7-BeneficialAndStrictConstruction.html
https://www.scribd.com/doc/7118910/Interpretation-of-Statutes
https://www.everycrsreport.com/reports/97-589.html
INTRODUCTION
Tax is the money collected from the people for the purposes of public works.
It is a source of revenue for the government. It is the right of the govt to collect
tax according to the provisions of the law. No tax can be levied or collected except
by the authority of law.
In general, legislature enjoys wide discretion in the matter of taxing statutes as
long as it satisfies the fundamental principle of classification as enshrined in Article
14.
Statutes imposing taxes or monetary burdens are to strictly construed. The logic

https://www.facebook.com/LLB.GujUni/ http://duralex.bhatt.net.in/
https://www.facebook.com/groups/LLB.GujUni/ Page - 93 of 119

behind this principle is that imposition of taxes is also a kind of imposition of


penalty which can only be imposed if the language of the statute unequivocally
says so. Any kind of presumption as to tax does not exist.
WHY taxing statute is to be construed differently from any other act ?
A person cannot be taxed unless the language of the statute unambiguously
imposes the obligation without straining itself. In that sense, there is no reason
why a taxing statute must be interpreted any differently from any other kind of
statute.
SC, in the case of CIT vs Shahazada Nand and Sons, 1966, observed that the
underlying principle is that the meaning and intention of a statute must be
collected from the plain and unambiguous expression used therein rather than
any notions which be entertained by the Courts as to what is just or expedient.
In construing a statutory provision the first and foremost rule of construction is the
literary construction. All that the court has to see at the very outset is what does
the provision say. If the provision is unambiguous and if from the provision the
legislative intent is clear, the court need not call into aid the other rules of
construction of statutes. The other rules of construction are called into aid only
when the legislative intent is not clear.
Lord Russel in Attorney General vs Calton Ban, 1989, illustrated categorically as, "I
see no reason why special canons of construction should be applied to any act of
parliament and I know of no authority for saying that a taxing statute is to be
construed differently from any other act."
However, as with any statute, a fiscal or taxing statute is also susceptible to human
errors and impreciseness of the language. This may cause ambiguity or vagueness
in its provisions. It is in such cases, the task of constructing a statute becomes
open to various methods of construction.
Since as a result of impoisition of tax, a person is compulsorily parted from his
money, a tax can be imposed only if the language of the provision unequivocally
says so.
This means that a taxing statute must be strictly constructed.
The principle of strict interpretation of taxing statutes was best enunciated by
Rowlatt J. in his classic statement in Cape Brandy Syndicate v I.R.C. -
"In a taxing statute one has to look merely at what is clearly said. There is no
room for any intention. There is no equity about a tax. There is no presumption
as to a tax. Nothing is to be read in, nothing is to be implied. One can look fairly
at the language used."
If by any reasonable meaning of the words, it is possible to avoid the tax, then that
meaning must be chosen.

https://www.facebook.com/LLB.GujUni/ http://duralex.bhatt.net.in/
https://www.facebook.com/groups/LLB.GujUni/ Page - 94 of 119

There is no scope for any inference or induction in constructing a taxing statute.


There is no room for suppositions as to spirit of the law or by way of inference.
When the provision is reasonably open to only one meaning then it is not open to
restrictive construction on the ground that the levy of tax, is oppressive,
disproportionate, unreasonable or would cause hardship. There is no room for such
speculation.
The language must be explicit.
Similarly, penalty provision in a taxing statute has to be specifically provided and
cannot be inferred.
Case law :
In A. V. Fernandes vs State of Kerala, AIR 1957, the Supreme Court stated the
principle that
if the revenue satisfies the court that the case falls strictly within the
provisions of the law, the subject can be taxed.
If, on the other hand, the case does not fall within the four corners of the
provisions of the taxing statute, no tax can be imposed by inference or by
analogy or by trying to probe into the intentions of the Legislature and by
considering what was the substance of the matter.
Taxing statutes and morality/ equity :
NOW, strict interpretation does not mean that equity/ morality and taxation are
complete strangers.
For example, in the case of CIT vs J H Kotla Yadgiri, 1985, SC held that
since the income from business of wife or minor child is includable as income of
the assessee, the profit or loss from such business should also be treated as the
profit or loss from a businesss carried on by him for the purpose of carrying
forward and set-off of the loss u/s. This interpretation was based on equity.
However, it does not permit any one to take the benefit of an illegality.
This is illustrated in the case of CIT vs Kurji Jinabhai Kotecha,AIR 1977, where
Section 24(2) of IT Act was constructed as not to permit assessee to carry forward
the loss of an illegal speculative business for setting it off against profits in
subsequent years.
This proves that even a taxing statute should be so construed as to be consistent
with morality avoiding a result that gives recognition to continued illegal activities
or benefits attached to it.
TAX AND FEE :
In case of a fee, there is a specific service rendered to the fee payer.
(Quid pro quo), whereas for the tax payer no direct services are rendered but

https://www.facebook.com/LLB.GujUni/ http://duralex.bhatt.net.in/
https://www.facebook.com/groups/LLB.GujUni/ Page - 95 of 119

the service assumes the form of public expenditure rendered to the public at
large.
Rules of interpretation
1. Charging Section :
The section that charges the tax must have clear words.
Before taxing any individual it must be clearly established that the person to be
taxed falls within the purview of the charging section by clear words.
There is no implication of the law. If a person cannot be brought within the four
corners of the law, he is free from tax liability.
Case law :
In Calcutta Jute Manufacturing Co. v Commercial Tax officer, the Supreme
Court held that in case of interpreting a taxing statute, one has to look into
what is clearly stated.
There is no room for searching the intentions, presumptions or equity.
In Mathuram Agarwal v State of Madhya Pradesh, the Supreme Court held
that
words cannot be added or substituted to find a meaning in a statute so as
to serve the intention of the legislature.
Every taxing statute must contain three aspects;
subject of tax,
person to be taxed and
the rate of tax.
2. Strict and favourable construction :
Taxing enactment should be strictly construed and the right to tax should be
clearly established.
Equitable construction should not be taken into account.
Courts should not strain words and find unnatural meaning to fill loopholes.
If the provision can be interpreted in two ways, then the one favoring the
assessee must be taken into consideration.
Case law :
In Saraswati Sugar mills v Haryana State Board, The Supreme Court held that
every Act of the parliament must be read according to the strict natural
construction of its words.
3. Clear Intention to impose or increase tax :
The intention to impose or increase tax or duty must be clear and in

https://www.facebook.com/LLB.GujUni/ http://duralex.bhatt.net.in/
https://www.facebook.com/groups/LLB.GujUni/ Page - 96 of 119

unambiguous language.
4. Prospective operation :
The cardinal principle of tax laws is that the law to be applied to assessee is the
law in force in the assessment year unless otherwise provided expressly or by
necessary implication.
No retrospective effect to fiscal statute is possible unless the language of the
statute is very clear and plain.
Fiscal Statute are generally not retrospective otherwise expressly provide by
necessary implications.
5. Meaning in common parlance :
In finding out the meaning of a taxing statute, the meaning in common usage,
parlance special in commercial and trade circles must be considered.
Case law :
In Annapurna Biscuit Manufacturing Co. v Commissioner of Sales tax, the
Supreme Court held that
statute imposing a tax should be construed in the way which they are
understood in ordinary language in the area in area in which the law is the
force.
In Dunlop India Ltd V Union of India, the Supreme Court held
latex comes within the meaning of rubber for the purpose of tax.
6. Imposition of Tax by authority of law :
The taxes and assessment can only be imposed by an authority established
under a statute. The tax can be levied only by an Act of the parliament.
Case law :
In Atlas Cycles Industries Ltd v State of Haryana, the Supreme Court held that
notification imposing a tax cannot be deemed to be extended to new areas
in the municipality.
7. Machinery provision :
Machinery provision means the procedure for calculation and collection of tax.
The person who claims an exemption has to establish it.
Case law :
In National Tag Traders v Commissioner of Income Tax, the Supreme Court
held that a fiscal statute must be construed strictly.
8. No presumption as to tax :
As regards to imposition of tax, no presumption exists. It cannot be drawn by

https://www.facebook.com/LLB.GujUni/ http://duralex.bhatt.net.in/
https://www.facebook.com/groups/LLB.GujUni/ Page - 97 of 119

implication or analogical extensions.


The presumption for equality and against partiality of taxation exists.
Case law :
In Mohammed Ali Khan v Commissioner of Wealth Tax, it was held that no tax
can be imposed by inference, analogy or probing into the intention of the
legislature.
9. Fiscal statute to be read as a whole :
The entire provisions of a fiscal statute has to be read as a whole and not in
piecemeal to find out the intent of the legislature.
Case law :
In Grasian Industries Limited v State of Madhya Pradesh, the Supreme Court
held that any exemption notification in a fiscal statute must be read in its
entirety and not in parties to find out the meaning.
10. No spirit of law :
A person is no liable to tax on the spirit of law or logic or reason.
11. Substance of matter :
The tax authorities must consider the legal aspect of a particular transaction for
levy of tax. This is called substance of the matter.
Illustrations :
Court fee Act : If the court fee is high, then it affects the right of the aggrieved
person to seek remedy. In interpreting the court fee Act, the benefit of doubt
always goes to the assessee.
Double taxation : In interpreting a fiscal statute, if one meaning gives rise to
double taxation and other meaning gives rise to single taxation, then the
interpretation must be in favour of single taxation.
Delayed payment of tax : Interest is levied by tax authorities on delayed payment
of tax. If provision exists, such delayed payment is valid.
Penalty- no criminal Act : The penalty provision cannot be equated with a criminal
statute as a criminal act requires mens rea.
Conclusion :
The general rule of construction is that in case of doubt, it is decided in favour of
the tax payer even if such a decision is detrimental to the government
The rule of strict construction applies primarily to charging provisions in a taxing
statute and has no application to a provision not creating a charge but laying down
machinery for its calculation or procedure for its collection. Thus, strict
construction would not come in the way of requiring a person claiming an

https://www.facebook.com/LLB.GujUni/ http://duralex.bhatt.net.in/
https://www.facebook.com/groups/LLB.GujUni/ Page - 98 of 119

exemption. The provisions of exemptions are interpreted beneficially.

GO TO MODULE-4 QUESTIONS.
GO TO CONTENTS.

What is the difference between remedial and Penal statute? Explain in detail as to
how penal statutes have to be interpreted. (Apr-2012, Apr-2016)
"Remedial statutes revise liberal interpretation where as penal statutes is strictly
construed. Explain. (Oct-2012, Apr2013, Mar-2014)
ANS :
Refer :
http://www.legalservicesindia.com/article/article/remedial-and-penal-statutes-
1470-1.html
https://www.scribd.com/doc/7118910/Interpretation-of-Statutes
Intro :
<Take general intro to interpretation of statutes from elsewhere in this doc>
Remedial Statutes :
Remedial statutes and statutes which have come to be enacted on demand of the
permanent public policy generally receive a liberal interpretation.
On constructing a remedial statute the courts ought to give to it the widest
operation which its language will permit. They have only to see that the particular
case is within the mischief to be remedied and falls within the language of the
enactment.
Eg The labour and welfare legislations should be broadly and liberally construed
and while construing them due regard to the Directive Principles of State Policy
(Part IV) and to any international convention on the subject must be given by the
courts.
In MC Mehta v. State of Tamil Nadu
the Child Labour (Prohibition and Regulation) Act, 1986 was construed as
remedial statute.
The Court, having regard to the Directive Principles in Arts 39(e), 39(f), 4(i), 45
and 47 of the Constitution, the fundamental rights in Art 24, the International
convention on the right of the child, not only directed a survey of child labour
and its prohibition but also directed payment of Rs. 25,000 as contribution by
the employer to the Child Labour-Rehabilitation-cum-Welfare Fund or alternative
employment to parent/ guardian of the child to ameliorate poverty and lack of
funds for welfare of the child which is the main cause of child labour.

https://www.facebook.com/LLB.GujUni/ http://duralex.bhatt.net.in/
https://www.facebook.com/groups/LLB.GujUni/ Page - 99 of 119

In case of a social benefit oriented legislation like the Consumer Protection Act
1986 the provisions of the consumer to achieve the purpose of the enactment but
without doing violence to the language. If a section of a remedial statute is capable
of two constructions, that construction should be preferred which furthers the
policy of the Act and is more beneficial to those in whose intrest the Act may have
been passed.
The liberal construction must flow from the language used and the rule does
not permit placing of an unnatural interpretation on the words contained in the
enactment nor does it permit the raising of any presumption that protection of
widest amplitude must be deemed to have been conferred upon those for whose
benefit the legislation may have been enacted.
Likewise, in case there is any exception in the beneficial legislation which curtails
its operation, the Court in case of doubt should construe it narrowly so as not to
unduly expand the area or scope of exception.
CrPC 1973 and Muslim Women (Protection of Rights on Divorce) Act 1986 :
In Noor Saba Khatoon v. Mohd Qasim, it was held that,
effect of a beneficial legislation is not construed to be defeated by a subsequent
legislation except through a clear provision.
Therefore, the rights of the minor children, irrespective of their religion, to get
maintenance from their parents as provided in 127 of the Criminal Procedure
Code 1973 was construed not to have been taken away in respect of Muslims by
the Muslim Women (Protection of Rights on Divorce) Act 1986.
3(b) of the MW (PRD) Act 1986 enables a divorced Muslim woman to claim
maintenance for the minor children up to the age of two years only from her
former husband.
It has been held that the right of children to claim maintenance under 125 Cr
PC is independent of the right of divorced mother to claim maintenance for the
infant children and the former is not affected by the Muslim Women Act 1986.
In Central Railway Workshop, Jhasi v. Vishwanath, the question before the court
was whether time-keepers, who prepared pay sheet of the workshop staff, maintain
leave account, dispose of settlement case and maintain records for other statistical
purposes, were workers as defined in the Factories Act 1948.
2 defined as a person employed directly or through any agency, whether for
wages or not in any manufacturing process used for a manufacturing process or
any other kind of work incidental to or connected with the manufacturing
process.
The court gave a liberal construction to the definition of worker and held that
time-keepers were workers being employed in a kind of work incidental to or
connected with the manufacturing process.

https://www.facebook.com/LLB.GujUni/ http://duralex.bhatt.net.in/
https://www.facebook.com/groups/LLB.GujUni/ Page - 100 of 119

In Motor Owners Insurance Co Ltd v. JK Modi, the words any one accident
occurring in 95(2) (a) of the Motor Vehicles Act 1939 was construed.
Having regard to the beneficial purpose of the Act, the words were construed to
signify as many accidents as the number of persons involved in the accident to
enable the limit of Rs. 20,000 payable by the insurance company to apply to
each person injured.
In Kuldip Kaur v. Surinder Singh, the Supreme court dealt with 125(3) of CrPC.
The court drew a distinction between mode of enforcement and mode of
satisfaction and held that even after a sentences of imprisonment, the person
concerned remained liable for arrests of maintenance for non-payment of which
he was imprisoned and the liability for payment could be satisfied only by
payment and not by suffering the sentence.
Penal Statutes :
The principle that a statute enacting an offence or imposing a penalty is to be
strictly construed is not of universal application which must necessarily be
observed in every case.
The rule was originally evolved to mitigate the rigour of monstrous sentences for
trivial offences and although that necessity and that strictness has now almost
vanished, the difference in approach made to a penal statute as against any other
statute still persists.
According to Lord Esher, the settled rule of construction of penal sections is that
if there is a reasonable interpretation which will avoid the penalty in any
particular case we must adopt that construction. If there are two reasonable
constructions we must give the more lenient one.
Interpretation of penal provisions must be in consonance with the principles
underlying fundamental rights.
Any provision which visits an accused with adverse consequences without
affording him any remedy to disprove an item of evidence which stands against
his innocence, is inconsistent with the philosophy enshrined in Art 21. It was
held by the Supreme Court that they should so interpret such a provision as to
dilute it to make it amenable to Art 21 of the Constitution.
When words employed in a penal statute are not clear the principle against double
penalisation would be applied. Failure to comply with a statute may attract penalty.
But only because a statute attracts penalty for failure to comply with the statutory
provisions, the same in all situations would not call for a strict construction.
This section provides for recovery of maintenance granted in favour of a wife or
minor child by issue of a warrant if the order for maintenance is not complied
with without sufficient cause and enables the magistrate, if the amount still

https://www.facebook.com/LLB.GujUni/ http://duralex.bhatt.net.in/
https://www.facebook.com/groups/LLB.GujUni/ Page - 101 of 119

remains unpaid to sentence the person against whom the order is made to
imprisonment for a period of one month.
An interpretation which strikes a balance between enforcement of law and
protection of valuable human right of accused (right of privacy) must be resorted
to.
105 of the Evidence Act 1872 says that the burden to prove that the case of
the accused falls within an exception to a statutory offence lies on him. But the
question whether the defence set up by an accused is really a defence of an
exception or a defence setting up non-existence of a fact which is an ingredient
of the offence to be proved by the prosecution depends upon the construction of
the particular statute.
In applying and interpreting a penal statute, public policy is also taken into
consideration.
In a recent case, the House of Lords held that consensual sadomasochistic
homosexual encounters which occasioned actual bodily harm to the victim were
assaults occasioning actual bodily harm, contrary to 47 of the Offences Against
the Person Act 1861 and unlawful wounding contrary to 20 0f the Act,
notwithstanding the victims consent to the acts inflicted on him.
The following are some of the propositions important in relation to strict
construction of penal statutes :
(a) if the scope of prohibitory words cover only some class of persons or some
well defined activity, their scope cannot be extended to cover more on
consideration of policy or object of the statute.
(b) prohibitory words can be widely construed only if indicated in the statute. On
the other hand if after full consideration no indication is found the benefit of
construction will be given to the subject.
(c) if the prohibitory words in their own signification bear wider meaning which
also fits in with the object or policy of the statute.
In JK (Bombay) Ltd v. Bharti Matha Mishra, it was held that
the expression officer or employee of a company applies not only to the existing
officer or employee but also includes past officers or employees where such an
officer or employee either
wrongfully obtains possession of any property, or
wrongfully withholds the same after the termination of his employment.
The expression would also include the legal heirs or representatives. It was held
by the court that the penal statutes should not be so liberally construed with the
aid of presumptions, assumptions and implications as to rope in for the purposes
of prosecution such persons against whom the prosecution is not intended by the

https://www.facebook.com/LLB.GujUni/ http://duralex.bhatt.net.in/
https://www.facebook.com/groups/LLB.GujUni/ Page - 102 of 119

statute because initiation of prosecution would be violative of Art 21 of the


Constitution and against public policy.
In Municipal Corpn of Delhi v. Laxmi Narain Tondon, the definition of sale in the
Prevention of Food Adulteration Act 1954 was construed in the sense having regard
to the mischief intended to be remedied. It was held that
the sale in the Act would include all commercial transactions where under an
adulterated article of food was supplied for consumption by one person to
another person.
Therefore, supply or offer of food to hotelier to a customer when consolidated
charge was made for residence and other amenities including food fell within the
definition.
In Tolaram v. State of Bombay, 18 of the Bombay Rents, Hotels and Lodging
Houses Rates (Control) Act 1947 was construed. This section provided that if any
landlord receives any fine, premium or other like sum or deposit or any
consideration other than the standard rent in respect of the grant, renewal or
continuance of a lease of any premise, such landlord shall be punished. It was
held by the Supreme Court that
the section did not prohibit the taking of money by owner of an incomplete
building in consideration.
Conclusion :
A statute may in certain aspects be a penal enactment and in certain others a
remedial one.
In respect of those provisions which are sanctioned on the pain of punishment for a
crime the rule of strict construction in the limited sense may be applied.
At any rate, as undue effort to construe such a provision liberally to promote the
beneficent purpose behind it may be effectively counter balanced on consideration
that a breach thereof leads to penal consequences.

GO TO MODULE-4 QUESTIONS.
GO TO CONTENTS.

Discuss in detail : Principles of Constitutional Interpretation :


ANS :
Refer :
http://lawtimesjournal.in/interpretation-of-statutes-important-questions/
http://hanumant.com/IOS-Unit8-Constitution.html
Introduction :

https://www.facebook.com/LLB.GujUni/ http://duralex.bhatt.net.in/
https://www.facebook.com/groups/LLB.GujUni/ Page - 103 of 119

Constitution is the supreme and fundamental law of our country.


Since it is written in the form of a statute, the general principles of statutory
interpretation are applicable to interpretation of the constitution as well.
As is the case with any other statute, the court tries to find out the intention of the
framers of the constitution from the words used by them.
For example, in the case of State of Bihar vs Kameshwar Singh AIR 1952, SC
used one of the standard principles of interpretation that where more than one
reasonable interpretation of a constitutional provision are possible, that which
would ensure a smooth and harmonious working of the constitution shall be
accepted rather than the one that would lead to absurdity or give rise to
practical inconvenience, or make well existing provisions of existing law
nugatory, while interpreting the constitution.
However, even if an argument based on the spirit of the constitution is very
attractive, it must be validated with the spirit of the constitution as reflected by
the words of the constitution. In the same case mentioned above, SC observed
that spirit of the constitution cannot prevail if the language of the constitution
does not support that view.
Note : ---> It is important to note that the constitution itself endorses the general
principles of interpretation through Article 367(1), which states that unless the
context otherwise requires, the General Clauses Act, 1897 shall apply for the
interpretation of this constitution as it applies for the interpretation of an act of the
legislature.
Courts have ruled in cases such as Jugmendar Das vs State 1951, that not only
the general definitions given in General Clauses Act, but also the general rules of
construction given therein are applicable to the constitution.
Having said the above, the fact remains that Constitution is a special act. It is a
fact that every provision of the constitution is constitutional and no part of it can be
held unconstitutional. This casts an important duty on the interpreters of the
constitution to interpret its provisions such that the spirit of the constitution is not
maligned.
In Keshvananda Bharati vs State of Kerala, AIR 1973, SC identified the basic
structure of the constitution that reflects its true spirit and held that nothing that
hurts the basic structure of the constitution, is constitutional.
In the same case, SC held that one should give the freedom to the parliament to
enact laws that ensure that the blessings of liberty be shared with all, but within
the framework of the constitution. It is necessary towards that end that the
constitution should not be construed in a narrow and pedantic sense.
The letters of the constitution are fairly static and not very easy to change but the
laws enacted by the legislature reflect the current state of people and are very

https://www.facebook.com/LLB.GujUni/ http://duralex.bhatt.net.in/
https://www.facebook.com/groups/LLB.GujUni/ Page - 104 of 119

dynamic.
To ensure that the new laws are consistent with the basic structure of the
constitution, the constitution must be interpreted in broad and liberal manner
giving affect to all its parts and the presumption must be that no conflict or
repugnancy was intended by its framers.
For example, applying the same logic,
the provisions relating to fundamental rights have been interpreted broadly and
liberally in favor of the subject.
Similarly, various legislative entries mentioned in the Union, State, and
Concurrent list have been construed liberally and widely.
The following are some of the key principles applied specially in interpreting the
provisions of the constitution
Principle of Harmonious construction
Doctrine of pith and substance
Doctrine of Colourable legislation
Principle of Ancillary powers
Principle of Occupied field
Residuary power
Doctrine of repugnancy
Principle of Territorial Nexus
Doctrine of stare decisis
Doctrine of prospective overruling

GO TO MODULE-4 QUESTIONS.
GO TO CONTENTS.

Write short note : Doctrine of Pith and Substance. (Apr-2012, Oct-2012, Apr2013,
Mar-2014, Mar-2015)
ANS :
Refer :
http://lawtimesjournal.in/interpretation-of-statutes-important-questions/
http://hanumant.com/IOS-Unit8-Constitution.html
Intro :
Pith means true nature or essence and substance means the essential nature
underlying a phenomenon.

https://www.facebook.com/LLB.GujUni/ http://duralex.bhatt.net.in/
https://www.facebook.com/groups/LLB.GujUni/ Page - 105 of 119

Thus, the doctrine of pith and substance relates to finding out the true nature of a
statute.
This doctrine is widely used when deciding whether a state is within its rights to
create a statute that involves a subject mentioned in Union List of the Constitution.
The basic idea behind this principle is that an act or a provision created by the
State is valid if the true nature of the act or the provision is about a subject that
falls in the State list.
The case of State of Maharashtra vs F N Balsara AIR 1951 illustrates this principle
very nicely.
In this case, the State of Maharashtra passed Bombay Prohibition Act that
prohibited the sale and storage of liquor. This affected the business of the appellant
who used to import liquor. He challenged the act on the ground that import and
export are the subjects that belong in Union list and state is incapable of making
any laws regarding it.
SC rejected this argument and held that the true nature of the act is prohibition of
alcohol in the state and this subject belongs to the State list. The court looks at the
true character and nature of the act having regard to the purpose, scope, objective,
and the effects of its provisions. Therefore, the fact that the act superficially
touches on import of alcohol does not make it invalid.
Thus, as held in State of W Bengal vs Kesoram Industries, 2004,
the courts have to ignore the name given to the act by the legislature and must
also disregard the incidental and superficial encroachments of the act and has to
see where the impact of the legislation falls. It must then decide the
constitutionality of the act.
Principle of Incidental or Ancillary Powers :
This principle is an addition to the doctrine of Pith and Substance. What it means is
that the power to legislate on a subject also includes power to legislate on ancillary
matters that are reasonably connected to that subject.
It is not always sufficient to determine the constitutionality of an act by just
looking at the pith and substance of the act. In such cases, it has to be seen
whether the matter referred in the act is essential to give affect to the main subject
of the act.
For example, power to impose tax would include the power to search and seizure to
prevent the evasion of that tax.
Similarly, the power to legislate on Land reforms includes the power to legislate on
mortgage of the land.
However, if a subject is explicitly mentioned in a State or Union list, it cannot be
said to be an ancillary matter.

https://www.facebook.com/LLB.GujUni/ http://duralex.bhatt.net.in/
https://www.facebook.com/groups/LLB.GujUni/ Page - 106 of 119

For example, power to tax is mentioned in specific entries in the lists and so the
power to tax cannot be claimed as ancillary to the power relating to any other
entry of the lists.
In the case of State of Rajasthan vs G Chawla AIR 1959, held
the power to legislate on a topic includes the power to legislate on an ancillary
matter which can be said to be reasonably included in the topic.
The underlying idea behind this principle is that the grant of power includes
everything necessary to exercise that power.
However, this does not mean that the scope of the power can be extended to any
unreasonable extent. Supreme Court has consistently cautioned against such
extended construction.
For example, in R M D Charbaugwala vs State of
Mysore, AIR 1962, SC held that betting and gambling is
a state subject as mentioned in Entry 34 of State list
but it does not include power to impose taxes on
betting and gambling because it exists as a separate
item as Entry 62 in the same list.

GO TO MODULE-4 QUESTIONS.
GO TO CONTENTS.

Write short note : Doctrin of colourable legislation. (Oct-2012, Mar-2014)


ANS :
Refer :
http://lawtimesjournal.in/interpretation-of-statutes-important-questions/
http://hanumant.com/IOS-Unit8-Constitution.html
Intro :
This doctrine is based on the principle that what cannot be done directly cannot be
done indirectly.
In other words, if the constitution does not permit certain provision of a legislation,
any provision that has the same effect but in a round about manner is also
unconstitutional. This doctrine is found on the wider doctrine of fraud on the
constitution.
A thing is Colourable when it seems to be one thing in the appearance but another
thing underneath.
Case laws :

https://www.facebook.com/LLB.GujUni/ http://duralex.bhatt.net.in/
https://www.facebook.com/groups/LLB.GujUni/ Page - 107 of 119

K C Gajapati Narayan Deo vs State of Orissa, AIR 1953 is a famous case that
illustrates the applicability of this doctrine.
In this case, SC observed that the constitution has clearly distributed the
legislative powers to various bodies, which have to act within their respective
spheres. These limitations are marked by specific legislatives entries or in some
cases these limitations are imposed in the form of fundamental rights of the
constitution.
Question may arise whether while enacting any provision such limits have been
transgressed or not. Such transgression may be patent, manifest or direct.
But it may also be covert, disguised, or indirect. It is to this later class of
transgression that the doctrine of colourable legislation applies. In such case,
although the legislation purports to act within the limits of its powers, yet in
substance and in reality, it transgresses those powers.
The transgression is veiled by mere pretense or disguise. But the legislature
cannot be allowed to violate the constitutional prohibition by an indirect method.
In this case, the validity of Orissa Agricultural Income Tax (Amendment) Act
1950 was in question. The argument was that it was not a bona fide taxation law
but a colourable legislation whose main motive was to artificially lower the
income of the intermediaries so that the state has to pay less compensation to
them under Orissa Estates Abolition Act, 1952.
SC held that it was not colourable legislation because the state was well within
its power to set the taxes, no matter how unjust it was. The state is also
empowered to adopt any method of compensation. The motive of the legislature
in enacting a law is totally irrelevant.
A contrasting case is of K T Moopil Nair vs State of Kerala, AIR 1961.
In this case, the state imposed a tax under Travencore Cochin Land Tax Act,
1955, which was so high that it was many times the annual income that the
person was earning from the land.
The SC held the act as violative of Articles 14 and 19(1)(f) in view of the fact
that in the disguise of tax a persons property was being confiscated.
Similarly, in Balaji vs State of Mysore, AIR 1963,
SC held that the order reserving 68% of the seats for students belonging to
backward classes was violative of Article 14 in disguise of making a provision
under Article 15(4).

GO TO MODULE-4 QUESTIONS.
GO TO CONTENTS.

https://www.facebook.com/LLB.GujUni/ http://duralex.bhatt.net.in/
https://www.facebook.com/groups/LLB.GujUni/ Page - 108 of 119

Write short note : Doctrine of repugnancy.


ANS :
Refer :
https://javidbagth.wordpress.com/2015/02/20/doctrine-of-repugnancy-and-
interpretation-of-constitution/
Introduction :
It is Article 254 of the Constitution of India that firmly entrenches theDoctrine of
Repugnancyin India. According to Blacks Law Dictionary, Repugnancy could be
defined as ---> an inconsistency or contradiction between two or more parts of a
legal instrument (such as a statute or a contract).
Before understanding the Doctrine of Repugnancy, let us first understand a bit
about the legislative scheme envisaged in our Constitution.
Article 245 states that Parliament may make laws for whole or any part of India
and the Legislature of a State may make laws for whole or any part of the State. It
further states that no law made by Parliament shall be deemed to be invalid on the
ground that it would have extra-territorial operation.
Article 246 also talks about Legislative power of the Parliament and the Legislature
of a State. It states that :
1. The Parliament has exclusive power to make laws withrespect to any of the
matters enumerated inList I or the Union Listin the Seventh Schedule.
2. The Legislature of any State has exclusive power to make laws for such state
with respect to any of the matters enumerated inList II or the State Listin the
Seventh Schedule.
3. The Parliament and the Legislature of any State have power to make laws
with respect to any of the matters enumerated in theList III or Concurrent List in
the Seventh Schedule.
4.Parliament has power to make laws with respect to any matter for any part of
the territory of India not included in a State notwithstanding that such matter is
a matter enumerated in the State List.The Legislative Scheme in our Constitution
is both complex and lengthy.
Supreme Courts Interpretation of Doctrine of Repugnancy : Article 254 has been
beautifully summarized by the Supreme Court in M. Karunanidhi v. Union of India.
The court said that:
1. Where the provisions of a Central Act and a State Act in the Concurrent
List are fully inconsistent and are absolutely irreconcilable, the Central Act will
prevail and the State Act will become void in view of the repugnancy.
2. Where however a law passed by the State comes into collision with a law
passed by Parliament on an Entry in the Concurrent List, the State Act shall

https://www.facebook.com/LLB.GujUni/ http://duralex.bhatt.net.in/
https://www.facebook.com/groups/LLB.GujUni/ Page - 109 of 119

prevailto the extent of the repugnancyand the provisions of the Central Act
would become void provided the State Act has been passed in accordance
with clause (2) of Article 254.
3. Where a law passed by the State Legislature while being substantially
within the scope of the entries in the State List, entrenches upon any of the
Entries in the Central List, the constitutionality of the law may be upheld by
invoking the doctrine of pith and substanceif on an analysis of the provisions
of the Act, it appears that by and large the law falls within the four corners of
the State List and entrenchment, if any, is purely incidental or
inconsequential.
4. Where, however, a law made by the State Legislature on a subject covered
by the Concurrent List is inconsistent with and repugnant to a previous law
made by Parliament, then such a law can be protected by obtaining the
assent of the President under Article 254(2) of the Constitution. The result of
obtaining the assent of the President would be that so far as the State Act is
concerned, it will prevail in the State and overrule the provisions of the
Central Act in their applicability to the State only. Such a state of affairs will
exist only until Parliament may at any time make a law adding to, or
amending, varying or repealing the law made by the State Legislature under
the proviso to Article 254. Now, the conditions which must be satisfied before
any repugnancy could arise are as follows :
That there is a clear and direct inconsistency between the Central Act and
the State Act.
That such an inconsistency is absolutely irreconcilable.
That the inconsistency between the provisions of the two Acts is of such
nature as to bring the two Acts into direct collision with each other and a
situation is reached where it is impossible to obey the one without
disobeying the other.
Thereafter, the court laid down following propositions in this respect :
That in order to decide the question of repugnancy it must be shown that the
two enactments contain inconsistent and irreconcilable provisions, so that
they cannot stand together or operate in the same field.
That there can be no repeal by implication unless the inconsistency appears
on the face of the two statutes.
That where the two statutes occupy a particular field, but there is room or
possibility of both the statutes operating in the same field without coming into
collision with each other, no repugnancy results.
That where there is no inconsistency but a statute occupying the same field
seeks to create distinct and separate offences, no question of repugnancy

https://www.facebook.com/LLB.GujUni/ http://duralex.bhatt.net.in/
https://www.facebook.com/groups/LLB.GujUni/ Page - 110 of 119

arises and both the statutes continue to operate in the same field.
In the case of Govt. of A.P. v. J.B. Educational Society, the court held that :
1. There is no doubt that both Parliament and the State Legislature are
supreme in their respective assigned fields. It is the duty of the court to
interpret the legislations made by Parliament and the State Legislature in such
a manner as to avoid any conflict. However, if the conflict is unavoidable, and
the two enactments are irreconcilable, then by the force of the non obstante
clause in clause (1) of Article 246, the parliamentary legislation would prevail
notwithstanding the exclusive power of the State Legislature to make a law
with respect to a matter enumerated in the State List.
2. With respect to matters enumerated in List III (Concurrent List), both
Parliament and the State Legislature have equal competence to legislate. Here
again, the courts are charged with the duty of interpreting the enactments of
Parliament and the State Legislature in such manner as to avoid a conflict. If
the conflict becomes unavoidable, then Article 245 indicates the manner of
resolution of such a conflict.
The Court also said that :
1.Where the legislations, though enacted with respect to matters in their
allotted sphere, overlap and conflict. Second, where the two legislations are
with respect to matters in the Concurrent List and there is a conflict. In both
the situations, parliamentary legislation will predominate, in the first, by
virtue of the non obstante clause in Article 246(1), in the second, by reason of
Article 254(1).
2.Clause (2) of Article 254 deals with a situation where the State legislation
having been reserved and having obtained Presidents assent, prevails in that
State; this again is subject to the proviso that Parliament can again bring a
legislation to override even such State legislation.
In the case of National Engg. Industries Ltd. v. Shri Kishan Bhageria, it was held
that
the best test of repugnancy is that if one prevails, the other cannot prevail.

GO TO MODULE-4 QUESTIONS.
GO TO CONTENTS.

Write short note : Doctrine of Prospective overruling (Apr2013, Mar-2014)


ANS :
Refer :
https://www.lawteacher.net/free-law-essays/administrative-law/jurisprudence-

https://www.facebook.com/LLB.GujUni/ http://duralex.bhatt.net.in/
https://www.facebook.com/groups/LLB.GujUni/ Page - 111 of 119

prospective-overruling-in-reference-administrative-law-essay.php
http://www.legalserviceindia.com/articles/prul.htm
Intro :
Doctrine of Prospective Overruling originated in the American Judicial System.
The literal meaning of the term overruling is to overturn or set aside a
precedent by expressly deciding that it should no longer be controlling law.
Similarly prospective means operative or effective in the future.
So, combined together, prospective overruling means construing an earlier
decision in such a way that it would not have a binding effect to the parties of
the original suit or to the cases decided on the basis of that judgment, and yet
changing the law, applying it only prospectively to the future cases.
For example, if principle A is laid down in the case of X v. Y and later on the court
disagrees with the Principle A, it changes the principle prospectively without
affecting the judgment of X v. Y and thus the new principle will apply only to the
future cases.
Prospective Overruling in India : Golak Naths Case :
The doctrine of prospective overruling was for the first time adopted in the case of
Golak Nath v. State of Punjab [5] (herein referred as Golaknaths case). Since then
it has been applied in many case laws and has also been a point of debate of many
jurists. Through this article, an attempt is made at briefly analyzing the stand of
the Indian Judiciary on adopting the doctrine of Prospective overruling.
It is very important in this context to analyze the holding of the Judiciary in
Golaknaths case. In Golak Nath case the doctrine is defined as :
The doctrine of "prospective overruling" is a modern doctrine suitable for a fast
moving society. It does not do away with the doctrine of state decision but
confines it to past transactions.
While in Strict theory it may be said that the doctrine 'involves the making of
law, what the court really does is to declare the law but refuse to give
retroactivity to it.
It is really a pragmatic solution reconciling the two conflicting doctrines, namely,
that a court finds the law and that it does make law and it finds law but restricts
its operation to the future.
It enables the court to bring about a smooth transition by correcting, its errors
without disturbing the impact of those errors on past transactions.
By the application of this doctrine the past may be preserved and the future
protected. Our Constitution does not expressly of by necessary implication speak
against the doctrine of prospective overruling."

https://www.facebook.com/LLB.GujUni/ http://duralex.bhatt.net.in/
https://www.facebook.com/groups/LLB.GujUni/ Page - 112 of 119

Because it was the first time that the Court was applying a doctrine which had
evolved in a different system of law so the Court laid down certain provisions
restricting the application of the doctrine in the Indian system. It was laid down
that
(1) The doctrine of prospective overruling can be invoked only in matters arising
under our Constitution;
(2) It can be applied only by highest court of the country, ie. The Supreme Court
as it has the constitutional jurisdiction to declare law binding on all the Courts as
it has India;
(3) The scope of the retrospective operation of the law declared by the Supreme
Court superseding its earlier decisions is left to its discretion to be moulded in
accordance with- the justice of the cause or matter before it.
Difference in application of the doctrine in USA and India :
It is pertinent to note that the doctrine of prospective overruling, which has its
roots in the American judicial system and from where the import was drawn from in
the Golaknath case, has been applied in a very narrow manner by Justice Subba
Rao.
In the case of Golaknath, it had been used for invalidating constitutional
amendments which had been in force for a long time and which in turn had become
the basis of mass legislation affecting agrarian economy.
In contract, in U.S.A., this doctrine had been applied in cases, as seen above in
case of changes in judicial views as regards the scope and interpretation of
constitutional provisions generally.
Also, one more distinction lies in the application vis--vis the invalidation effect.
In United States, the doctrine was used to hold the impugned law invalid from
the date of the decision and not earlier.
But, in Golaknath, all the constitutional amendments were to remain valid for
ever; only the principle of non-amendability of fundamental rights was to apply
in future.
If the American doctrine had been strictly imported into India, then the
constitutional amendments would have been declared invalid from the date of the
judgment. Therefore, one sees that the Supreme Court has diluted the application
of the doctrine based on the needs and the social scenario prevalent at that time.
Present position : Keshvananda bharathi
The Supreme Court in the landmark case of Keshvananda bharathi v. State of
Kerala held that the Parliament under the Indian Constitution is not supreme, in
that it cannot change the basic structure of the constitution. It also declared that in
certain circumstances, the amendment of fundamental rights would affect the basic

https://www.facebook.com/LLB.GujUni/ http://duralex.bhatt.net.in/
https://www.facebook.com/groups/LLB.GujUni/ Page - 113 of 119

structure and therefore, would be void.


Thus, one can see that this case is drawn on a larger canvass as compared to that
of Golaknath.
It also overruled Golaknath and thus, all the previous amendments which were
held valid are now open to be reviewed.
They can also be sustained on the ground that they do not affect the basic
structure of the constitution or on the fact that they are reasonable restrictions on
the fundamental rights in public interest. Both the cases, is seen closely, bear the
same practical effects.
What Golaknath said was that the Parliament cannot amend so as to take away the
fundamental rights enshrined in Part III,
whereas in Keshavananda, it was held that it cannot amend so as to affect the
basic structure.
As we all know, the basic structure is a figment of judicial imagination. So what
exactly constitutes basic structure cannot be clearly underlined. The above case
has laid that down and as a matter of fact, we all are bound by it as it is the law as
of today.

GO TO MODULE-4 QUESTIONS.
GO TO CONTENTS.

Write short note : Doctrine of Eclipse.


ANS :
Refer :
https://www.scribd.com/document/242105632/Doctrine-of-eclipse
Intro :
Supreme Court on Record Association v. Union of India, AIR 1994 SC 268: (1993) 4
SCC 441
Constitution is the will of the people whereas statutory laws are the creation
of legislators who are the elected representatives of the people. Where the will of
the legislature declared in statutes stand in opposition to that of the people
declared in the Constitution the will of the people must prevail.
Definition of Doctrine of Eclipse :
It provides that a law made before the commencement of the constitution
remains eclipsed or dormant to be extent in comes under the shadow of
fundamental rights

https://www.facebook.com/LLB.GujUni/ http://duralex.bhatt.net.in/
https://www.facebook.com/groups/LLB.GujUni/ Page - 114 of 119

i.e., is inconsistency brought about by the fundamental rights is removed by the


amendment to the Constitution of India.
Discussion :
In giving to ourselves the Constitution, We the People have reserved the
fundamental freedoms to ourselves. This reservation was incorporated by framers
of our Constitution in Article 13.
Art-13 is not only the source of the protection of fundamental rights, but also the
expression of reservation. This article ensures that the validity of all laws would be
tested on the touch stone of the Constitution.
It provides that all laws made before the coming into force of this Constitution and
any future law enacted by any authority in India have to stand the test for their
validity on the provision of Part III of this Constitution.
The scope and extent of article 13 of the Constitution has been a matter of debate
in various cases and has resulted into conflicting decisions of the Supreme Court.
In the quest of effectuating this Article, Courts have evolved various doctrines like
the doctrines of severability, prospective overruling, and acquiescence
The Doctrine of Eclipse is one such principle, based on the premise that
fundamental rights are prospective in nature. As a result of its operation, "an
existing law inconsistent with a fundamental right, though it becomes inoperative
from the date of commencement of the Constitution, is not dead altogether."
Hence, in essence, the Doctrine seeks to address the following
The doctrine also raises some profound questions about legislative competence and
the interference of courts in law making.
DOCTRINE OF ECLIPSE :
This doctrine is applicable to pre-existing laws only. It implies that an existing law
which violates fundamental right is not dead or void per-se but only becomes
unenforceable.
"It is over shadowed or eclipsed by the fundamental rights and remains dormant
but it is not dead." If by subsequent amendment fundamental rights are
amended in such a way as to give way to these laws, then these laws will again
become active.
A post-Constitutional law which abrogates or takes away fundamental rights will be
void ab initio as against the persons on whom such rights have been conferred.
A post-Constitutional law cannot be revived by a subsequent Constitutional
amendment.
Case laws :
In Sagir Ahmed v. State of U.P., a Constitution Bench of the Apex Court

https://www.facebook.com/LLB.GujUni/ http://duralex.bhatt.net.in/
https://www.facebook.com/groups/LLB.GujUni/ Page - 115 of 119

unanimously stated that


the Doctrine could not be applied to the impugned post-Constitutional law.
A legislation that contravened Article 19(1) (g) and was not protected by clause
(6) of the Article, when it was enacted after the commencement of the
Constitution, could not be validated even by subsequent Constitutional
amendment.
However, in State of Gujarat v. Shree Ambica Mills, the Supreme Court (Matthew,
J.) held that
like a pre-Constitutional law, a post Constitutional law contravening a
fundamental right could also be valid in relation to those, whose rights were not
infringed upon. For instance, when a post Constitutional law violates a
fundamental right like Article 19 which is granted to citizens alone, it would
remain valid in relation to non-citizens.
ie ias regards citizens whose rights were infringed, the law must be regarded as
stillborn and void ab initio, and therefore, in order to make it apply to citizens,
the law would have to be re-enacted afresh.
Conclusion :
A statute (post constitutional) held unconstitutional cannot be revived except by re-
enactment, a statute (pre constitutional) under eclipse can be revived by
obliteration of the limitations generating the taint of unconstitutionality.

GO TO MODULE-4 QUESTIONS.
GO TO CONTENTS.

https://www.facebook.com/LLB.GujUni/ http://duralex.bhatt.net.in/
https://www.facebook.com/groups/LLB.GujUni/ Page - 116 of 119

Module-5) Practical Section ---> These are not part of the text of syllabus.

MODULE-5 QUESTIONS :

The Ahmedabad Municipal Corporation (AMC) framed rules for entry into Kids Zone at
Kankaria Lake. One of the Rules provides as under : (Apr-2012)
"No person will be allowed entry with his two wheeler and four wheeler vehicles".
"A" wants to enter the Kids Zone with his rickshaw having 3 wheels. According to him
entry of only two wheelers and four wheelers is prohibited and entry of his vehicle
being a three wheeler is not prohibited. According to AMC, the purpose of the Rule is
to prohibit entry of any vehicle so that kids can safely enjoy the Kids Zone. Can A
enter Kids Zone with his rickshaw Give reasons in support of your answer.
Assume that under the Customs and Excise Act, the Customs Duty is leviable on import
of gold in India. Gold is not defined under the Act. (Apr-2012)
"A" brings a designer golden spectacle frame with 22 carats gold in it from USA to
India. The Customs Department wants to levy the Customs Duty on the golden frame
as it contains gold.
According to "A", it is popularly known as "frame" and not "gold". Is "A" liable to pay
Customs Duty ? Would your answer be different if 'A' brought 22 carat golden chain to
India ? Give reasons in support of your answers.

GO TO CONTENTS.

MODULE-5 ANSWERS :

The Ahmedabad Municipal Corporation (AMC) framed rules for entry into Kids Zone at
Kankaria Lake. One of the Rules provides as under : (Apr-2012)
"No person will be allowed entry with his two wheeler and four wheeler vehicles".
"A" wants to enter the Kids Zone with his rickshaw having 3 wheels. According to him
entry of only two wheelers and four wheelers is prohibited and entry of his vehicle
being a three wheeler is not prohibited. According to AMC, the purpose of the Rule is
to prohibit entry of any vehicle so that kids can safely enjoy the Kids Zone. Can A
enter Kids Zone with his rickshaw Give reasons in support of your answer.
ANS :
Refer :

https://www.facebook.com/LLB.GujUni/ http://duralex.bhatt.net.in/
https://www.facebook.com/groups/LLB.GujUni/ Page - 117 of 119

GO TO MODULE-5 QUESTIONS.
GO TO CONTENTS.

Assume that under the Customs and Excise Act, the Customs Duty is leviable on import
of gold in India. Gold is not defined under the Act. (Apr-2012)
"A" brings a designer golden spectacle frame with 22 carats gold in it from USA to
India. The Customs Department wants to levy the Customs Duty on the golden frame
as it contains gold.
According to "A", it is popularly known as "frame" and not "gold". Is "A" liable to pay
Customs Duty ? Would your answer be different if 'A' brought 22 carat golden chain to
India ? Give reasons in support of your answers.
ANS :
Refer :

GO TO MODULE-5 QUESTIONS.
GO TO CONTENTS.

https://www.facebook.com/LLB.GujUni/ http://duralex.bhatt.net.in/
https://www.facebook.com/groups/LLB.GujUni/ Page - 118 of 119

Module-6) Case-laws : -- These are not part of the text of syllabus.


Neverthless, questions on landmark cases are asked regularly.

MODULE-6 QUESTIONS :

Discuss briefly rules propounded in Menaka Gandhi V/s. Union of India. (Apr-2012, Oct-
2012, Apr-2016)
Discuss briefly rules propounded in Ashvinikumar V/s. Arvind Bose. (Apr-2012)
Discuss briefly rules propounded in Rajnarayan V/s. Mrs. Indira Gandhi. (Apr-2012, Oct-
2012, Apr2013, Apr-2016)
Discuss briefly rules propounded in Golaknath V/s. State of Punjab. (Apr-2012, Oct-
2012, Apr-2016)
Discuss briefly rules propounded in Keshwanand Bharati V/s. State of Kerala. (Apr-
2012, Apr2013, Apr-2016)
Discuss briefly rules propounded in Madhavrao Scindia Vls. Union of India. (Apr-2012)

MODULE-6 ANSWERS :

For answers to above questions, please refer to the notes on subject 214K
Legal Principles through Case Study.

GO TO CONTENTS

*** End-of-Compilation ***


Source : Public domain print/ internet contents.
URLs of some such resources are listed herein above.
Credits/ copyrights duly acknowledged.

Suggested Reading :
G.P. Singh, Principles of Statutory Interpretation, Wadhwa, Nagpur.
P. St. Langan (Ed), Maxwell on The Interpretation of Statutes, N.M. Tripathi, Bombay.
K. Shanmukham, N.S. Binclras Interpretation of Statutes, The Law Book Co. Allahabad.
V. Sarathi, Interpretation of Statutes, Eastern, Lucknow.
M.P. Jam, Constitutional Law of India, Wadhawa & Co M.P. Singh (Ed), V.N. Shukla's

https://www.facebook.com/LLB.GujUni/ http://duralex.bhatt.net.in/
https://www.facebook.com/groups/LLB.GujUni/ Page - 119 of 119

Constitution of India, Eastern, Lucknow.


U. Baxi, Introduction to Justice K.K. Mathews Democracy Equality and Freedom
Eastern, Lucknow.

https://www.facebook.com/LLB.GujUni/ http://duralex.bhatt.net.in/

Você também pode gostar